Sei sulla pagina 1di 242

TEXT FLY WITHIN THE BOOK ONLY

<> W

c^ DO ^
>[g ""

u<OU_1 64808
CD

^D

CHAPTER I
i.

Definitions.

Scalar

and Vector Quantities.


physicists

The mathematicians and

have got to deal

with two different kinds of quantities. Some of them are specified by a single real number called the magnitude
or the measure of that quantity and are not related to any Such quantities aie Scalars. The direction in space.

examples of such quantities are mass,


charge, temperature, density etc. etc.

volume, electric Thus the mass of a


to the

body may be

specified

by a

positive

number m equal

ratio of the given

mass

to the unit mass.

Thus

for scalars

we

should have a unit of

measurethe

ment and a

real

number m that expresses the

ratio of

given scalar quantity to that of the unit.

There are however other types

of

quantities

which

have got magnitude as well as a definite direction in space. Such type of quantities are called vector quantities
or simply vectors. The most familiar examples of this are velocity, acceleration, force, displacement etc. type If we say that the speed of a train is 45 m. p. h., our

statement is not complete so long as we do not specify the direction in which the train is moving. Similatly we cannot content ourselves by simply giving the magnitude
of a force;
in

we have got to specify the particular direction which it acts. Thus a scalar quantity cannot completely
a vector quantity.

specify

Vector Analysis

Representation and Notation qf Vectors*


Symbolically a vector is often denoted by two letters with an arrow over them; the tail of the arrow is called the origin of the vector whereas its head is called the

terminus
origin
is

or the end.

Thus

in the vector

AB A
of
is

is

called the

and B the terminus.

The magnitude
its

the vector
to
J5.

given by the length

AB and
may

direction

from A

Such
vectors*

vectors are called line

Thus a

vector

be

represented by a directed line segment i.e. a given portion of

a given line on which

the

two

Fig.

1.

end points origin and the terminus are specified, i.e. they cannot be interchanged, for it will change the direction
of the? vector.

In addition to the above notation of vectors by giving their origin and terminus ws shall use sigle letters

(Clarendon

letters)

in

heavy (bold
letters

face)

The corresponding
tude of the vectors.

italic

a, b, c

type like a, b, c* denote the magni-

then

Thus
by
a.

if

^B=a,

AB
|

i.e.
\

its

magnitude

is

denoted

printing of the books is concerned but it is quite inconvenient for the teachers and the student to use the above notation on the

The above

notation

is all

right so far as the

black-boards and their note-books respectively. Alternatively we may adopt the Greek letters a, /3, y, 8 to denote the
vectors and the letters
respectively.
a, b, c 9

When we

d to denote their magnitudes have to deal with large number of

Introduction
vectors

3 F. .for

use the capital letters A, B, C, D, vectors and the corresponding small letters a, b, c,

we may

for their
i.e.

magnitudes or by
IT,

placing bars on
of

d, *,/... the letters

by V,

etc.

In this book,

however, we shall
bold-face
italic

follow the standard


letters for vectors

notations

using

typed
for

and the corresponding


**

letters

their magnitudes.

Free and Localised Vectors.

When we
vector, at

are at

liberty
it is

to choose

any point, then


restricted to

said to be

the origin of the a free vector, but

when
vector

it is

is

a certain specified point, then the said to be a localized vector.

Equal Vectors. Two vectors are said to be equal when they have the same length (magnitude) and the same direction and the equality of two vectors is written as usual a=b. Thus equal vectors may be represented by parallel lines of equal length drawn in the same sense of
direction irrespective of the origin.

Like Vectors.

Vectors are said to be

like

when they

have the same sense of direction.

be collinear

Collinear Vectors. Any number of vectors are said to when they are parallel to the same line what-

ever their magnitudes

may

be.
of vectors

Coplanar Vectors.
to be coplanar

Any number

are said
plane.

when they

are parallel to the

same

Co-initial Vectors.
it

vector
in

about parallel to

itself

not altered by shifting Hence any vector space.


is

a=AB may

be drawn from any

moving the segment

AB

parallel to

itself

assigned origin by so that the point

Vector Analysis
coincides with

and the point

falls

on some other point

be replaced by

say

P;

then AB=*OP*=a.

In this
vectors

way

all

vectors in

space

may

drawn from the same assigned origin by moving them parallel to themselves till their origin coincides with All such vectors which have the same assigned orgm 0. the same point as the origin are called co-initial
vectors*

Zero Vector.
vector
coincide,

If

then

the origin and terminal points of a it is said to be a zero vector.

nate.

Evidently its length is zero and its direction is indetermiA zero vector is denoted by the bold face typed o.
are equal and they can

All zero vectors

be expressed as

->

AA,

BB

etc.

Unit Vector.
its

A
a,

vector

is

said to be
If

a unit vector

if

magnitude be of unit length.


is

a whose module

any vector then the corresponding unit vector

there be

A
in that direction is

denoted by a which

has

its

magnitude

unity.

Thus we have a = ia

A
or

a=

a
.

Note.

be equal so long as are also the same.


equal. Again
if

Any two unit we do

vectors

may

not be confused to

not ascertain that their directions


like unit vectors are

Thus only two


say that

A A
we
a=b,
it

would mean that

a=b

and that afc, the first relation specifying same direction and the second one specifying equal magnitude.
Position vector.

The

position vector of

any point P

with reference to an origia


is

the vector OP.

Thus

taking

Introduction
as origin
in space.

r there

is

we can find the position vector of every point Conversely, corresponding to any given vector -> a point P such that 0P=r.
whose direction is the a given vector a but whose magnitude is
vector
of the

Reciprocal vector.

same

as that of

reciprocal of the magnitude

given vector

is

called

the reciprocil of a and

is

written as a~*.

Thus

aaa.

..

reciprocal vector r

a""

t 1

A A

A
<x.a

a=

a2

-.

a =--a*
is

Now since
whose

the magnitude of a unit vector reciprocal is again a unit, we conclude

a unit

that the

reciprocal of a unit vector is the unit vector itself.

Negative vector.

vector whose

magnitude

is

the
is

as that of a given vector a but opposite direction called the negative vector of a and is written as a. If

same

is

then

represented by
a.

OA

-a

is

represented by AO.

Addition of vectors.

Let there be any two given vectors a and


b.

Now

point vectors

choose any and draw the


so that

a and b

the terminus of a coincides with the origin of

->

Fig. 2

->

Then

the vector given

by OB

is

defined as the

sum

of

Vector Analysis

the given vectors

a and b and

is

written as

The above sum


position of 0.
It

independent of the choice for the should be clearly understood here that
is

->
the magnitude of

OB

is

not equal to

sum

of

the

magnitudes
is

of

OA

and

AB

as

we know

that any side of a triangle

less

than the sum of the other two.

OB
~

will only

be equal to the >

The magnitude of vector sum of the magnitudes of


the

OA and OB when OA and OB have


vector

same

direction.

The

OB
-

represents the combined effect -

(i.e.

resultant) of

vector

OA and OB.
a-+b~0

The above law


when
is

is

called

triangle

law

of addition by which the vector quantities are compounded,


If
i.e.

and

coincide then

b=-b+a
length as

showing that

-a

is

a vector which has the same


opposite to that of a.

a but whose

direction

Vector addition
If

is

Commuative,

i.e.

a-fb=*b+a.

(M=a

and AB=*b,

then

OBs=a+b.
the

Now

complete

parallelogram whose two sides are OA and AB.

Since the opposite sides of a parallelogram are equal and parallel, we can say that

and AB**OC**b.

Introduction

or

a-fb=b+a. ^
Vector addition
i.

is associative

e.

(a+b)+c=:a+(b+c):
and

Let

OA

OA^c.
Join

to the

terminus of

last vector C.

-*

Similarly

is

the

Thus we see that (a+b)+c=OC-a+(b4-c) where OC sum of the three given vectors and is written as
Thus
the

a+b+c.
sum of any number of
vectors is

independent of the
to

order in which they are added

and of their grouping

form partial

sums.

Sum of any number


sum
line

of vectors.
vectors

If

we

are to find the

of

any

number

of

a, b, c, d,

e say, then form a broken


represent
in length and these vectors. -

whose segments

direction

In the above figure


>

OA= a, AB=b,
joining
terhiinal

BCssc,

etc.;

then the vector


first

the origin of

vector to

8
point
of

Vector Analysis
the last vector will represent the
i. e.

vactor

sum

a+b+c+d+e

OA+AB+BC+CD+DE=OE.
will

The above sum

be zero

if

the terminal point


first

of

the last vector coincides with the origin of the

vector.

04+45+ J5C+CZ)+Z)0=>0.
From above we
observe that
or

04+44=04
Now 44=o
EX.I,
quadrilateral,
the

00+04=04.
if

and

00=o
d

and

we put 04=a,

then

a+o=a and o + a=a.


7/a^b,
c,

represent

the

consecutive
sufficient

sides

of a
that

show that necessary and quadrilateral be a parallelogram is that

condition

a+c

and that this

implies

b+d=0.

Since the^ origin of the first vector coincides with the end point
of the last vector,

we have

AB+BC+CD+DA^Q
or
If

a+b+c+d=0.

..

(1)

the figure be a parallelogram, then 4J3=CDand they are parallel.

/.

4Bs=

CD

as they are in opposite

directions

or

a= -c

a+c=0. Hence from giving that BC and DA are


i. e.

(i),

we

get

b+d=0.

:.

C= - DA

figure
it

is

parallelogram.

We

equal and parallel. Hence the can prove the converse of

easily.

Introduction

3,

Subtraction of vectors ,

have already defined negative vectors as a vector the same magnitude as that of a given vector but having opposite direction.

We

Thus

if

AB~b,

then AC=*

b,

where

AC=AB.
The
traction of

operation of vector subtwo given vectors a and

b may
of

be regarded as the operation


of

addition

vecctor

a and

-b

and written as

a-b=-a+(-b).
Similarly,

b-a=b + (-a).
Fig.No.7

Expression of a vector in terms of the position


vectors of its end points.
Let us choose any point as origin and the position
vectors of
the

extremities

A and B
respect

of a vector

AB

with

to

this

origin be

a
.

and b; then

Similarly

BA~BO+OA~OA+(-OB)

10

Vector Analysis

Ex.

2.

//"a

and

are the

vectors

forming consecutive sides


the vectors

of a regular hexagon

ABCDEF,

express

CD, DE, EF,

FA, AC, AD)

AE

and

CE

in terms of

a and

b.

(Utkal B. Sc. Hons. 53)

a)
(2)

~*

~*
is

because
twice

AD

parallel to

BC

and

its

length.

Fig.No.8
or

(a+b)+CD=2b.

[by

(1)

&

(2)]

.(3)

.*.

F-4

is

equal to CD.bat in opposite direction,

(4)

CD f DE^b - a - a

=b - 2a,

[by

(3)

and

(4)]

2b-a.
Ex. 3.
The position
vectors

[by

(2)

and

(4)]

of Jour points A, B, C,

are

Introduction

11

a, b,

2a+3b, a- ab -

respectively.

Express

the vectors

AC,

--DB BC
9

0m/

C4
Let
->

i/z

tarww

0/a

0/zd b.

be the origin so that

OA =a, OJSb, OC=2a+3b

and OZ)=a-2b.

2 (a+b),

Ex.

4.

Five forces AB, AC,

->>>-> AF
AD, AE,
If

-(a+3b).
act at the vertex

->
of a regular hexagon

ABCDEF.
2.

Prove that the resultant

is

6AO

where

is the

centroid of the hexagon.

Refer figure Ex.

----R

be the resultant, then

R=AB+AC+AD+AE+AF
-

-->->--->-->

=*AB+(AD+DC)+AD+(AD+DE)+AF.

Now AF
directions

and DC are two equal vectors in opposite and hence they cancel each other. Similarly

AB

and D-E cancel each other.

/.
i.

R*=3AD**6.AO where

is

the

mid. point of

AD

e.

the centroid of the hexagon.

Vector Analysis
4.
If

Multiplication of vectors by scalars.


is

OA=SL and m

any positive

real

number then the

vector m.

OA

ma. is

defined to be a

vector in the direction


is

of the given vector

but whose module

times the module

of the given vector.

In a similar
vector
in

manner the vector -m..OA=-ma is a the direction opposite to that of a and having
that of a.
of a

module m times

The

division

vector

by a

real

number m may be

considered as the product of that vector by r J

Multiplication of vectors by scalars is


tive,, associative

commuta-

and

distributive,

e.

w.a m (na)s=(w) a
(m+ri)

a.m,
(wa),

a~;

m(a+1
where m and
n

are

any scalar numbers and a and b are

any two vectors.

The
we

first

three results follow

from the definition and

are going to prove here the last result.

Let
so that

CM=a, AB~b
If

OB=a+b.
+ive
A'

m
r

be

then choose

number, and B

on

OA and OB produced
so
that

respectively,

OA'=*m.OA and OB'^m.OB.

Introduction

13

.'.

04'o.ro.a,
is

Ofl'=i.(a+b)
AB,
.'.

(1)

Since A'B'

parallel to

4'5'=7n

^4^.

AB

or

A'B'^m b
.

.(2)

Now
or
or
iw $
.

OA'+A'B'^OB'

OA+m AB~m OB m a+w b-m (a+b).


.
.

[from

(1)

and

(2)]

Note In case be negative then


a

we should choose
point

A_

A'

on
(and

^40

produced

not

^ <&+h

produced) such D that 0/1' ism times Fig Ho. 11 OA but in direction opposite to that of 0-4. Tto above result can be similarly proved by the help of the diagram
given.

OA

Any of two
->

like vectors

can be expressed as a

multiple of the other.

Let 4Z?=sa==fl . a and


be two
like vectors so

A
.

A CZ)=b=& b A A that a=b. .(1) A A


.
.

<*<

_A
-

^
=s

^
(1).

a from

^ a

Hence proved.

14

Vector Analysis

Similarly
5.
If

we can show

that

a==~ b
.

i.

e.

AB**~ CD.

Linear Combination of Vectors.

a vector r can be expressed as


x, y, z.

where

any scalar numbers, then r a linear combination of the vectors a, b, c. .


.
.

.are

is

said to be

6.

Components of Vectors.
have already defined that

We
collinear

when any number

of

vectors are parallel to

the same line, they are said to be and when they are parallel to the same plane

they are said to be coplanar. Collinear Vector : (i)


given vector

can be expressed as

Any vector r xa where x is a

collinear

with a

scalar.

Let

04 =a and OP=r.
"*
"*
is

O
OA,
.

Since

OP

collinear with

it

can be expressed as a

-*
scalar multiple of OA,
i.

e.

OP=x OA
r=*.a.
Any
vector

or

Coplanar vectors
given (non-collinear)
as a
linear

r coplanar with any two


can be

vectors

and

uniquely

expressed

combination of the given vectors

i.

e.rxa+yb where
(^ aj B.
Sc. 1960)

andy

are scalar s.

Let

04= a and OBb be two


and

non-collinear vectors

OP^v
b

be a vector coplanar with a and

P draw PM and PN OB and OA respecparallel tively and meeting OA in Af and OB in JV.

Now

through
to

Fig.No.

13-

Introduction

15

OM being collinear
-

with

0^=#a,

ON

-> being collinear with OB=*yb.

Hence proved.

The above combination

is

unique*

In order to prove that the linear combination

r-xa+jb
is

...... (1)
in

unique,

let

us suppose

that

can be expressed

another

form as

r-*'a+/b
.

....

(2)
(2)]

[from

(1)

and

or

If

/>

be not equal to zero, then a

is

a scalar multiple

of b.

The above form shows that a and b are collinear which contradicts that a and b are non-collinear. Hence
we must have

pQ or x

#'s=0 or #=*'.

Similarly by writing
or

b=

a we can prove

that <?=()

y-j'asQ or ^=j'.
is

Hence we prove that the


vectors

above

combination

unique.
equal
are

Therefore if two

expressed in

terms of the

same two
coefficients

non-collinear

vectors

a and

b,

the corresponding scalar

are equal.

The above result will not be vectors a and b be collinear [see note
Note
i

true

in

case the two

after case (Hi).]


that

From

above

we

also

observe

if there

exists

16
relation

Vector Analysis
of the form

pa+qbO
(p

between two

non-collinear

vectors

a and

b, then

p=0, q~0

and q being scalars).

If there -be

several vectors

rt r2 r 3 ... coplanar with


,
,

two

non-collinear

vectors

a and

b,

then

they

may

be

expressed as

If

r be

their

sum, then

Above
vectors are

relation

shows that the components

of a

sum

of

case

r=0

the sums of components of these vectors. In then each of its components must be zero (as a

and b are non-collinear). Therefore the vector equation rs=0 te equivalent to the two scalar equation?.

Ex.

5.

Prove that the following lectors are coplanar

3a-7b-4c, 3a-2b+c, a+b+2c,


a, b,

c being any
If

vectors.

these vectors are coplanar, we should be able to express one of them as a linear combination of the other two. Let us suppose that the given vectors are coplanar.
/.

and y are scalars. we get 3x+jy=3,

.8a-7b-4c*(3a-2b+c)+j>(a+b + 2c), where Comparing the coefficients of a, b and c,


-Zx+y=*-l, *+-2y=-4.

A:

3. first two, we find that #=2, and ;> and.y satisfy the third equation as well. vectors are coplanar.

Solving the These values of x

Hence the given

(iii)

Non-coplanar vectors.
expressed

Any

vector

can

be

uniquely

as

a linear combination

of three

given (non-

Introduction
coplanar) vectors a, b,
scalar*.

17

i.e.

raa+^b-f
be

C where

x,

y and z

are

Let

CM=a, OB~b, OC=c


-

---*
OP=r.

(Pb. 60)

any

three

non-coplanar

vectors and let

Fig.NoJ4

The
three

three lines 0^4,

planes

0, OC taken in C04 and AOB. Now BOG,


to

pairs

determine

through
JV"

planes parallel respectively

the
in
,

planes

Pdraw BOC, COA and


respectively
is

AOB

cutting OA,

OB and OC
->

Af and

thus giving us a parallelepiped whose diagonal

OP.

Also

->
i.e.

OL

is

collinear
>

with 0-4
>

a;

.'.

OL==^a.

Similarly

-->->
Now r^OP

OM=jb and

The above combination

is

unique*

In order to prove that the linear combination

r**xa+.>b+c,
where
a, b,

...... (1)
let

c are non-coplanar vectors, is unique, that r can be expressed in another form as suppose

us

r*'a+/b+*'c

...... (2)

18
.'.

Vector Analysis

/.

or
If

+j*+*c~*'4/b+<;'c. [from (1) and (*-*') a+(j>-/)b+te-*')c0 La+Ajfb+JVc0.


then

(2)]

L be

not equal to zero,

a=

j-

b- ^

c.

bination of

Above shows that a can be expressed as a two non-collinear vectors b and c.

linear

comit is

But

essential here that a,

and we are given that


there
is

b and c should be coplanar [ a, b and c are non-coplanar.


So we must have
x
#'s=0,
i.e.
i.e.

5 P. 14]

Hence

a contradiction.

L=0
Similarly M=Q

or or or

#==#'.

^-y==0,

y=y'
z~z'.
is

and

JV=0
Hence we

^-^=0,
vectors
the

i.e.

prove that the above combination


are

unique.
of the

Therefore if two equal

expressed in terms

same

'three

non-coplanar vectors,

corresponding

scalar coefficients

are equal.

It should be understood here that the

above

result will not


in that

necessarily be true when a, b and c are coplanar as case c can be expressed in terms of a and b,
i.e.,

c=/>a+tfb.
b,

/.

rL - ra

*!

r!

and r2 are equal.


/.

since

a and b are two

vectors supposed non-collinear,

we must have
and

their coefficients each zero.

The above

relations do not necessarily

mean

that

*i*2,

J>i-J>2

and *i-*..

Introduction

19
if

Note

From above we

also observe that

a, b,

c are

three non-copalanar vectors and there exists a relation of the form

a+Afb+JVc=0
then

(L,

M,

JV being scalar)

L=0, M=0, JV=0.


Note
:

In

the relation

r=xa+jb+c,
P

the vectors *a,


#,

jb,

c are called the

components

of the vector r and

y, z

are called the coordinates of the point the vectors a, b, c.


If

with reference to

there

be several vectors rl9 r 2 r3


,

terms of three non-coplanar vectors be expressed as

a, b, c,

.. expressed in then they may

If

r be their sum, then

r=(Z
Above
of

xx )

a+(r^) b+(r

^)

c.

shows that the components of a the sums of the components of those vectors. In case r=0, then each of its components should be zero (as a, b, c are three non-coplanar vectors). Hence
relation

sum

vectors

are

the

vector

equation

r=0

is

equivalent

to

three scalar

equations.

2
Ex.
6.

#!=(),

Z j)'!0, Z ^=0.
non-zero,

//a,

b5

be any three

non-coplanar vectors,
:

Jind the linear relation between the following system of vectors

7a+6c,

a+b+c, 2a-b-fc, a-b~c.


coefficients of a,

Let 7a+(5c=* (a+b+c)+;> (2a-b+c)+* (a-b-c).

Comparing the

b and c we,
we
find

find that

Solving these three equations

that #

2t

-!.

d)onaiedby

Mr. N. Sreekanth

20

Vector Analysis

Hence the required


6.
If

linear relation

is

7a+6c2(a+b+c)+3(2a-b+c)-(a-b-c).
Linear dependence of vectors.
there exists a relation of the type

*a+jb+sc+...0
where
If

(1)

x, y,

z,

are scalars (not all zero), then the system


.
.

of vectors a, b, c.

is

said to be linearly

dependent*

it is

the system of vectors is not linearly dependent, then said to be linearly independent and in that case

*-0,j-O *-0...
f

If
i.e.

r=*a+j>'b-f-c+...
r-f*a4-rb-|-c.. .=0, then the system of vectors
. . .

r, a, b, c.

is

linearly dependent.

Now we know
exist

that between two collinear vectors there


the

a relation of

form r=*a.
a

Similarly

between
form

three coplanar vectors there exists

relation of the

raa+jb.

Also between any four vectors there exists a

relation of the type

rxa+jb+^c.
that

Therefore

we can say

or three coplanar vectors or any

four or

any two collinear vectors, more vectors form

a linearly dependent system.

Combining the
say that

results

proved

in

and
two

6,

we can
be

The

necessary

and

sufficient

condition

that

vectors

linearly independent is that they be collinear.

and
The necessary and
sufficient

condition

that

three

vectors be

linearly independent is that they be coplanar.

The Unit Vectors i, j, k* In 5 case (iii), we expressed any vector r three non-coplanar vectors a, b and c as r**x
7.

in

terms of

Introduction

21

Here
and

in this case the directions of the three non-coplanar

vectors will be

mutually perpendicular say OX,

OT and 0%

unit vectors in these

directions

are

denoted

by i, j, k respectively. Such a system of vectors


is

called

ortho-normal

system.

Now we

shall
Fig.No.15

express any vector r in terms of the three unit


vectors
i,

and k.
5,

Proceeding exactly as in
*z, then

case

(iii), if

OL~x,

x,y and z are called the coordinates of the point


Also
of

P.

OP*OL +LP*
2
i.
.

as

OL

is

the orthogonal projection

OP on OX,

^OLF^77/2

or

OP*=OL*+PR*+RL*=OL*+ON*+OM*
r
2

or
i. e.

=sx 2 +j> a

+
k

2
,

where r

is

the module of OP.


r is

square

of

the

module of vector
tuhen

sum of

the squares

of the

coefficients

ofi, j and

is

expressed in terms ofi, j, k.

Direction cosines of OP.

Again
then cos
/.
j8

if

OP makes an OL x
cos
a.

angle a

with the direction of

i,

oc^^^y
*=r
Similarly y

=r

cos ft

=r
f

cos

y,

where

and y are the angles which OP makes with the directions of unit vectors j and k respectively, cos oc cos ft cos y are

22

Vector Analysis

called the directions cosines of


1,

OP and are
cos 2
2

written as

m, n respectively.
*2

+y+
2

=r 2
\

(cos
:.

a+cos 2
2

j8-f

y).
2

But

+J

-f

2=1*

cos a-fcos

0+cos

= l.

Direction ratios of OP.


Also cos

a=

#
-,

cos /?=

cos

y=

Z
,

shoeing that the

direction cosines of

OP
j

the

coefficients

of

i,

are propoitional to #, jy, and z, i. e. and and the actual direction

cosines are obtained

module

of

by dividing their OP; i.e. V(*2 -h? 2 + 2 ).

coefficients
*,.)>,

by

r,

the

are

called the

rectangular coordinates of P.

Note : In the case of unit vector, the module is unity and hence if a unit vector be resolved in terms of i, j, k, then their coefficients themselves are the direction cosines.
Distance between two points P l and the direction cosines of the line joining them.
8.

P 2 and

Choose any point

as origin

then the position vectors

of

P and P 2
l

terms of unit

OP l and OP2 vectors i, j and k as


are

We

can express them

in

where

(xll

yl

Zi)

and

(xa ,ya ,

zj are the rectangular Cartesian

coordinates of PI and

Pa

i)

J+U,- *j)

k.

Introduction
If r

23
it

be the
is

distance

P-f^ then

is

the module of
of the squares

PiP2 and

equal to square root of the


i,

sum

of the coefficients of

and

k and

is

therefore equal to

Also the direction ratios of the line PjP2 are the the coefficients

#a~ *i ^a-.?! and


direction

22

1\ of

i,

and

recpectively.

The

actual

cosines

are

obtained

by dividing

these coefficients
or

by the module

of
2
J

P a P2

i.e. r

V[(*
-2
r
?

-*i)
<1

and are therefore

<2
r

l
m

Note

In case

OP l

and

OP2

are unit vectors, then

Putting

the value of

r,

we

find

the corresponding

direction cosines.

Ex.
and

7.

Three vectors of magnitudes


are

a,

2 i, 3a meet
f

in

a point

their directions

along the

diagonals of three

adjacent faces

of a cube.

Determine their relultant


Sc. 51,

and

its direction cosines.

(Lucknow B.

Utkal B.

Sc.-

Hon's

53, B.

H. U.

M.
Prove also that the

Sc. 54)
the

sum of

the three

vectors determined by

diagonals of three adjacent faces of a cube passing


corner, the vectors being directed

through the same

from that

corner is twice the vector

determined by the diagonal of the cube.

24

Vector Analysis

Let the edge of a cube be taken as unity and the


vectors represented

by OA,
length

OB, ous be

OC
i,

the three coterminof

edges

unit

OR,

j and k respectively. OS and OT are the

three diagonals of the

three

adjacent faces of the

cube

8
Fig.No.16
2a and

r\

along which act the vectors

OL,

OM and

OJf of magnitudes

a,

30 respectively.
corres-

In order to find these vectors


in these directions

we

shall first find unit vectors

and then multiply them by their

ponding given magnitudes.

and
/.

its

module

is

clearly

V (0+1+1)=

unit vector along

OR*= ~

V^ (j+k)
-

Now

the magnitude of vector

OL

is a.

Exactly
tude 20
is

in a similar

manner

the vector

OM

of

magniis

vj~

(k+i) and vector OJVof magnitude 80

(i+J)

Introduction

25

The magnitude

of the resultant

is

,.

direction cosmes.are

40

50

30

or

V2'

e5V2'

5V2*
-

Again OB+05+Or=2 (i+j+k)=s2.0P which proves second past [on putting *=j=2=l in (1) of 7 P. 21],

Exercise No*
i.

a,

fiat of a point

a point

D in
If

position vectors of A and B respectively, find C in AC produced such that AC3AB, and that of BA produced such that BD2BA.
are the
'

be the origin, then


get
--

0^=a, OB=b. Now from

>

AC^SAB, we

>

/.

>

OC- 0/1=3(05-0.4),
Similarly

OC3b-2a.

OD=2a-b.
vertices

of a triangle are

the points

r^

/A^

vectors

determined by the sides ?

Find

the length

of

these vectors.

See

8.

(bi-aj i+(* 2 -<z 2 )


{(fc 1

+ (i 3 -s) k etc.
A
and

Sides are
3.

-fl 1 )H(*2-^)a +(fr3 -'fl3) 2 } 1/2 etc.


rectors

If

the postiion

of

arc

i+3j-7k
cosines

respectively.find

AB

and determine

its

direction

and

its

module.

^Donated by

26

Vector Analysis

Ans.
,
,

4i-5j
4

module9V2.
-5
11

4.

The position

vectors of the points P,

Q, R, S are

(1+ j +k), (21+5J), (3i +2J - 5k), (i - 6j -k). Prove that the lines PQ and RS are parallel, and find the
these lengths.

ratio

of

In this question
lines

show

that the direstion

ratios of the

PQ

and RS are proportional.


In
the ad-

Ratio uf their lengths

is*.
5. (a)

joining figure if

OC-c,
find the vectors given

by

a+b+c, cta-b,
and
independent.
in terms

b-fc-a,

a+b-c,
along the four diagonals are linearly
along the

verify that the four vectors

Also find the vectors

diagonals of the Jaces

0/a, b, c.

0/l-a+b-c..(3)

......... (4)
From
(1), (2), (3)

and

(4),

we

find that

Introduction

27

Hence they

are linearly independent.

Similarly
faces.

we

can find the other diagonals


at the

of

the other

(b)
1,

particle

corner of a

cube

is

acted on by forces

2,

Ibs.-wt. respectively

along

the diagonals

of the faces of the

cube which meet the particle.

Find

their resultant.

Putting

0=1

in

solved

Ex.

P.

23,

we

get

fl=5.

force represented

by

V ^ (5i+4j+3k).
Forces

6.

ABCD

is

a quadrilateral.

BA, BC, CD,

----DA
i. e.

act

at a point.

Show

R=BA+BC+CD + DA.

--

2BA.
-

that their resultant is

Add

AB+BA

7.

ABODE is

a pentagon. Forces

AB, AE, BC, DC,


is

ED and

AC
a

act at a point.
8,

Prove that their icsultant

SAC.
of the sides of

Find the position vectors of


in

the mid-points

triangle

terms of the position

vectors

of

the vertices

and hence
triangle

prove that
directed

sum of

the vectors determined by the medians of a

from

the vertices is zero.

(Agra 37)

being the mid point

oiBC.
/.

->-OD^OB+BD

28

Vector Analysis

c-b b+c - -

etc.

etc.

9.

Prove that the following vectors are coplanar

5a+6b+ 7c,
10.

7a-Sb-f9c, 3a+20b+5c.
between the following systems oj
vectors
:

Find

the

linear relation

vectors, a,

b, c being any three non-coplanar

a+3b+4c, a-2b+3c, a+5b-2c, 6a+14b+4c.


Ans.

(ia+14b+4c(a-f 3b+4c)+2 (a-2b+3c) +3 (a+5b-2c)


If the
resultant of two forces is equal in
to
it

n.
of
the

magnitude

to one

components and perpendicular

in

direction, find the

othet component.

Let the force


zontal and Q, be
it

be

hori-

inclined

to

at

an angle
is

0,

so

that the

resultant

a vertical force
vectors along
vertical

P. If the unit

horizontal

and

be

denoted by i and j, then force P is Pi and Q, is Q, cos 0i along the direction


()

of

and

sin 0j along

j.

The

resultant

is

Pj.

Equating the

coefficients of

and

j,

we
or

get

Q, sin

0=P,
/.

and
tan

P+Q,

cos

0=0.

or

Q, cos

0= - P

0= -

0135

Introduction
12.
angle

29
the force inclined at vertical force

Find

the

horizontal force

and

an

of 60

to the vertical

whose

resultant is a

ofP

Ibs.-wt.

Ans.
13.

V3P,

2P.

If two concurrent forces be represented by


prove
that
their resultant
is

n.OP and

w.OQ,
>
respectively,

given by (m+n).

OR

where

divides

PQ, such

that

n.PR=m.RQ.

/.

n.OP=n.OR+n.RP,

.*.

m.OQ^m.OR+m.RQ.
-

A
is

resultant of

72.

OP

and

Now we

are given that

n.PR=sm.RQ.
-^
or

->

->

-n.RP*

or

.%

resultant

is

(m-h)

0^2.

The
~

point

/?

divides PQ, in

the ratio

l.OQ,, then their

Cor.

In case the forces be

1,OP and

80

Vector Analysis

resultant will be (1+1)

i.e.

OR

QOR where R

divides PQ, in

the ratio 1
14.

i. e.

is

the middle point of

P&.

Prove that the system of concurrent forces acting at a point

--*
and
represented by

OA, OB,

OC

is

equivalent to the system

of forces

____
represented by

OD, OE,

OF acting at
sides

the

are the middle points


the triangle

of the

EC, CA

same point where D, E, F and AB respectively of

ABC.
that

We have to prove
-

_>____._
OA+OB+OC=*OD+OE+OF.
where

Now OB+OC=2OD
(by Q.
13).
1*5.

is

middle point

ofBC

Write similar other relations and add.

(a)

Two*forces
AB,

act

at the

corner

of a quadrilateral

->

->

ABCD
-

represented by

AD

and two at

represented by

CB

and

->
Prove that their resultant
the
is represented

CD.

by

4EF

where

E and
Q,. 13]

F are

middle points of

AC

and

BD

respectively.

(Agra M.

Sc. 58)

[Use cor.

Proceed as in Q. 14. ABCD is a quadrilateral and (b)


of the
lines joining the

P the point
->

of

intersection

--*-A, OB, OC,

middle points of opposite sides. Show that the

resultant of
point.

OD is
at

equal

to

40P

where

is

any

16.
transversal

Forces
cuts

Q,

act

and have a resultant


action at A,

R.,

If any

their

lines oj

and

C
.

respectively,

Prove that

A Q- + OB =Q C

'

(Agra

^ ^ ^. Luck B

Sc. 49)

Introduction

Let the forces represented by


that
the

P and
-

Q be
so

OL and
-

OM

diagonal

ON

repre-

sents the force R, so that

R ...... (1)

Let

CU=a

and

(2)

P is

in the

same
>
.

direction as

a and hence

it

can be

expressed as

0-4.

- p
->
.

Similarly

Q**n OB

and

let

R*=t
.

OC.

P
P

R
R

We

have to prove that Q'd+ffB^n^

From
or

(1),

we

get

m .OA+n. OB=t .OC

--get

or

...... (3)

Comparing a and

b,

we
or

=1-A; and "T"^Hence proved.

A
Note.

3=1

m-j-n=f.

Alternative method for this question will be


(Ex. 16 P. 70)

gevin in next chapter.

82

Vector Analysis

Ex. 17.

Prove that the

Pl

and

P2
/!,

is

^i +^ 2
fy

>

magnitude of the resultant offorces is the angle +2P1 /J 2 cos 0) 1 ' 2, where
n 2 be the direction cosines of the

between the direction of the forces.


If

%,

and

/2 ,

^2

lines of action
i,

of the

forces with reference to unit vectors

and k, then these forces can be expressed as Pil l i+P l ml \+Pl nJk and P
If

be their resultant, then


of

is

given by

The magnitude

is

Note.

If there be
is

number
{2P1
2

of

forces

Plf P 2 P3 ,.
,

then

the magnitude

given by
is the

+2ZPf P, cos (Pr P


and 0'

s )}

1/2 .

Ex.
a triangle

18.

IfO

circum-centre

the ortho-centre of

ABC,
(i)

then prove that

-->-->

OA+OB+OC=00',
0'A+0'B+0'C~20'0,
AO'+0'B+0'C=*AP,
of the circum-cirde
.

(ii)

(ioi)

where

APis

the diameter

Before doing the above problem we should note that


in the adjoining figure

20DAO'
(i)

(by geometry).

is

OB+OC=*20D, where the middle point of BC

D r
Fig.No.22

(O. 13 Cor,).

Introduction

33

/.

OA+OB+OC=*OA+<20D=OA+AO'=00'.
->

---

(ii)

O'^+O'C^O'/)^ (0'0-fOZ))20'0+20Z)

(iii)

(Note)

from

(ii)

(the vector represented

by the radius

through

of the circum-circle)

= AP where AP is diameter.
19.

ABC

the resultant

of AP,

->> PB
}

is

triangle

and

P any point
that

in

BC

If

PQ

is

PC, show

ABQC

is

a parallelogram

and Q, therefore a fixed point.

(Luck. B. Sc. 54)

AP+PB+PC
Now
C draw
is

through

CD
AB.

parallel

and equal to

Therefore

ACDB
so
that

parallelogram,

But the resultant


with Q.

is

given to be PQ,. Hence

D
in

coincides

Since with the change of position of

BC,

AB

34
is

Vector Analysis
not affected or

CD

i. e.

CQ,

is

not affected, there ore

Q,

is

fixed.

20*

A
to
it

man

travelling

east at

8 miles per hour finds that

the

wind seems
he finds that
the wind.

blow directly from the north.


appears to come from
JV.

On

doubling his
the

speed
oj

E.

Find

velocity

Let us
unit vectors
velocities
of
-E.

suppose
i,

that

/V

represent

^
w~

.^

m.
JV.

p.

h.
res-

towards

and

pectively so that the


city of

velo-

the
i

man
as he
p. h.

is
is

repretravel-

sented by
lling

at

8 m.

towards
in the

5 Fig. No 24
i

east.

the velocity of

Again suppose that the wind


x
find.

and j

plane

is

given by

xi+yj where
required to

and j are scalars whose values

we
is

are

Now
by

the velocity of

wind

relative

to the

man

given

actual velocity of

wind + velocity

of

man

reversed.

We are given that the wind appears to blow front north


and hence
/.
it is

given by

/J.

(*-l)i+.rf--/d; /. Again when the man doubles


2i then the relatjyej^locity -is

*-l0
his speed
it

or
i. e.

*L
it

becomes
i+jj.

(*i+jj) 2i
-/2

or

(x - 2)

But

in

this
is

pas'e'-* tyfe

are given that

appears to

blow

from N. E. and

therefore given by

(i-f j).

and
Putting *==!

j>

-/,.
/

we

find that

sl,

and hence

j-

Introduction
/.

35

velocity of
is <\/2

wind
e.

is

module
8

units

i.

8V'2 m.

given by i-j -(j-i). Its h. as one unit represents p.


is

in. p. h.

and clearly the direction


Show that if two

from N. \V.
of Hence or otherwise show
(11, 3, 7)

21.

vectors are parallel, the components

the one are proportional to those

of the other.

that

the

three points

(1,

-2, -8),

are collinear and find the ratio in

B (5, 0, -2), C which B divides AC.


two
like

Ref. P. 13.

Any

of

the

vectors

can be
:

expressed as a multiple of the other.


22.

Ans. 2
are
3).

The

vertices

of a

quadrilateral

A(l,2,l),
At
the point A,

B(-4,2,-2),C
forces
lively.

(4,1,
2, 3,

- 5)
2

and

(2,

- 1,

of magnitudes

Ibs-wt. act along

AB, AC,

AD

resrec-

Find

their resultant.

Refer Ex. 7 P.
and then
find

23.

Express

in

terms of unit vectors

unit vectors along

AB,

AC
.

and

AD
.

then the

force of 2 Ibs. alog


1
.

AB** *2. unit


.

vector along
.

AB
.

etc.

9
G

'' ''

vas

magmtude

1S

V UT>

//11B\

direction cosines of result are

_ _
i
'

-y_
)'

-^
vUi8)'

CHAPTER

II

CENTROID, LINE AND PLANE


i the

To find

the

position

vector

of

the point

with divides

join

of two given points

A
:

and

whose position vectors are

a and

b in a given ratio say

n.

\Ve are given that

Now

m+n

AB.

*
TZ

m-f

-^
or
,

(b-a);

-.

m-\-n

->

->

Alternative.

From

(1),

we

get

n.AP=*m.PB

or

or

OP~n.OA+m.OB

Centroid, Line and Plane

37
get

Middle point*

Putting

m=n=l, we

the position

vector of the middle point of

AB

as

JL

......... (3)

Note.

The

point

is

called the

centroid of the points

A and B

with associated

and divides

AB

internally

numbers n and in the ratio m n,


:

respectively

Cartesian equivalence. Let us suppose that in terms


the point

of unit vectors

i,

and

given by tfii+JiJ+^ik so that the point and that of B is

is

is (x lt

y l9 ^)
*}
get

*2*+.)'j+**k so that the point


If

is (* 2 J>a
(2),

P be

the point *i+j>j-|-dc, then from

we

Equating the

coefficients of

i,

and k we get
;

m+n
2 Centroid

V_ 7

m+n

^ Z __

*"

~~"

m+n
M.
Sc. 53, 58)
relative

and centroid with associated numbers.


(Agra
vectors
to

If

there be n points whose position

be given by any origin whose position vector is


C/Crsss

a, b, c, d, ...

then the point

(a+b+c+d+...)
n

---mean
position
of

is

called the

centroid or centre of

the

given points.

Again if there be n scalars whose position vector is

q, *, s,

then the point


.

38
is

Vector Analysis
called

the centroid of the given points with associated


q, r,

numbers p,

s...

Note
cover up

The words associated numbers


all

are

used to
stand for

forms of centroid,
,

e.

g.

they

may

the masses

m^ m 2 m 3

. .

of a

given set of points; then of mass, i.e. c.m. Similarly centre


parallel
forces.

particles placed at system the centroid is called the centre


of

of gravity or centre of

Thus

if

we

place

particles

of

masses

mi> f*hi

tt*3-

at points A,
.

are a, b, c, d.

.,

J3, C, position vectors then the position vector of their centre of


. .

whose

mass G

will

be

Equivalence of Cartesian Forms.


Let us express the vectors unit vectors i, j and k, so that
i.e.
i.e.

a, b, c,

etc.

in

terms of

the point the point

A B

is (x l9

y lt

Zi\
2)

is

(x^y^

............................ and so on,


us suppose that the the coordinates of the centre
let

and

point
of

G=*i4:jj+k,
are (x,y,

so that
z)i

mass G

then

from

(1),

we

get
-

2m

Equating the coefficients of i, j and k, coordinates of the centre of mass as


.

we

get the

27

nij

27

nil
1, 2, 3,

27

m^
7,

Ex.

z.

Particles

of masses
at the corners

4 9 5y 6,

grams
four
their

respectively

are placed

of a

unit cube,
the
last

the first
at

at the corners

A, B, C,

of one face and

four

Centroid, Line and Plane


projections A', B', G',
the coordinates

39
Find,

D'

respectively

on the opposite face.

of
as

their centre

of mass.

A
and
let

origin

AA',

AB

and

AD
tors

represent unit veci,

and k respec-

tively.

The
tors

position

vec-

of
(the

the 8

corners

are

masses of the

are also particles written within brackets(1)

B
(5)

B'

Fig.No.26

,4=0
J5=j

IT=i

(2) (3) (4)


.*.

(0)
(7)
(8)

4=i+j
C'

C=j+k
Z)=k

D'

position vector of the centre of


.

mass say

is

Tt

.0+2 j+3(j-)-k)+Jk+5i+C(i+j)+7(i+j+k)+8(k+i;

1+2+3+4+5+6+7+8

/.

magnitude

of

^G
(Jfi

Also cooadinates of c.m. are


3.

Ai
wiVA

is)-

//

60

/A* centroid

of a system of points A, B, C,.


associated

. .

position

vectors

are

a, b, c...

numbers
.

q, r,.

<w<f

G'

/Afl^

of another system of points A' 9 B', C'.


are a',

whose position vectors


P'
q'i r '*

b',

c'

wtVA

associated

numbers

^^

centroid of all the points taken together is the

40
<

Vector Analysis
of
the
.)

centroid

points

and
. .

G'

with

associated

numbers

(P+4+r.

and (p'+q'+r'+

.) respectively.

By

definition,

we have

Now
associated

if

fce

the

centroid

of the points
.)

and G' with


.. .,)

numbers

(/>+?-fr-f-..

and (p'+q'-\-r'+

respectively, then

0//=-

or

/a+?b-H-c-f/a'4-?'b'+r'c'4- .

-----prove
that
centroid
is

H
vectors.

is

clearly the centroid of all the points taken together.

4.

To

independent of the origin of

(Agra

M.

Sc* 53, 58)

Let G be the centroid


1

of

Q'

points
tors

whose

position
to

vec-

relative
. .

an origin

be a, b, c.

with associated
g,

numbers
tively.

p,

r.

respec-

Fiq.No.27

a)

Now we have to prove

that

if

instead of

we choose any

point 0* as origin, then the position of the centroid

will

Centroid, Line and Plane

41
i;

remain unchanged.
the position vector of

Let the position vector of 0' be A relative to 0' will be

then

Similarly position vectors of


origin aie
.

C,

etc.

relative to

0' as
of

b-i, c-i. .etc. If C' be C. G. relative to new origin 0', then

the

new

position

or or

0'G'

= OG-00'

0'G' = 0'G.

[from

(1)]

with G.

Above relation shows that the point G' should coincide Hence the position of the centroid remains unof origin.

changed with the change


5.

To prove

that the vector relation

/a-f qb+rc+...==0
will be independent of the origin if

and only if />+#4-?"+

=0,

where p,q,i.. -are scalars.

With

reference to the origin 0,

we have

the relation
(I)

/>a+0b+rc+...=*0

........

Now
to

be

i;

choose 0' as origin whose position vector relative then as in last article the relation (L) w. r. t. new
is

origin 0'

p (a-i)+ j (b-i)+r (c-i)+


or
If

......... (2)
. .

(^a+^b+rc+
the relation
(1) is

. .

.)-i(p+q+r+

.)

...... (3)

then

(1)

and

(3)
if

independent of the origin should represent the same relation which is

to be

possible only

..=Q

.........

(J)

42
Also
if

Vector Analysis

p+q+r+.

.=0, then

(1)

and

(3)

represent the

same.

Hence proved.

Note.
is
.

We

have

already

proved
shall
if

that

centroid

is

independent of the origin


property
a, b, c.
.

and we

see that

the above
of

satisfied.

Thus

be
.

the
.,

centroid

with associated numbers

p, q, f .

then

or

In
vectors

- R (p+q+r+ .)=0. the above relation the sum of the coefficients of is (p+q+r+ .)-(p+q+r+ .) which is zero.

p*+qb+rc+
. .

. . .

the

Exercise No.
I.

is

the

centroid

of tetrahedron

ABCD.
and
centroid

A'B'C'D'

is

another tetrahedron
bisected at G.

such that
that

AA', BB', CC'


is

DD'
of
the

are all
second

Prove

also

the

tetrahedron.

With

respect

to

any

origin

say

the

point

is

a+b+c+d where
B,
t

a b c d

are tbe pos ; tion

vectors of A,
of A', B',

C and

D respectively.
a', b', c',

C' D' be

Let the position vectors d' and we have to prove that

Now G

is

the mid. point of AA', BB',:,CC', DD'.

..

b+c+d-a D=
,

-g-

'

= d_? b^5

d'^

Centroid, Line and Plane

43

~~

4
2,
so on
-4

line

AB

is

bisected in

Ol9 O^B

in

2,

in

and
are

ad

infinitum.

Particles of masses m,

-^ m, ^~

m
c.

ttlaced at

O ly

2,

j.

Show

that the

distance of their

m.

from

is equal to one-third of the

distance from
2

to

B.

Am 2~b2
m
M

m
"93 ^-

B
4 be
a
3

Let us choose
that the points C^,

as origin, the point


3
.

taken as a so

2,

are
"
'

^L
Y"'
If

^'-

2
c.

a a-A vir^/'J 2

^ a

V^y^^
w

^^

be the

m. of the particles
\

at these points,

then

w /a (

m/a\
m
m

2+02 +g 3

"
of

G. P.
a
L

or

BG=

("Sum
T

...CO

7^
)

__^
r

Above shows
that of

that the distance of

from

is

one-third

A from

44
3.
3j,
.

Vector Analysis
Find
the

centroid

of 3n points

i,

2i,

3i...wi;

j, 2j,

.j; k, 2k, 3k,.

"k.

mats are placed at (n 2) of the corners Find their c. m. of a regular polygon of n sides. b>3 the position vectors of the Let a, b, c, d, e, f
4.
Particles of equal
.

vertices

of

the

polygon with respect

to

centre

of the

polygon. If equal masses were placed at all the corners, then the c. m. will coincide with and hence

Now
the

let

the vertices a,

b b2 vacant anl G be

the

c.

g. of

remaining (-'2) particle,

placed at the other vertices

so that
*T^ ----- 4-f -OG~ c -f d -f-e--- -(-... =
-

1
.

r u>i {-(afb;} from


/
,

/IN (i)

-25
where P
or
is

the mid. point of the join of vacant vertices,

=
5.

w-2r
is

or
acted

particle

on
repel,

by a number of centres offorces,


the force in each

some of which attract anl some


as
the

case

varying

distance.

The

intensities

for different centres are different.


all

Prove that the resultant passes through a fixed point for


tions

posi-

of

the particle.

(Agra M.

Sc. 40)

Let

centres of force

be the position of the particle and the various be a, b, c. . w. r. t. as origin. L 3 2


,

The
.

forces
. .

acting on the
p. 2 , /i 3
.

particle

are

given by

/t^a,

/u 2

b,

where p l9

are constants which

may

bs +ive

Centroids, Line and Plane


or
ive

45

according as the centres attract or repel.


is

The

resultant of these forces

given by
a, b, c.

l^a+^b+^c-f ............ (1)

Now
numbers

if

G
/;

be the centroid of
,

with associated

^3

..,

then

But

is

independent of the origin of vectors

I. e.

the particle. Hence G is fixed. Thus the resultant passes a fixed point for all positions of the particle. through
6.

IfG L

is

the

mean

centre

of

A l} Bls C l

and

Gx

that of

A 2,

RZJ

C* 2 ,

then show that

A4+A+C C
1

Hint.

If

be the origin, then

7.

//

be the in-ccntre of the triangle

ABC

and

a, b, c

be the

lengths

of

the sides, then prove that the forces

->-->

We

a .iA, b. IB, c.iC are in equilibrium. have to prove that

Now we know
centroid of
points

that
A,

the

incentre of a triangle

is

the
a,

C, with associated numbers

Also we know that centroid is indepenrespectively. dent of the origin of vectors. Therefore if we take i as
c

and

origin

of

vectors,

then the position

vectors of the vertices

are i4,

->
ifl

an
1

incentre

i itself

1C respectively and the position vectors of w. r. t. \ as origin will be 0.

46

Vector Analysis

--a+b+c
divide
the

0=

a.iA+b.iB+c.iC

or

a.iA+b iJ5+c,iC=0.
Hence the
8.
in
the

forces are in equilibrium.

The points
ratio

D,E,F
1:4,

sides
:

BC, CA,

AB

of a

triangle

--->
triangle
the

3:2

and 3

respectively.

Show

is parallel to

that the

sum of the

vectors
:

AD, BE, CF
3.

CK where K
1,

divides

AB

in the ratio 1
vertices

g.

The

of a

ABC

are

(2,

3),

(4, 2, 3),

(6, 3, 4).

Find

coordinates

of points

P and
>

which divide

BC

in the ratio

3:2
i,

Also show that vector

AQ has

direction.cosines proportional to 8, 6, 9.

In terms of unit vectors

is

21- j-3k
10.

etc.

7 P. 20.
vectors

j and k the Ans. P (V,

position vector of
,

),

Q, (10, 5,

6).

d respectively. ABCD BD intersect at the point P which divides AC

The position are a, b, c and

of the

vertices

of a quadrilateral The diagonals AC and

and

BD

in the ratios
the point

n and m'

ri

respectively.

Find
and

the ratios in

which

Q, which

is the intersection

of

AB

CD

divides these sides.

The
forms w.

P written position vector of point are respectively r. t. AC and

in

two

different

BD

wa-f me
n (m'+n')

n'h+m'd "~

/.

a+m (ro'+n') c=*n'


n'

or

n (;rc'+;z')Ja-tt' Now n (m'+n')-

(m+n) b+m' (m+n) d bro' (m+n) d-m (m'+n c. .(A) (m+n)


f

(m+n)=*nm'

n'm

and

m' (m+ri)-m (m'+n')=*nm'- rim. Hence dividing both sides of (A) by wn'-n'm,

we
f

get

n(m'+n') *-n' (m+n)b "~ m' (m+n)


nm' - rim

d-m (m'+n

nm' - rim

Centroid, Line and Plane

47

L. H.S. represents a point on AB dividing it in the r ratio -n (m+ri) n(m'+n') and R. H. S. represents a point on DC dividing it in the ratio -wz (m'+n f ) m' (m+n).
: :

Since the

position

and
i.e.

CD

are

same,

hence

vectors of both these points on AB it is their point of intersection


line.
line

&
6.

Vector equation of a straight


the
vector

To find

equation

of a

straight

that

passes

through a given point

a and

is parallel to

a given vector b.

Let P be any point on the straight line through

a which
and
its

is

parallel

to

position

vector

be r so that

r=OP.
to

Now
b
Fiq.No.2a

AP

is

parallel
it
t

and
>

hence

must be some multiple


is

of b.

Therefore

where

and negative

a constant positive for points on one for points on the other side.

AP=tb side of A

Now

-->OP=OA+AP.
.\

r=a+/b
the line
is

(1)

Every point
of
t

P on

obtained for some value

and for every value of / we get the position vector of a point on the line and hence above represents the vector
equation of the required line.

Cor.
origin.

x.

To

find the vector equation of a straight line through

Putting

a=0

in

(1),

we

get the required line as r/b..(2)

48
Cor.
given points
2.

Vector Analysis

To fnd the vector equation of a line A and B whose position vectors are a and b
(Agra

through two
respectively.

M.

Sc, 42, 45, 47, 52)

Now AB^OB-OA^b-a;
therefore the required line is and passes parallel to b-a A. through

Hence from
tion
is

(1) its

equa-

given by

Fig.No.29
or

r=(l-/
Cor.
3.

To fnd

of the

lines (1), (2), (3) found

the cartesian equivalents above.

of the vector equation

Let
the

in

[Refer author's book on Solid Geometry] terms of unit vectors i, j, k through the origin 0,

vectcr
2

r=*i+^j+k so

that

is

the point
(fl 1?

(x,
2,

}\ z)

and and

j+0 3 k so that the point A is b=i 1 i-)-fc2 j+4 3k so that the point B is (4 lf 4 2 Substituting in (1), i.e. r=a+/b, we get.
assflji-f

a3)

4 3 ).

Equating the

coefficients of

i,

and

k we
;

get

corresponding cartesian equation of a straight line through a given point (a l9 a 2 a z ) and whose
is

Above

the

direction cosines are proportional to b l9 b 2 b 3


,

Again substituting

in

2) i.e.

r=/b we
b%

get

bi

b2

which

the cartesian equation of a straight line through with direction cosines proportional to b lt b 2 b 3 origin
is
.
.

Centroid, Line and Plane

49
*0

Again substituting
ing as above,

in

(3), i.e.

r=a-H

fa

and proceed-

we

get

which

the cartesian equation of a straight line through the points whose coordinates are (a lt a z 3 ) and (b lt b& fr 3). Condition for three points to be collinear. 7*
is
,

(Agra M.
To prove
points
in

Sc. 32, 37, 51)


three

that the

necessary and

sufficient

condition for

three dimensional space to be

collinear is that there exists


vectors

a linear relation connecting


algebraic

their position

such

that

the

sum of tlie coefficients in it is zero. We have seen that the equation of a through a and b is given by
which may be written as

straight

line

(1-0 a-Hb-r=9.
Above
that
is

a relation between

the

position

vectors of

three points A,

B and P which are collinear and we observe algebraic sum of the coefficients of a, b and r is

1 which is zero. Hence the condition is necessary. In order to prove that the condition is sufficient let us suppose that any three vectors a, b and c be connected by

the relation
n (n^O),

/a+wb+nc
get

0,

where

/+m+=0

Dividing by

we

L a+

m
.

b+c0,

where -~ +
get

^ +1-0.
n

(Agra 45)

Putting
/.

--=-*, we n
n

-n

-a-

or

c-(l-<) a+/b.
is

ing

Above relation shows that c a and b and hence a, b, c are

a point on the^ine joint

collinear.

50

Vector Analysis

Alternative Proof*

Let us suppose that the points a b, c are collinear. Let c divide the line joining a and b in ratio m / so that
:

or

let

(l+m)=^n or /+m+n=0. order to prove that the condition is sufficient Again us suppose that /a+mb+-c=0, where

/a+*ttb+wc=0, where
in

or
or

/a-}-mb

-nc=(/+w)

c,

/a+mb =

l+m

c.

in

Afcove relation shows that c divides the join of a and Hence the three points a, b, c are the ratio m I.
:

collinear.
8.

Bisectors of the angle between

two

straight

lines.

Let the equations of the lines AL and AM be given by r a-Mb and r=a + *'c
so that their point of intersection
tion vector
is is

the point

whose

posi-

b and

a and they are parallel respectively to vectors

c.

Let
denoted
italic

the

modules

be

by corresponding
choose
three

litters a, b) c.

Now

AM
at

points B,

C and

on AL,

and
a

LA
unit
If

(produced)
distance
Q,

F^ No 3

from A.
then

P and

4Pand

AQ

middle points of BC and CD, are the required bisectors whose equations
are the

we

are to find.

Now

position vector of

is

OB~.
.

Centroid, Line and Plane

51

.*.

AB

is

a unit vector along

b and

as such

it is -T-

Similarly that of

is

a+

and that

of

b
is

a--,-.

Therefore

P and

Q, being the mid. points of

BC and CD
(

have their position vectors

a+J

-f

\ a+J

~Ty
as the

and

is

a.

Now

using the formula


line

r=a+* (b-a)
the points

equation of
position
lines

a straight

through

whose

vectors are

a and

b,

we

get the equation of the


-\

AP and

AQ, as

r=a-fk.iT
(

and
or

r=a+k'.i

T-

] respectively

were

Note. In case the point of intersection of the two lines origin, then the corresponding equations would be

A
If

A
A A

b and

c are the unit vectors, then

r; (b+c) and r**t' (c-b).


Alternative method.

A b=6b A A

A
and c=cc.

AL

Taking the point A as origin the equations are r==pb and

of the lines

AM

and r~p'c. Take any pt. P on the internal


bisector
parallel

and draw PJV


to the
direc-

tion of
fore

AM.

There-

52

Vector Analysis

hence
directions
of

ANNP* Now
b and

24JVand JVPare in the

vectors

same

scalar

multiples of

c and therefore they are the the corresponding unit vector in


-* A c then JVP=/c=/
c

that direction,

~*
i.e.

if

AN- tb =

b
t -r-.

--b

AP' is obtained by Similarly the external bisector of lines which are paralconsidering the internal bisectors
lel

to

b and

c and therefore

its

equation

is

A A
Ex.
and find
ist
i.

Prove

that the

medians of a triangle are concurrent,

the point

of concurrency.

(Lucknow

52,

Agra

52, 55)

Method.
a, b,

Let

c be the
the

posi-

tion vectors

of

vertices

of

the

triangle
of

so that the
the

co-ordinates
points

mid.

D,

and

of the

sides are

b+c c+a
2
respectively.
'

and

Now A
trisection) is

is

a and

D
in

is

b+c T-

and therefore the point


2
:

which divides

AD

the ratio

(i.e.

the point of

a+b+c
"2+1

Centroid, Line and Plane

53

lies

The symmetry of the result shows that the point G on the medians BE and CF as well and divides them in the .ratio 2 1. Therefore the three medians are concur:

rent at the point

which

is

also the centroid of the

given triangle.

2nd Method.

We

know
a
t

that

the vector equation of a line joining

the points or

ind

is

r=a+' (b-a) r=(l-/)a-f/b.


B
is

is

b,

is

b and E

is

Equations to medians

AD

and

BE

are

......... (1)
and
If the

rd -*) b+*
two
straight lines intersect,

......... (2)
we
should be able to

find

some

suitable values of s

and

which
shall

should give

identical

values

of r.

For

this

we

compare the
of r.

coefficients of

equal vectors in the

two values

~~"
2 '~2

Solving
t

we
or
(2)

get

*ss,y=f.

Substituting the values of

and

s in (1)

we

observe that the medians

AD

and

BE

intersect at the point

*
^ o

*c
.

Ex.

2*

Prove that the internal bisectors of the

angles

of a

triangle are concurrent.

(Agra

47, 5*, 57;

Lucknow

53;

Dacca

54
Let
a, b,

Vector Analysis

c be the posivertices

tion vectors of the

A B and C
9

respectively

and

a,

0, y

be the lengths

of the

sides

BC,

CA and
Also
-

AB
-

respectively.

C=c-band CM=a-c and ^=b-a. Now by 8, the equation of bisector AD

is

or
Similarly
bisectors

.-_+.b+c
\
y

P J

...... (1)
the

we can
as

write

down

the equations of

BE and CF

_._-c-y

a ......... (2)

and
If (1), (2) intersect

able values of p,

q, s

we should be able to find some suitwhich make the values of r identical.


compare the
coefficients of vectors a,

For
and c

this

we
P

will
(2).

in (1)

and

Centroid, Line

and Plane

55

putting the value of p in (1) or of q in (2), we get the position vector of the point of intersection of (1) and (2) as

On

oca+/3b+yc

The symmetry
point
of

of the

result

shows that

this is also the

intersection

of the

other bisectors and hence the

three internal bisectors meet at the point r


is

~r

-c which

the centroid of the points a, b, c with associated


i.e.

numbers

a, 0, y,

the lengths of the opposite sides.

(See alternative
(b)

method

in

Ex.

5).

The

internal

bisector
the

the external bisectors

of

of an angle of a other two are concurrent.

triangle

and

Let C be the origin and the


position vectors of

A and B be
Again

respectively
let

a and b.

the lengths of the sides BC, CA and AB be a, b, c respectively.

Equation
bisector of

to
is

the

internal

57
Ftg.No. 34

LC

A A
(1)

Equation to the external bisector of


bisector of
is

i.e.

internal

produced and

angle between

CA

^Bixb-a

by

8,

56

Vector Analysis

Equation to

external

bisector of

Z.S,

i.e.

internal

i.e.

bisector of angle between

CB produced and BA

a-b

is

by

8,

...... (3)

Now
paring,

(2)

and

(3)

intersect

at

L say and hence on com-

we have

or
'ft J I

JL-I

V
or

abc
2 c -{-bc-\-ac

J
ab\
c
*

(ab
s
S

abc

J
be

or

(a-\-b-c)bc

=j+b=c'
Substituting the value of s in
(2),

we

get the point

as

or
r

1+ +

-.

---

fl+i-^

r~r~" r~r~"^ a+b~

a + ~TT

This point
*
if

will lie

on

(1), i.e.

we choose

Hence the

three bisectors are concurrent.

Centroid, Line and Plane

57
oj any angle of a

Ex.

3.

Prove

that

the

internal

bisector

triangle divides the opposite side internally

in

the ratio of the sides

containing the angle.

(Pb. 60,
to

Lucknow
is

B. Sc. 53)

With reference

Ex.

2,

the

position vector of the

point of intersection of the internal bisectors

Thus

(i)

is

the centroid of the


/3-f y

point a and
respectively.
-

vuth associated numbers a and

Now a

corresponds to the point A\ therefore


vector of the
point I) \\here
the
is

--

is

the position

internal bisector of the

ancle

A meets BC.

But

>:

the centroid of points

and C with associated numbers

/?

and y and therefore divides

BC

in the ratio y

/2

or

AB

AC.

Hence proved.

Alternative.

^A (Origin)

>

3D
Fiy.No.tt
origin, let the
let

Taking A as
be

position vectors of

B and C

b and c The

respectively and

AB*=y and AC=f}.

equation of the internal bisector

AD

is

A A

58

Vector Analysis

,. t
For
Choosing
all

......... (1)
py

values of

/,

we

get a

By *=T-> we

point on the j8b4-yc


.

bisector.
.

g et the P oint

on

!t >

but

li

1S

the centroid of the points b and c with BC. ft and y and is therefore a point on

associated

numbers

Hence

-^
:

gives the point

D which
BC

clearly divides

BC

in the ratio y

or

AB AC.
:

Note.

1.

Writing the equation of

as

and solving with (1), we can D direct as found above.


2.

find the position vector of point

By

considering the external bisector,

and
can

its

intersection with

BC

is

at

the point

P-y

----

We

now show

that external
in

bisector divides
sides

side externally

the ratio of the

the opposite containing the

angle.

Ex.

4.

ABC

is

a triangle in

which the internal and

external

bisectors of angle
the middle point

A meet of DD'

the opposite side

BC
the

in

and D'.

A'

is

Similarly B' and C' are the middle points

ofEE' and FF'


are collinear.

respectively.

Show

that

points A', B' and C'

With
_, D

reference to Ex,
,
l

2,

the position vectors of


,

and

are
.

j8b4-yc
/,-.

P-ry
A' A
1S

,. ~ 5 *and Bb-yc respectively.

p-y

^b

4-yc

+
,

j8b

-*
- yc\
.

//3

b - y ac\

--" a

say

Centroid, Line and Plane


Similarly the position vectors of B' and C' are

59

-'

.,

,,/- jWb\ sayandi--c

say.

Now we know
position vectors are a relation of the

by

a', b', c' will

7 P. 49 that the three points whose be collinear if there exists

form

la'+Afb'+JVc'O such

that

Now
i. e.

2
(/3

-y 2

a'-f (y
2
(j8

La! +Aflb'+JVc'-i
2

-a2 b'+(a 2 -/32 b- y ? c+y 2 c~ a 2 aa


)
j

c'

and also

(^

2 2 -y 2)+(y*-a )+(a2 ~ e )=0

i.

e.

Hence
Ex.
5.

the three points A', B' and C' are collinear.


Prove that internal bisectors of the angles of a triangle

are concurrent.

have already proved that the internal bisector divides the opposite side in the ratio of the sides containing the angle. Thus if the internal bisector of A meets BC in
0, then

We

Z)-^25

an d

is a.

AD

in the ratio

Therefore the position vector of a point which divides /3+y a is


:

]8b+yC
or

The symmetry
the

of the result

shows that
internal

it

also lies on

other bisectors.

Hence the

bisectors

are

concurrent.

Ex*
ctntroids

6. (i)

The

lines joining the vertices

>/
535

tetrahedron to the

of area of opposite faces are concurrent.

(Agra 43,

Rajputana

56)

The joins of the mid. points of the opposite edges of a (ii) tetrahedron intersect and bisect each other. (Agra 34, Utkal 53)

60

Vector Analysis

Let the position vectors


of the

points
b,
c,

A, B, C,

be

a,

respectively
.centroid of

and Gj be the
B,
C,

D
9

so
,

that
.

is

b+c+d
Now

A and A

is

a.

the position vector

Fig. No*
in the ratio 3
:

36

of the point

which divides

AG l

1 is

b + c+d
'"*

+ 1 -* a+b+c+d ~~
4
result

8+1
The symmetry
be on B<7 2
,

of the
,

shows that the point


lines

will

CG 3
-.

and Z)G 4

Hence these
is

concur at the

point

which
:

the centroid of the tetrahedron.

Alternative

The

vector equations of the lines

AG

and
I

BG 2

are

r=U-/)a-Kl

+ c+d\
:-

/1N (1)

they intersect, then for some suitable values of / and the corresponding values of r should be identical. ComparIf

ing,

we

get

l-/-, 1-J-,
(1)

--,-.
S
t

Putting in

or

(2),

we

get the point

of intersection as
it

and the symmetry shows that

lies

on other

lines also.

(ii)

The

mid. point of

DA

is

--y- and that of'#C

is

Centroids, Lines and Plane

61

and the mid. point

of these mid. points

is

and symmetry
Ex,
of
the

can proves the theorem. also prove the same by considering the vector equation.
of the result
7.

We
the

The four diagonals of a parallelepiped and


edges are
concurrent
at a

mid. points of opposite

pins common

point of bisection.

Taking
of A,

as origin,

let the position

vectors

B and C

be a,

and c respectively so that those of I), E and

F
of

are

a+b, b+c, c+a


and
that

respectively

is

a+b+c.

If

M!

be the mid. point of diagonal OG, then

t is

a+b+c
If

Af2 be the mid. point of diagonal

AE and

2 is

a+b+c
Similarly mid.
also the point

{ e

M
DC
.

point of other diagonals


is

and

BF

is

whose position vector

Again mid. point

of

BD
CF

is

(b+a+b)=s* ~

and

mid. point

M of
of

is

/.

mid. point

LM is

i
~~ >

a+b+c

62

Vector Analysis
is

which

same as the mid. point

of diagonals.

In a similar manner
of the join of mid.

we can show

points of opposite

sides

that the mid. point is also given by

a+b+c
2
Ex*
*

Hence proved.

8.

a parallelogram

IfM,N are the mid. points of the sides AB, CD of ABC D, prove that DM and BN cut the diagonal
of trisection which are
respectively.

AC

at its points

also the points

of trisection

of DM and J5JV

(Agra 48)
let the

Taking A

as

origin of B,

position vectors

C and

be b, c and d respectively so and that position vctors of

M
is

JVare

and

Now

equation to

AC

r=/c=J (b+d).
i.e.

...a)

AC=AB+BC
Again equation
Equation of
or
of

DM

is

v**(l-s)

d+*.~

(2)

AB

is

(3)
'2

V
(1)

c-b+d.
,

and

(2) intersect at

therefore
s

we
we

should be able to
identical

find

suitable
r;

values of

and

which should give


get

values of

comparing the

coefficients,

point

is

--

or

AE=$AC.

Centroid, Line and Plane

63

b-2d

Again
or

=f (AM.

=IDM.
-

Similarly

we can prove
is

that

AF=\ AC

and

Ex. 9.

A BCD

a farallelogram.

M and N

BF=f

BJf.

are the mid-

points of the sides

AB

and

BC

respectively.

Prove that

DM and
51)
are

DN trisect the diagonal


Ex.
produced
section

AC.

10.

Three concurrent
respectively.

(Agra 48, Lucknow straight lines OA, OB OC


9

toD,E,F

Show

that the points of inter*

of AB and

DE, BC and EF, CA and

FD are collinear
(Agra 45)

A,

B C
9

Let us choose the point as origin and the points as a, b, c so that the points D, E and F may be
c respectively. Equation to AB is r =(!/>) a+/>b
/ja,
/ 2b, / 8

taken

(1) (2)

Equation ty
If

DE is

r=(l-g) tp+q.tjb

at
to

l9

they intersect say we should be able

find values of

p and

which

give
r.

us

identical

values for
paring,

Hence comFig. No.

39

and />=

1-qttti-qti
or

or

Substituting the value of p in


position vector of the point

(1)

or q in
is

(2),

we
e.

get the

which say

rx

t".

OXV

64

Vector Analysis

Similarly if ra r3 be the position vectors of the points of the intersection of other pair of lines, then proceeding as
,

above,

Now we know
are rx , r2 r3 will
,

that three points

be collinear

if

whose position vectors there exists a relation of


Adding
(3),

the form xi^-fj>ra


(4)

f ^r3 =0 where tf+^+^0.

and

(5)

7 P. 49),

ll
(1

-/!)(!-/!)

(!-/) (l""
-

i.e.

^^+^2+^3=0 where
is

which

clearly
,

zero,

Hence the
converse
real

points

whose

position

vectors are r 1? ra r a are collinear,

Ex. XL
points of

Prove

Ihe

of the

last

exercise,

i. e,

if the

intersection

are

and

collinear, then

DA, EB

and

FG

are concurrent.

to

DA and EB. Join that F also lies on OC. Taking as prove let us choose that -4, B, C are a, b, c and and origin, F lies on OC, then position are ^a and f 2b. In case
Let
be the point of intersection of

C and we must

vector, of

vector of

F should come out to F be *a+jb-f sc.

be

/8 c.

Let the position

BCis

r(lr-(l
intersect.

BF
BC
and

is

comparing the coefficients, 1 -/>=(! -0) fa +ft? and J)S and 0*=0. When gxsO, we get either #s=0 or A?=0. But if ?=0 then clearly p is also zero and we get from 1st that f2 =sl which

On

Centroid, Line and Plane

65

shows that

B and
#=0.

coincide which

is

impossible and hence

q^Q

but

Similarly

considering

the

points of

intersection of

CA and DF, we
is

get.y=0. Therefore the posi-

tion vector of

given by

-> c=,cOC.

Hence

lies

on

OC

or

FC

also passes through 0.

12.
the

Using

the

vector
the

equation

of a straight

line,

show

tliat

mid. points

of

diagonals
the

of a complete quadrilateral are


i. e.

collinear.

Establish

also

harmonic property ofthefgure


is

each

diagonal of a complete quadrilateral

cut harmonically by the

other two.
(a)

Let

ABCD

be

any two

quadrilateral,
of
its

diagonals

being AC and ED. Let BA and CD meet at E and AD and BC

meet

at

F;
is

then
a comFtg.No.4O

ABCDEF
plete

quadrilateral

and

EF

is

its

third diagonal.

Let

P,

and

be the

middle points of the diagonals BD,

and we have to prove that

P, Q,,

respectively are collinear.

AC

and

EF

Chooss
vectors of

as origin of

coordinates and let the position


if

A and C

be a and c respectively, so that

B and

D may
BC

be taken as/>a and #c respectively, the equation to joining pa and c is

The

equation to

r(l-0.*a+fc AD joining a and


r=(! -j)a+,y
intersect

(1)

qc

is

0c

(2)

Now

(1)

and

(2)

at

F and

hence on comparing

66

Vector Analysis

we have

l-t)p=l-s ............ (3)


!-*
t=sg ............... (4) or s(l-pq)=\-p.

Substituting the values of t in (1) the position vector of the point F as

or of s in

(2),

we

get

.........
the mid. points of AC, BD and EF respectively so that their position vectors are say r 1? r 2 r3

Now

P,

Cl,

are

*i

(a+c) and r 2 =4

Now
Now

if

rlf r2 r3 be
,

collinear then

^^+^2+^3=0 where
scalars.
c]

and

x,

j, 2 are

any three

(1-jty)

r,=4 [(/>-^) a+(q-pq)

or

(1 -/>?)

r3 ==ra -^rt

[from (A)]

or

/><?*!- r 2 -Ml -/?0)


/>(/

Ta0
(by

where
,

- 1+1 -pq=*Q.
collinear.

Hence r1$ r 2 r3 are


(b)

7 P. 49)

AC and
B, T,
in
Z>,

Now let the diagonal BD be cut EF in points T and 5 respectively


5
will
i
,

by the diagonals
;

then the points


9

form a harmonic pencil


1

if

BT BD
t. *.

TJT> or H. P.

^ and

are
joining

-AOm A. P.
a and c

112 +^

and

-65 are

Now

equation to

AC

is

(1)

Centroid, Line and Plane

67

Equation to

BD
EF

joining pa and gc

is

.qc

......... (2)
to point

Equation to
of part
(a) is

joining

origin

given by

(5)

AC and BD
and
(2),

intersect at

and hence on comparing

(1)

we

get

.*.

(l-^)~(l--0/> or
(2),

rl*^ and

-s
=*iplq

Substituting in

we

get the position vector of

as

~
/.

length

BT
1

is

the
j?

module
'

of

BT.
'

__ I
"|/-jc|
at

Sr^i-^
Again ing (2) and
fiZ)
(3),

...... ^ w

and

F intersect

5 and hence on compar-

we

get

68

Vector Analysis

/.

position vector of S
888

i. e.

ES
t

\pq
*>/i

-1"

rp(i

q)

f_TT~

Q (Is-P)

*~i _

T
r

_1-p-q

Again

BD-ED-EB=qc-pa=-(pa,-qc).
'2

Now

from
1
,

(4)

and

(5),
l

we

get

_L

BT^BS

R^-L"
/
?

r/w-^c| Li~^

Hence proved.

Theorem of Pappus
Ex. 13.
(a) If there
that

be two sets of collinear points


the

A l3 A, A&

Bl9
of

B%i B$; then prove

Q
/
'\^

the pair points of intersection of


lines
,

A^B^ A 2 B^ A^B^

\
^

A^B2 A^B ls
Let

A^
be

z are collinear.

^
7

the

point of

/ ^>-^^'

\
^^^^B^

intersection of

^g^g^ and
,
'

^2/^y^i^^

B^B^

which may be taken

^^^^^

as origin. Again suppose that A 19 AK A z are

A,A

^^^^^

Fig No.41

Centroid, Line and Plane

69
If

P&L and
points of
to

B,,

2,

53
,

are ftb, ? 2b,

g 3b.

intersection of the

three pairs of

Clf C2 C8 are lines, we have


,

prove that

Clf C 2 C 3
A 2 B%

are collinear.
/> 2

Equation to

joining

a and

qjb

is

r~(l-OAa+'ftb
Equation to ^ 3 Z? 2 joining />3 a and
Since they
#a

...... (D

...... (2) r=(l- s) A*+*ftb intersect at C l9 we have on comparing

or

A-

Ifrjbe the position vector of the point C 1? then on substituting for * and (1 *) in (1), we get
*i

(Prtt-pzq^PiPs

(ft -ft)

a+ftft

(P2

-A)

c.

Multiply both sides by /^ft,


or

r^ft Mt-Ms)^!^ A-ft (ft- ft)


(

+?iftft-A ( A-#s)

r2 and r 3 be the position vectors of other Similarly points of intersection, we have


if

't-Mt^sft-M^iPtA-ft
'3.'.

(ft -ft)

a+?iftft-A

(&-A) b
-

Ma (Aift-Aft)-^t A-ft (?i-ft) a+Jrfrft-A (A -A) b


27 rjL.ftft

(Aft-A?s)-O
2

i.e.

Irx+Af^+JVra-O
,

i+A4+JV is also zero. Hence Aft (Aft Aft) the three points whose position vectors are r ls ra rs are
where
27

-^

collinear.

Theorem of Desargues.
Ex.
lines

13. (b)

If ABC,
are

A&Ci be two

triangles such that the

AA 1} BB l9 CC X

concurrent, then

the points of intersection

of pairs of sides BC,

BjC^ CA, G^Atf AB, AJB^

are collinear.

70

Vector Analysis

Mr. N.
M.Sc. (Maths)
Fiq No 4?

U*

Note

For Ceva and Menelaus Theorems see Q. 9 P.


the

98,

and Q. 10 P. 95. Ex. 14* Through


parallelogram

middle point

P
is

of the side

AD of a
AC
at

ABCD,
at

the straight line

BP

drawn

cutting

and

CD produced
as

Q.

Prove that

Take A

as origin and

B and b

b and d

res-

pectively.

Now

proceed

exactly as in Ex. 12, 2nd


part.
A

FigNo.43

Ex* 154
and

The

median
to

AD of a
and.

triangle

ABC

is bisected at

BE is produced
Ex*
16.

meet the side

AC

in

F.

Prove that

Forces

P9

Q.

act at

and have a resultant R.

If

any transversal cuts their lines of action in A,

P
then prove that

d
-

and

respectively,

R
(Agra 49, Lucknow 42)
Q,

Let the forces Pand


-

be represented by OL and -

OM

so that the diagonal represents the force R so that


/Z

ON

...(1)

Fig No- 44

Centroids, Lines and Planes

71

Again suppose -

that

-* P=*m>OA and Q*=*n.OB and

let

R^t.OC.
.\

-^,-^and/-^r

(2)

Now

substituting for P, Q, and

-/=0

in

(1),

we

get

m.OA+n.OB=t.OC
or

(3)

m
But A,
B,

are collinear

and as such sum


in

of the coeffi-

cients of their

position vectors
zero.
/.

a relation connecting
m-{-n=t.
/

them should be

m-fw

or

Putting the values of m, n and


required result.

from

(2),

we

get the

Ex. 17. Prove that the straight lines joining the mid. points of two non-parallel sides of a trapezium is parallel to the parallel sides and half of their sum.

We
PQ
is

have to prove that AB and parallel to

equal to 4 (AB+DC).

PL
let
/
J:

Taking
be

_
*
~

as origin,

the position vectors of

B andD

^
Fig.No*45

b and d

respectively.

Now DC

is

||

to

AB]

.\

DC**t.AB**t.b,

where

.\

position vector of

is

AC**AD+DC**d+fo.

72

Vector Analysis

being the mid. point

AD
of

has

its

position vector d/2


its

and Q, being the mid. point

BC

has

position

vector

Since PQ,
parallel to

is

some

scalar multiple of

AB, hence

PQ

is

AB

and hence to DC.

A,so
/.

PQ.-1 (AB+DQ.
/Ae?

Ex,
of

1 8.

P;0#e //m/

straight line joining the


is

mid .points

oj

the diagonals

of a trapezium

parallel

to parallel

sides

and half

their difference.

Ex,

19,

Prove that in any triangle the line joining the mid.

points of any
length.

two sides

is

parallel to the third side

and half of its

(Agra 56, Rajputana B. Sc. 60)


220.

Ex*

Prove that

the

diagonals

of a parallelogram bisect

each other and, conversely, if the diagonals of a quadrilateral bisect

each other,

it is

a parallelogram.

(Agra 36, Lucknow B. Sc. 54)


formed by joining
is

Ex. 21.

Show

that the figure

the

mid. points

of the sides of a quadrilateral taken in order

a parallelogram.

(Lucknow
Ex. 22.
triangle,
the

48)

// a straight
line

line is

drawn parallel
to

to the base

of a

joining

the

vertex

the

intersection

of the

diagonals of the trapezium so formed

bisects the base

of the triangle.

(Agra 59)
Taking
be

as origin let the position vectors of

B and C

b and c

respectively.

Now

JfM

is

parallel to

BC.

Centroid, Lire and Plane

"

AJf

AM
position

AjrAu=*^
N
and
to

Therefore
vectors of

are

*b and xc respectively.
Equation

BM
CJV

is (1)
?

r=(l

0b+*.xc
to

Z>

Equation

is

Fig.

No -46

Comparing
or

1-;

(l-s)c+s.xb sx and !-$=*# 1-*


.

Hence

the point
to

is

given by

.-

(b-f c).

Equation

AO

is

r=

Again equation
Relations
(b)
(8)

to -4Z)

where

Z)

is

mid.

point of

BC

is
(4)

r=i(b+c)
and
(4)

show that

/) lies

on

AO

produced*

parallel to the sides, the

If through any point within a triangle, lines be drawn sum of the ratios of these lines to the corresis 2.

ponding sides

(Agra 51, 61)


as

Take the vertex C


origin let the
tors of

position vec-

A and B be a and b
and
that
of

respectively

point
c.

within the

be

Stoce
therefore
of
if

PQ

is

to

\\AB

P be

/a,

position vectors then then that of

74

Vector Analysis
be /band also Q=*l .................. (1)
let

will

P AD
:.

Again

the position

vector of

L
to

on

CA

be ma.

Now

in

&ABC,
.

LM LM

LA=(\-m)a.. is drawn parallel

BC.

LA

Similarly let the position vector of


.'.

on

CB

be nb

fl=(l-n)b.
x- i

RS^RB n
A

LT~

""""

-i

-*-

yj)

Now
Its

PQ, passes through /a and /b.


is

equation
is

r=(L-/)

/a-(-//b

............
is

(4)

RS
Its

passes through

i.e.

nb and

parallel to

CA

i.e.

a.

r^=nb+ja ............ (5) iW passes through L t. e. ma and is parallel to CB i.e. b. Its equation is r*=wa+ttb ............ (0)
equation
Since
all

the three lines

(4),

(5)

and

(0)

intersect

at

0,

therefore

we have on comparing
(1

the coefficients,

- /) l=s, tl^n
from
(1), (2)

[from

(4) (5)

and

(5)]
(6)].

and

H=M, s=m

[from

and

Now sum

of the ratios

and

(3) is

Hence proved.
Ex. 23.
Prove that the sides about the equal
angles

of equi-

angular triangles are proportional .

ADE having a common vertex at A. We AB^AC^BC AD AE DE'

Let us consider two equiangular triangles ABC and have to prove that

__>_

Let

AB**c, BC**a, CA**b.

Centroid, Lire and Plane

AD

EA

Kg. No 47

Again
or
or
(2)

- *a (a+b)+* 1 m+ t t b=

[from

(1)]

Now we know
exists

from
of

relation

5 page 16 chapter I the form ^a+^b-^0

that

if

there

non-collinear vectors,
\ve get

then

*0, j=0.

between two Hence from (2),

or

AB
Ex. 24.
side

=^ CA
BC

/A a parallelogram

ABCD
The

a point

Hence proved* P is taken on the


the diagonal

AD,

such that
.

n.AP^AD.
Prove that

tine

BP cuts

AC

AC

in the point

(n+ 1)

AQ^AC.
B. Sc. Supp. 48)

(Lucknow
Taking A
b,
c,

as origin let be the position


B,

vectors

of

C and D
and
since

respectively

--*>

we have

76

Vector Analysis

b+d-c ............. (1)


"*

AP=
Hence from
Above
be zero.
is

"*

say.

AD=*

.d,
fz

AQ**x.AC
J _.

-*
(I),

-*

we

get

AB+nAP-

AQ=Q.
of

relation

between the position vectors

three collinear points;

hence sum of the coefficients should

or

Ex. 25.
of the same.

ABC is any AB BO, CO


9

triangle

and

any point

in

the plane

meet

the

sides

BC,

CA

and

AB

in

D,

E F
y

respectively.

Prove that

Ex. 26.

(a) Prove that the points are collinear.

a-2b+3c, 2a+3b-4c, -7b + J0c

Let the above points be A, B, C respectively relative Now if we can find three scalars L, M, JV to any origin 0.

--Hence

such that L.
then A, B,

OA+M. OB+N. OC^O

where

are collinear.

we

find that 2 (a

Choosing L=2, - 2b-f 3c)~ 1 (2a+3b - 4c)collinear.

M
1
(

I+Af+JV=0, - 1, JV= 1,
- 7b+ L0c)=0,

where

21

1=0.

Alternative.

2a-10b+14c.

Centroid, Line and Plane

77

->

.*.

We observe
(b)

that

BC=24C.
the points

B, A,

are collinear.
fc,

Prove

that

whose position vectors are a,


54,

3a-2b
points

are collinear.

(Delhi 50, Banaras


the vector

Agra

55, 58)

Ex. 27.

(a)

Find

equation

of the

line joining the

i-2j+k
Stoie;

and

3k- 2j.
Zine

Ans.
(b)

r(i-2j+k)+/(i-2k),
#za*

joining

B (8, -1,
Ans.
4c and

2) has equations

-^=1^ =$--. Find two points on


A
is

the points --

(2,

-3, -

Jj

dwd

the line whose distance from


(14, 1, 5)

14.

and (-10, -7, -7).


the points

Ex.28*
-4c.
Ex. 29.
'the

Prove that the line joining


joining

6a-4b+^c,
5c
intersect at

the line

-a

2b

3c,

a+2b

In the triangle

ABC, points
at

and
that

are

taken

in

AB sides CA and AR=2RB. BQ and CZ


Also find the ratio in which

respectively
intersect

such

CQ^QA
that

and

0.

Prove

CO=30R.

AO

divides

BC.

Let a, b, c be
position A,

the

vectors

of

the
res-

points pectively so that

B and C

under

given conditions the position

vectors

of

Q,

and
.

C
are
J&

and

a
c4-a

-^ o

Equations to BQ,

is

Equation to

CR

is

r(l-*)b-M -^- ......... (1) 2b+a - -~r-(I-,s)

c+*

..

...

.(2)

Now
we
get

(1)

and

(2)

intersect at

and hence on comparing,

Vector Analysis

The

position

vector of

is

therefore g

+}(c+ a)
position

=|(a+2b+c).

Now
vector of
is

if

divides

C# in the q 2b+a
8""" g -*

ratio

3:1, then the

should be

- -

or

(a+2b+c) which

true.

Again equation to AO is r=(l-Oa+*.J(a+2b+ c). Since they intersect at P we have on comparing,

LH-^O,
From
first

1-1-., i-..
satisfy

two we get /=f .,y=J and these

the

third relation also.


/. If

position vector of

is

P divides BC

in the ratio

1,

then

position

vector

of

is

^-nr
-- =i
BC
in

Comparing,
/.

and

-=f
:

/.

m=^.

divides
-4

the ratio 1

2.

Ex, 30.
side.

linejrom a vertex of a
similar
line

triangle- bisects the opposite

It intersects a

issuing

from
:

the

oilier

vertex.

Prove that these lines intersect in the ratio 3

L
show that the
lines

Ex* 31.

// a, b, c

be

non-coplanar,

whose equations are T**(-

J0a+b-cj-H

are coplanar> and find their point of intersection.

sect for

coplanar, then they must interwhich we should get identical values of r. Comparing a, b and c in the two equations we will have three relations in s and /. Solving the two we find that the
If

the above lines are

Centroid, Line and Plane


third
is

79

also satisfied
is

and hence they intersect and the point

of intersection

obtained as

6a+5b

7c.

Ex. 32.
which
is the

Prove by vectorial methods the equation


line

--(-:=./
intercepts

equation of a straight

in

terms

oj

its

on the axes of reference.

The axes may be rectangular or oblique.

Let the position vectors of and B be a and b respectively


b,

and their modules be a and

A
so that

a=0.a and b=i,b. Let P be any point (x, y)


line.

on the
and

From P draw PM
OB, so that

parallel to

OM =x

MP=y.
.a and

Now P being

any point on AB,

Also
a

Comparing a and b

in (t)

and

(2),

we

get

Eliminating
tion.

t,

we

get

+ ^- = 1 as the
the

required equa-

33
to the

The straight line through

mid. points

of

three

coplanar edges of a
point,

tetrahedron, each parallel to the line joining

ajixed

mid. point of the opposite edge are concurrent at a point


is bisected

P such
hedron.

that

OP

by the centroid

(of volume)

of the tetra-

(Agra 46)

80

Vector Analysis

Take
the
position

as

origin

and
9
9

let

vectors of
c,

A B C

and

be a, b,

d
t

respectively.

Let P 1 and
points
of

be the mid.

opposite edges, so that

their position vectors are

b+c
Now
through

and

a+d
2

the equation of a line


parallel to

P l and

OT

is

Similarly

we can

write

down

the

equation of line

through Sj

i.e.

c+dand
-

pirallel to 0/? a

where

#x

a+b
is

"t

as

2V 2;/"
fcj
i.

'

.(2)

The equation
lei to

of the line through

e.

b+d and

para-

00

where
r

is

a+c -- x

is

b+d
intersect at

Now
we have

(1)

and

(2)

P and

hence on comparing

/=5 = 1
if

Again

and we get the point we choose />=*!, the point

as $ (a+b-j-c-f-d).

satisfies (8) also.

Hence the above three

lines are concurrent at P.

Now

mid-point of

OP is

HO+Ha+b+c+d)]~
which
is

a+b + c+d

the cetitroid of volume


is

of the tetrahedron*

Hence OP

bisected at G.

Centroid, Line and Plane

81
show
that the

4
points
only if

If a*)* are two


/.a-f^ab,

non-collinea)
/s

vectors,

L-ftf'ib,

a-rff*3b=0

are

collincar

if

and

rij

wg

rn$

(Agra
If the

given vectois are collinear, then

we must have
(1)

where
or

x+y+z^Q
18
(xl

7.

P. 48]

+j>/2 -f c/3 )

a + (*m i +J^2 + ^2) b = 0,


are

Since a and
there
exists

two
of
1,

non-collinear

vectors and

if

a relation
5 chapter

the

type

/a+tfzb=0,

t] ien

/=0,
or

m*0

P. 1GJ

*/i+jtf|+-rf 3

-0
(2).

(2)

Eliminating

x, y,

hetwen

(U)

and

(1),

we

get

the

required condition.
$ 9.

Vector equation of a plane.


tin vector equation
ts

To find
the origin

of a

plane
vectors

which passes

through

and

parallel to

two given
let

a and b.

Choosing
the vectors
b.

as origin

OA

and

OB

be a and

P be "any point on the From P draw PM and plane. PJf parallel to OB and OA resOA and OB pectively meeting
Let
in

Fig.No.52

M and JV respectively.

OM bemg collinear with

82
-

Vector Analysis

ON being collinear with


If

OB^tb.

r be the

position vector of

P any

point on the

plane,

then

r=OP
Hence the vector equation
of

the

required

plane

is

given

by
s

where

and

are scalar* which vary as the point


5 P.

P moves on

the plane.

Note.
Cor.
given point
i.

See

Iti

Chapter

I.

To find

the

vector

equation

of a plane through a

c and

parallel to

a and

b.

[Agra 40]

of

Let c be the position vector any point C on the plane and


point on
is
it,

P be and
vector

Now

the

GP

coplanar

with

a and

b and
If

as such CP=ja-|-/b.

Q
I

fiy.Nc.53

(5

chapter

P. 15).

r be the position vector

of P, then

Hence the required vector equation of the plane is r **c -Ma *Hb wh^re s and t are scalars which vary as the the plane, point P moves on
Cor.
a.

To find

the vector equition *of a


vectors are a,

plane

that c.

passes

through three points

whose position

b
of

and

(Agra 39)
Leta,
b,

cb2

the

position

vectors

the

points A,

S and C

respectively on the plane, so that

^4Bb

and

Centroid, Line and Plane

83

Now
one

the

required

plane

is

through

the

point
i.e.

and

parallel to

AB

and AC,
to

a and

parallel

b-a
1, its

through and c-a


is

A
fiq.No>54

and hence by Cor.


or

equation

r*(l-$-J) a+sb+te..

..

....... (3)
through

Cor. 3. To find the vector points a and b and parallel to c.


If

equation of a plane

two

a and b be the
then

position vectors

of

A and B
is

respec-

tively,

^B=b-a. Hence
is
is

the required plane

one that

passes through a and


Cor,
1, its

parallel to

b-a

and c and hence by

equation

or

r=(l-j)a+jb+fc ........... (4)


Cor.
4.

To find

the

Cartesian

equivalents

of the vector

equations of planes found

above.

(Refer author's Solid Geometry.]

Case
given
Let

i.

Plane through origin and


and

parallel to the

line.
in

terms of unit vectors

i,

through origin

G, the vector
so that

is

(x,y, z).

so that direction cosines of vector

and

baa^i+taj-f *k

are proportional to <Z], a a , a 9 , so that direction cosines of vector b


are proportional to ov b tt b+

(a)

Now

vector

equation of a plane through origin

84

'

Vector Analysis

and
or

parallel to vectors

a and b

is

Equating

coefficients of

i,

and k, we get

Eliminating
*

-5 and
ai

-/,
bl

we

get

=0.

or

Above

is

the

coi responding
arid

cartesian equation
to
3

of

plane through origin

parallel
,

lines

whose
.

direction

cosines are proportional to a^ a 2


(b)
llel

and

b l9 b 2 , b 3

to

Case a* Plane through a given point and paratwo given lines.


straight lines
is

Vector equation of a plane through a point and parallel


to

two

r=sc-f Ja+'b
or

[by Cor.
ai;d

Ij

(r-c)=jfl+/b.
Putting in terms
of

unit

vectors

equating

coeffi-

cients of

i,

and k, we get

Eliminating

^,

/,

we

get

=0

aa

Centroid, Line and Plane

or
*2
1

*2

*3

*3

Above

is

the

corresponding cartesian equation of a


,

plane that passes through the point (c^ c2 c 2 ) and is parallel to two lines whose direction cosines are proportional to
a v a * a s an d b l9 b 2 b z .
(c)

Case

3.

Plane passing

through three given


is

points. Vector equation of a plane through three given points

r=(l-,y-/)a+,yb+Jc
or

[Cor. a]

r=a + J (b-a)+f
i,

(c-a).

Putting in terms of unit vectors and equating coefficient?


of
j

and k, we get

Eliminating

and

/between the above equations,


=0.

we

get

#3

bz

a$

r3

The

above determinant can be written as a fourth order


:

determinant as following

Vector Analysis

Adding fourth column to

first,

second and third column,


x

we get
x
bl
cl

al

or

1111
or
x

=0.

Above
(fj, C 2 ,

is

the

a plane -through
C3\

corresponding cartesian equation of three points (a l9 a 2 a 3\ (b^ b^ fc a) and


,

10*

Condition for four points to be coplanar*


that the

To prove
four
points in
exists

necessary

and

sufficient

condition for any


that

three-dimensional space

to be

coplanar is
vectors

there
that

a linear relation connecting their position

such

the algebraic

sum of the

coefficients in

it is

zero.

(Pb. 60;

Agra

37, 54)

seen that the vector equation of a plane through the points whose position vectors are a, b and c is

We have

(l_,y_f)

a+jb+'c0.
be written as

[Cor. a]

The above
Above
is

relation

may

the positron vectors of ) which are coplanar and we points A, B, C and observe that algebraic sum of the coefficients of a, b, c and r is 1 J /+$+* 1 which is zero. Hence the condition

a relation between

four

is

necessary.

Centroid, Line and Plane

87

In order to prove that the condition is sufficient let us suppose that any four vectors a, b, c, d be connected by the relation Ja-f;wb-f-;ic-|-/>d0 where

/+m-f n+p=*Q.

Dividing by p (p^Q),

we

get

d==
.

_L a _.. b _
I

c
,

where
Putting

m +-r +
,

/>

~-i.
=/,

~s

and

'-;-;
shows that d
hence
a, b, c,
is

'++
a point on the are coplanar. plane

Above
through
a,

relation

b and

c;

Alternative Proof.
Let us suppose that the points A,
tion vectors
B, C,

whose
let

posi-

are a, b, c,
at

are

coplanar

and

AB and

CD

intersect

P
if

not parallel and

being assumed that AB and CD are they are, then we will choose any other
(it

pair of non-parallel lines formed


If

by given
'

points).

P divides AB
:

in the

^o
A~
wi^^-^^P
"

ratio

q and

CD

in

n,

then

its

position

vector
is

written from

AB and CD

Fig.No.5S

m+n
or

m+n

where
Hence the condition
is

necessary.

88

Vector Analysis

Note.
point

From
-

here

we

find

the position vector of any

d on

the plane through a, b, c as

Lm+Afb+JVc

Lm+Afb+Afe

Converse*
Again
let /a-f

wb + flc+/>d=0 where

/-f

m+fl+/>=0 and

we

will

show

that the points A, 5, C,

are coplanar.

Now
is

of the
if

not zero for

three scalars l+m, /+, /+/>, one at least all of them were zero then

/.

Also

n-p=Q or /+w+n+>=0 but


;H

w=0,

or
/.

/?=*;
is

/.

^==.

;=0=m = w=:j&

which

impossible.

Let us suppose that

l+m

is

not zero and therefore

From

the given relation,


/a

we

get

+ mb

"""

/+^i
L. H. S.
/

is centroid of points A and B with associated and TW and hence is on AB, and R. H. S. is centroid of C and D with associated numbers n and/*, hence is on CD. Hence we find a point on AB is same as a point on CD showing that they intersect and hence A 3 B, C, D are

numbers

coplanar.

Also the point of intersection of

AB and CD
'

is

Ja+mb l+m'

nc+pd n+p

Centroid, Line and Plane

89

Exercise
Ex.
i*

//

any point

within a

tetrahedron

ABCD

is

joined

and AO, BO, CO, faces in P, Q,, R> $, show that


to the vertices

DO
i

are produced to cut the opposite

OPOQ,OR OS_
(Agra 33,
Taking
b,
c,

35, 39, 43, 46, 49, 58, 61)

as

origin

let

the
a,
fi

position vectors of A, B, C,

be

respectively and we know that any four vectors are linearly


i.e.

dependent
tion of the

there

exists a rela-

form
...(1)

Fig No 56

r=s

Equation of AO is r= a and hence equation to OP is ksi where k is a scalar positive for points on AO proive for those
(1),

duced and

on OA.

But from

we have

mb+nc+pd
^

Putting for a from above the

equation to OP\

is

given

(mb+nc+pd)
Again equation to plane BCD is r=(l-,y-f) b+5c+/d [Cor. 2]

.(2)

...

...(3)

Now

(2)

and

(3)

intersect

at

JP

and hence we have on

comparing,
^m
i

*?

*A_

90

Vector Analysis

Adding,

y(m+n+^)l

or

k^---

...... (iv)
a.

Hence
A

position vector of 1

is

m+n+p
w

Aga.n

4P
>

OP AP
0(1
'

----;/
l-i-

Proceeding as above,

we have

n m OR_ BQJ*l+~m + n+jr ~CR^l + m + n+p'

OS
and
,

T^c-=7 ,~ ; D6 l+m + nrp

the

Adding, we get the required result* Find the equation of the plane through the origin and Find also the point in which this plane points 4j and 2i-fk.
2.

is cut

by the

line

joining the points i

2j+k

and

5k

2j.

(Agra
Equation of plane
Equation
of line of
is

42, 45, 56)

is

r=j 4j-H

(2i+k).

r=*(l~/>)

(i-3j+k)+/> (Sk-2j).

For point
find

intersection,

that

f|,

j-i
the

comparing and solving, we and />=~5', an 1 hence the point is


points

K6i-10j+3k).
3.

Prove

that

four

2a+3b-c,

a-2b-f-3c,

3&+4b-2c
ist

and

a,6b+6c
will

are coplanar.

Method.

Let the given points be A, B, C,

respectively. intersects the

We

line joining

prove that the lines joining any two the other twxo provided they are

not parallel and hence the four points are coplanar.

Centroid, Line and Plane

91

We
/.
.

find that
let

AB= -2CD

i. e.

us consider the lines

AB is parallel AC and BD say.

to

CD.

Equation to 4C is r=2a f 3b-c+/ (a+b-c). Equation to BD is r=a-2b+3c+.y (-4b+Hc). If they intersect, we have on comparing,
Solving the first two, we find t=* -1 and s= 1 and these values satisfy the 3rd equation as well. Hence the

two

lines intersect showing thereby that the four points are coplanar and the point of intersection is a-f-2b.

and Method.

AB=* - a- 5b+

Ic,

and
the given points are coplanar, then the three coter-* ~ should be coplanar and as such minous vectors 4S, ^!C,
If

AD

they should be linearly dependent, t. *., there must exist a relation between them. Let us suppose that

or

/(-a-5b+4c)+w(a-hb-c)=-a-9b<+-7c. Comparing; we get

-/+:= -1, -

92
Solving the
values satisfy
first

Vector Analysis

and these -* the third relation 4?-ro=7, and hence AB, AC


two,
get /=2,

we

m=l

and

AD are coplanar.
72a
Prove

Therefore the points A, B, C,

D are
a+ 2b

coplanar. Prove that 4.

the points

6* +2b - c, 2a - b+3c, -

- 4c and
5.

- b - 3c
that

are coplanar.

the

points

~ a -Hb-3c, 3a4 2b-5c,

3a-f #b - 5c, - 3a+2b+c


6. (a)

are coplanar.

Find the point

in

which the plane

is

cut by the line through the point 2a-f-3b and parallel to c.

The
(b)

equation to the

line is

r=2a+3b+jbc.
Ans.

For point

of intersection

compare

etc.

2a+3b
(2, /,

4c
2

Find

the intersection oj the line joining the points (1,

(4,

1)

and

(2, 3, 1)

with the plane through the points


one

-5J,
and

-1,
7.

2) and (3, 0, 1).

Ans*
six

(f, |, J)

Prove that the

planes

containing

edge

bisecting the opposite edge oj a tetrahedron bisect each other.

Plane

containing

OA
plane

and bisecting

BC

i.e.

OADis
IS 9]

Plane

OCF

is

Plane

PBC

is

r-(l-Js-W J-+*sb+/c 4
Compare
intersection.

9 Cor.
a,

1].

the coefficients of

b and c

for

point of

Centroid, Line and Plane

.i.!- V-'s
2

"2

'i_

'

'

7"""2

_*
is

'i
f

8'

Hence

the point of intersection

a+b+c

Similarly

we can show

for other planes.

Prove that the middle points of the six edges of a cube 7. (b) which do not meet a particular diagonal are coplanar.

Let

A be

taken as origin

and AB 9

AD

and AA' as axes

along which the unit vectors be i, j and k respectively.

Let the edge


Q/,

of

the

cube be

of unit length. P, Q,,

R and

P',

are the

mid. points of
vector

the edges on

which they are


position

marked.

The

D
'

of these mid. points are

pj.af
P'

R^
i

i+j+k,
2 tj
, "
.

(i+i+kWj+k)_ "
2
is

.
,

Similarly, Q,'

i+4 +k

and

Now the

equation to the plane

PQR

is

...(D

The

3 point P' -^--f-j+k will

,A

lie

on

it,

if

on comparing

the coefficients,;

94

Vector Analysis

]-,-/!,
Solving the last two, we get values satisfy the first also.
Similarly,
of s

,+*-!,
.

^-1.
and these
values

/=*2, 5s=s-2,

we can show

that for

some

suitable

and /the other two mid. points Q/ and will also lie on the same plane. Hence P, &, #, />', Q/ j?' are coplanar. 8. If a, b, c be any three non-coplanar vectors, then prove that the points ^a-f T^b+WjC, / 2 a+m 2b+w 2 c, / 3 a+w 3 b
,

/4

a+m4b+w 4 c
;

are
/,

coplanar if and only if


/4

/i

*s

ll 11
the given vectors are coplanar, then there a relation of the form
If

must

exist

+w
where

(/ 4

a + w4 b +

4 c)

= 0,

x+y+z+w=*Q.
b

...(1)

there exists a relation of the form

Since a, b, c are three non-coplanar vectors and if /a+^b+c=sO, then 5 Chapter 1, P. 15] /w=/z=0, L
Q.
,

.(2)

0.

. .

.(4)
(1),

Eliminating

x, y,

^ and

M;

between

(2),

(3),

(4)

and

we

get the required result.

Therom
9-

of Ceva
points

Vp

Q,/

Rw*

three

on

the sides

BC, CA,
BQ^

AB

respectively of a triangle

ABC,

such that the lines

AP,

i>il

Centroids, Lihes and Planes

95

CR

are concurrent, then

BP -

Cd ~~

AR
.

^-

-1

<wd conversely.

Let

be the point

of intersection of AP,

BQ
d

and the position vectors


of A, B, C,

be

a, b, c,

respectively.

Since these
coplanar,

four points are

xa-

where
xa

- (& + wd)

and

The
on
to

CD
AB

point on AB and R. H. S. and both being ame, either gives the point and CD and hence R.

L. H.

S. is

is

a point

common

Thus
troid

position vector of

is

which

is

the cen-

x+y
x and

thus

A and B with associated numbers divides AB in the ratio y x. AR _. y r\r AR y


of
:

and

RB~

_,

z.

x
z
i and

c-

-i

BP
r-~^. ux
'

Similarly

y
'

BRCQ *-r^
8

^*><,

AR BR
Converse.

BP CP

Cd ^_
^(^^

x __

~lc'~y'~z~~~
Hence proved.
x

We

are given that

AR BR

'

BP CP

'

and we are to prove that AP, BQ, and C/2 are concurrent.

98

Vector Analysis

Let us assume that 7^5*=


t /n 4jR from(l),

and

~
A\^

-- and hence
z

^
get

j3#=- ^ From above, we

Suppose I) is a point on -4P dividing x and therefore its position vector is


:

it

in

the ratio

or

x+y+z
The symmetry
of the
it

result
z.

shows that

this point will

also

lie

on BQ, dividing
it

in the
:

ratio

z+x

:y and on

CR

dividing

in the ratio

x+y

Hence the three


10.

lines are concurrent.

//

P,

Theorem of Menelaus Q, R are three points in the


ABC,
.

sides

BC,

AB

respectively

of a triangle
L

such that the points P, Q,,


.

CA and R are

BP
Lti

collmtar, then prove that -~~

CQ, ~?Z
"?<,

, "&=** and DJK,

AR

Let
of

D
and

be

the

point of intersection

BQ

CR and
vectors
m

the position
of A, B, C,

D be taken
d
respec.

as a, b,
tively.

c,

Ftg,No.59

Since these four points are coplanar,

we have

x*+yb+zc+wd**Q where

x+y+z+weQ.

Centroid, Line and Plane


Just as in Q.

97

and

AB

the position vectors of Q, and respectively can be taken as


9,

R on AC

x+z

'

"

(1)

(2)

Subtracting,

we

get

(*

q- (x+y)
j,

Dividing both sides by c

we

get

L. H. S. gives a point on the line joining points whose position vectors are q and r z. e. on RQ, and R. H. S. gives
a point on the line joining b and c, /. e. BC. Since they are equal therefore L. H. S. or R. II. S. gives the position vector
of the point of intersection of

BC and RQ.

i. e.

of P.

Now

from

(1), (2)

and

(8),

we
, '

get

_
Q_A~

x
'

AR_y am RB~"x

B? PC~

,BP_z AR_
.
'

CP~ y' BR~~ x' AQ_ BP CQ AR "


CP
'

AQ_

M~

Converse.

We
CP

are given that

CQ,

AR
'

Ad

BR~
y
and -7^= ---- and hence
*

np
Let us suppose that

CO

^SBLI r

AR

Bit"' x'

98
.

Vector Analysis

BP_
if

x
'

AR
P, Q,

r are position vectors of respectively, then from above ratios,


p, q,

Hence

and

Now

if

p, q,

r are ollinear then there must

exist a

relation of the

form Lp4-Mq-|-JVr=0, such that

where

- fe -y) +(x +^) -

(x +y>)

=0.

Hence

collinear.

CHAPTER
i.

III

MULTIPLICATION OF VECTORS
There are two
different

ways by which

vector

quantities are multiplied : one is called scalar or dot product and the other is called vector or cross product*

The former
tion

is

a mere

number and does not involve any


is

direc-

whereas the

later

ass3ciated with a definite direction

and as such

is a vector quantity. However, in each case the product is proportional to the products of the lengths of the two vectors and they also follow the distributive law

just as in the product of ordinary

numbers.
is

The

scalar or

dot

product

of

two vectors a and b

written as

a b

i,

e.

by placing a dot . between a and cross product of the vectors a and b by placing a cross
a.

b whereas
is

the vector or

written as

axb

i.

e.

X between a and
(Agra

b.

Scalar product.
32, 33, 40, 44, 51, 57, 58)

Def.
and
tlte

The

scalar product of two vectors


is

a and b
is the

respectively

equal

to

ab cos

where 6

of moduli a angle between

directions of

and

Thus a . b~ab co
Again a
.

=
|

.
| |

.
|

cos

0.

b=0
is

(b

cos

0).

Now

b cos

length

of

the resolved part OL of OB i. e. module of vector b in the direction

of

a whose length
write
it

is

or

we can

as
(a

a
where a cos
direction of

b=/>
is

cos

0)

Pig No<6o

the length of the resolved part of


length
is b.

in

the

b whose

100

Vector Analysis
Hence
dot product

of two vectors

is

equal to product of the


the

length of one of them with resolved part of the other in

direction

of the former.

Again if we take that b represents a force in magnitude and direction whereas a represents a vector drawn in
an assigned direction, then
the force
the
b cos 9 is

in the direction of a.

Thus
moving

the resolved part of ab cos represents


its

work done by the


to

force

in

point

of

appli-

cation from

along

OA and work

doe? not involve


is

the idea of a direction


quantity.

and

hence dot product

a scalar

Properties of Dot product of Vectors


!

From

above

we

find that

b=b a=a&
.

cos 6.

Jrhnce scalar product is commutative.

cos 6=*
2.

or

ab

a a

.
.

b
i

The

dot product of two vectors will be -\-ive, zero or


is -\-ivr,

ive

according as cos 6

zero or

ive

which means that according

as B is acute or a right angle or obtuse.

Again a
3.
zero.

Jtow

\>ab cos 0. a b will be


.
.

zero

only

if either a or b or cos 9

is

Hence dot product of two


be

vectors is zero if either

a zero

vector

or

if they are non-z*ro vectors

of the vectors then their direc-

tions should be perpendicular.


if

[Remember],

be the angle between the directions of a and Again ir-Q will be the angle between the directions of a b, then and b or of -a and b

Thus a
or

(w-0)(-a) (b) a (-b)=(-a) b ~ab cos 0-(a


.

(-b)

aft

cos
,

b).
is

4., fn

dot pfo'duct

of two vectors if either of the factors

Multiplication of Vectors
multiplied

101
is

by minus sign,

then the total

product

multiplied by

minus

sign.

Again if be the angle between the directions of a and then since vertically opposite angles are equal, is also b, the angle between the directions of a and b. Thi\s

(a) (-b)=0
of vector

cos

(a

b).
is

Again

if

be

the

module

a then vector ma
(wa).(ttb)=(w0)

a vector

in

the direction of

but of length ma.

Thus
or or
5.
If
i.

(nb)
.

cos Q=*mn(ab cos

8)

=mn (a b) =0 (mnb) cos 0=a


.

(mrcb)
.

=(rzfl) (mb)

cos 0=fla
two

wb,

Thus

the scalar product of

vectors in associative.
is 0,

the vectors

be

like,

then angle between them


.

e.

cos

= 1.
/.

b=0fe.
vectors
is

6.

77*1/5

product of

their

the product moduli.

of two like

equal to the

In case they be unlike then


/. If

0=180,
.

i.

e.

cos

0= -1.

a.b=-0*.
a=fl
a
.

the vectors be
2
.

equal then a

cos 0=fl 2 *and

written as a2 =fl
7.

T&MJ

the

square of any vector

is

equal to

square

of

its

module.

(Remember)
there be

In case

two
.

unit

vectors
1

then their moduli

are each unity.


8.

A A Thus a b = l

cos

0=cos

0.

Thus

the product of two

unit vectors is equal to the cosine

of the angle between their directions.

Orthonorinal vector triads

i, j,

k.

We know
unit vectors,

that

i, j,

are three mutually perpendicular

3)on<*td by

102
/.
i
2

Vector Analysis

=j*k
.

and
9*

jj

kk

l (Properties 7)
.

i=*0 (Property 3).

Distributive

Law

i. e.

(b+c)=a . b+a

c.

(Punjab 60)
Taking
->
-*
as origin let

and

04,

BC

represent

the vectors a,

b and
->

res-

pectively so that

OCb+c.

and CJVbe perpendiculars from B and C Again if on OA, then and OJV are the projections of OB and OC on CM and hence is the projection of BC on 0-4.

BM OM

MN

Also

OM+MN=*ON.

Again we know that dot product of two vectors is equal to the product of the module of one with the resolved
part
i.

e.

projection of the other on the former.

.'.

(b+c)=a . (OC)fl
=(2
.
.

OAW (OM+AfJV
.

OM+a MjV= a
.

b-f a
.

c.
.

(b-c)=a [b-h(-c)]*a b+a (-c) =a b +[-( c)]=a b-a c. By repeated application of above, we obtain (a+b) . (a-b)=a . a-fb a-a . b -b .b = aa -b 2 V a.b=b.a.
Similarly

For geometrical interpretation of above see Q. 5 next


exercise.

(a-f b)*=:(a4-b)

.(a-hb)a

af b a+a b+b . b
.

In general (a

==a

+b+c + ...) p+a q+a


.

(p-fq+r +-.,.)
r
.

-f-b

p+b q+b*
.
.

-fc.p-fc

q-f

c.

rf ...

Multiplication of Vectors

103
of unit

Again
as

if

a and b be expressed
.'.

in

terms

vectors

and
then a
or
.

b^i+iaJ+baJk,

*V(V+V+V)
.

b^a^+a^+a
ab cos

V
Or

i'j*~k':l
fla

and

j-j

k-k i0.
.

(Prop. 8}

COS

n
Note.
the points
(al9

a& a z ) and

(4 lf

b& b^

are the coordinates of

A and B
if
I

respectively.
Z2 ,

of

l9 Again a and b, then

m ^ and
l9

m2

nz

be the direction cosines

/i

55

*
,

m!^=

^=5-^ and k

etc.

10,

.%

cos tf/1 /2+m1 in2 +n1 n2.


(Imp.).

[From

(1)]

n*
If

Components of a vector
is

(Delhi 50,

Lucknow

52)

any vector r

inclined at

an angle B

to the direction

A
of

a and a be a

unit vector in this direction then the resolved

A
part of r in the direction of
ar cos

a
a)

is r

cos

<l>

^ _._..*__
(r
.

(r m-- mta) -.a,


.

V aaa.
(Remember)

Again
of

if

a and
;

r,

r be resolved into two components in the plane one parallel to a and the other perpendicular to

'!

then these components are

-V-

a and

r- -A-

a.

You can

write

a2

for a*

104

Vector Analysis

Similarly if a vector r be resolved into components 2 since i j a=sk2s=sl parallel to unit vectors i, j and k, then
these components are
(r
.

i) i,

(r

j) j, (r

k)

(Remember)
any vector r
[

Alternative.

We

know

that

can be

expressed in terms of three non-coplanar vectors let us suppose that r=xi+jj+* ...... (1)

i, j,

and
1ft]

5 P.

Mulplying successively by i, j 2 . i=0 and i - j*-k*--l. i . j *j .

and

k and

noting that

kk
/.

(Property

8)

i=x, r

.j>,

k=<:.

Putting the values of x,y and z in

(1),

we

get

r be expressed in terms of any three there exists a relation non-coplanar vectors a, b and c, then
iflj,

Again

if

between them

i. e.

r=*a+jb+sc

............ (1)
b and
c
(

Multiplying the above relation scalarly by a,


successively,

we
r
.

get
. .
.

5 P. 15)

a=*a a+?b a+^c a ......... (2) r .b xa b+jb c+zc b ......... (8) ... ...('!) r c=#a c+jvb c+^c c ...
.
.
,

Now a
minating x,y,

a . b and c b etc. are scalars. Hence z between (1), (2), (3) and (4), we get
a,
.

eli-

0.

Examples
Ex,
i.

Prove

that

in

a tetrahedron if two pairs of opposite

Multiplication of Vectors
edges are perpendicular, the third pair are also perpendicular.
opposite edges is the

105
Also

show that sum of the squares on two


each pair.

same for

(Lucknow

49,

Agra

389 53,

Utkal 52)

Taking
the
points

as origin, let

position
A,

vectors

of

B and C

be a,

and c respectively.

We
AC
is

are

given
to

that

Fig.

Ho

perpendicular
to

and we are

prove that

AC 1 toZ);

DB and AB is perpendicular to C; DA is perpendicular to BC. b (c-a^O cr b cb a...(l) /.


.

V
zero.

dot product of
/.
.

two perpendicular vectors


(b-a)=0or c. b=*c
(1)
.

is

JBltoDC; Now a b=b


.

a.

...(2)

a; therefore from

and

(2),

we

get

a.b = b.c=c.a .......... (3)


From ahove we deduce
that

(b-c)

showing that

DA

is

perpendicular to

CB.

Hence proved,

Again
*/

square of a vector
.

is

module.

equal to square of its (Property 7)


is

=a2 +ba -fca -2a b b=b c=c a from


.

which

symmetrical

because

(3).

Ex.

2,

Prove that

sum of

the

squares

on the edges of any

tetrahedron
(i)

is

equal to four times the


edges.

sum of
sum of

the

squares on the join*

of the mid-points of opposite


(ii)

(Agra 51)
the squares

is

equal

to four times the

of the

lines

joining the vertices to the centroids

of opposite faces.

106

Vector Analysis

Ex.

3.

Prove

that

in

right-angled

triangle

ABC,

AB* + AC***BC*>

being a right angle,

Taking A as origin and the position vectors of as b and c, we have b . c=0, V Z. 4=ir/2 etc.
Ex.
4.

and

Prove

that

the

points

2i - j

+k,

- 3j - 5k,
52)

3i~4j-4k
Find

are the vertices of a right-angled triangle.

(Lucknow
also the other two angles of the triangle.

of the squares

Either calculate modules of each and prove that sum on the two is equal to square on third, or if

the given points be A, B and C, then find A3, BC, prove that dot product of any two is zero.

-*

CA

and

Again a
(b)
vectort

b=a6

1 cos 6 etc. cos*" 1 VrA')> cos- V(if).

(Use Prop. 8)

7/r=a+/b
that
.

be the equation of a sf. line,


line

being a unit
to
it is

prove
(a

the

through

origin perpendicular
is

r=ss {a

b) b} and length of perpendicular


2 2 V{a -(a.b) }.

(Agra 4 1)
is

parallel to given line passes unit vector b. are to find the equation to the line

The

through Pa and

We

OM

which passes through origin and is perpendicular to it. Note : In the figure take OM perpendicular to MP. AfP= projection of OP a in fr

p^

the

direction
8.
.

of

unit

vector

ba cos
/.

But a

b0

cos

6.

vector
.

MP in

the direction of

unit vector

is

of

module a

b.

b)

Multiplication of Vectors

307

-^

.->

Now
or

MP=OP- OM

OM0?-MP=a~ (a
Hence
the equation to

b) b.
is

OM
2

i.

0.

a line through origin

r^s
Also
;.

(a -(a. bib}.
.

OM*=OP -MP 2

OM=V{*
in
is

-(a.b)

2 }.

Ex.

5.

/V000 //wf
diagonals
the

any parallelogram the


twice the

sum

of the

squires on the
adjacent
sides;

sum of
of

the

sauares on two
is four

difference

of squares on
either

the

diagonals

times the rectangle contained by

these sides

of

the other upon it;


is

and

the difference

of

the squares on

and projection two adjacent

sides

equal

to

rectangle
it.

contained

by either

diagonal and the

projection of the other upon


*
>

Let 4#=b, AC~c, so that

-of

diagonal

^4Z)=b + c and

BC=*AC
module
(b)

AB*=sc

b.

Remember

that

c=bc cos

0,

i t e.

multiplied

by the projection c on

b, etc. etc,
is

Prove that a

parallelogram

whose diagonals are equal

rectangle.

Refer Q.
or
5.

or

AD**BC\
i.e

:.

AD*~BC*
-*

(b+c^c-b)2

4b
Ex*
6.

c~0,

AB
line

AC and
is equally

hence a rectangle.

If a straight

inclined to three coplanat

straight lines, prove that

it is

perpendicular to their plane.


let

Taking
coplanar lines

04=a, OB=b, OC=c,

--as origin,

OA,

OB

and

OC

be the three

103

Vector Analysis

Again
let

OZ)=d and we
~

are given that

OD

is

equally

inclined to all the above lines.

.
;

COS v
cos 6

d
sss

b d
.

c
ss
-

d
;~

aa
cos

~f~j

bd

cd

cos

But

-^, ^ ^
i
c

Hence the above

will

hold good only


is

if

cos
all

0=0
it is

i.e.

0=90 which means


lines

that

OD

perpendicular to

the three
per-

OA, OB,

OC which

in other

words means that

pendicular to their plane. Ex. 7. IfPbs the middle point of the side

BC

of a triangle

ABC,pwve

that

AB*+AC*=2

(AP*+BP*).

Choose P as
origin and

so
that

JM = a and P/?=b,

Square of a vector=square of its module. (Property 7) Ex. 8. Prove that in any triangle ABC,
3
where P,

(AB*+BC*+CA*)=

-9
Q,,

are the middle points of the sides

BC,

CA

and

AB

respectively

of

the triangle

and

is the centroid.

Use AB***AB*.
Ex.
9.

(Property

7)

In a qurdrilateral

ABCD, prove
the

that

where

and Q, are

the

middle points of

diagonals

AC

and

BD

respectively.

Multiplication of Vectors

109

Ex. 10.

Prove that the

middle point of the hypotenuse of a

right-angled triangle is equidistant

from

its vertices.

(Pb. 60)

Choose P the middle point of BC as origin and let the position vectors of
A,

be a and b; so that of
/.

is

-b.

PB=PC. We have

to prove

that

PA=PB=PC

or

or or

(b-a) +(-b-.a) =(-2b)

2b2 +2a 2 =4b2


Ex. n.
(a)

or

b2
in

a2
any

or

PB a ^P^ 2 PC 2

Prove that

triangle the perpendiculars

from

the vertices upon the opposite sides are concurrent.

(Agra 42,
Also prove that
the right bisectors

47;

Utkal

53;

Luck. 54)
(Luck. 49)

of the sides are concurrent.

Let the point

of inter-

section of altitudes BQ.

and

CR meet

at

and taking

this point as origin let the

position vectors of vertices

A,

and

be

a,

b, c.

Let

AO that AP

produced meet is perp, to BC.

BC

at P.

Then we should prove

4C*c-aand

110

Vector Analysis

BQ is
CR
.'.

perp. to AC]
perp. to

:. ;.

-fib

(c-a)=0;

.'.

b=*b

c,

is

AB\

-vc
or

(b-a)=0;

.'.

b.c=c.a.

a.b=b.c=c.a
is

-Aa.(c-b)-0,

i.e.

AP

perp. to BC.
right bisector of sides
let

Let the
Taking
vertices be a,

BC and CA meet

at 0.

this point as origin

b and
>,

c,

the position vectors of the so that the position vectors of the


----,

middle points

E,

F are

~-

Now OD

is

perpendicular to BC.

*+5.(c-b)-a
Similarly

b2 -c2
;.
is

OE is

perp. to CA;

c2

a2

Noyy we have to prove that


will

OF
i.e.

also perp. to
is

AB

which

be true

if

^-

ib-a)=0,

b a =a 2 which

true.

(b)
is

Prove that the median


to the base.

to the base

of an

isosceles triangle

perpendicular

Ex. 12. (a)


at right angles.

Prove that

the

diagonals of a rhombus intersect

(Luck. 50)
as origin, let the

Taking
be

of position vectors

b and d, we have b*=*d2

and since
.

D ABAD,
and

Also

Multiplication of Vectors

111
.

sd a -b 2 which

is

zero as

ba =d2

Hence AC
(b)
angled

is

perpendicular to BD.
is

// a

'point
its

equidistant from
to

triangle,

join

the

the vertex of a rightmid. point of the hypotenuse is

perpendicular to the plane of the triangle.

(Agra

52, 54)

Let
position

a, b,

c be the
of
tri-

c^

vectors

the

right-angled

angle.
2

or

(b-a) =(c-a) 2

a . b=a . c-fb . c-c* (1) Let p be the position vector of any point P which is equidistant from the vertices.
or
/.
.'.

(p-a) =(p-b) =(p-c)


2
2
,

2
.

2p

(a-b)=a -b 2p . (b-c)=b2 -c2


2 2p.(c-a)=c -aa

and
or
or

(2)

DP is

perpendicular to BA.

We
for

should also prove that


I

DP is
J
.

perpendicular to

AC

which we must have

^ ^

L (c-a) equal to
(1)

zero.

Now
or

p.(c-a)-J
C

(a.

c+b c-a2 -b.a)


from
and
(2).

-^----!

(c

-a2)=0

is

perpendicular to AB as well as perpendicular to the plane ABC.


is

DP

AC

and hence

it

Ex* 13*
an

Prove that in a parallelepiped, the

sum of (he squares

the diagonals is equal to

sum of the

squares on the edges.

112

Vector Analysis

The
are

four diagonals

OP,

CD, AE, BF,

and

clearly,

(a+b+c) 2 +(a+b-c) 2

+(b+c-a)a +(c+a-b)2

= 4 (a +b +c
a

a
)

V
Ex.
a cube
;

a2
14.

04 2 =04 2

(Property
a, ft y, S
2

7)

line

makes angles
2

prove that

cos

that the angle between

OL+COS* fi+cos y two diagonals of a cube

cos 2
is

with the diagonals of 8==|. Also prove


cos" 1 J.

->

and

Refer figure Q. 13 and


that the diagonals are

let

OA*=i,

OB^j

OC=k,

so

Let OL be any

line

OP. 01= OP
(i+j+k)
or

OLcosa,

V (*
cos
a.

+J>

T*

COS a

(Property 8)

Similarly cos

cos

cos 8=
/.

cos2

a+cos /3+cos y+cos

Multiplication of Vectors

113

Ex. 15.
any point

Prove that
the
its

sum of the

squares of the distances of


triangle

from

angular points of a
distances from
sides.

exceeds the

sum

of the squares of

the mid.

points of its sides by

sum of the squares of half the


Ex.
1 6.

(Agra 48)

If

be the

circum centre,
that 0, G,

the centroid

and

H the
that

ortho-centre of a triangle, prove

H are
"

collinear

and

divides

OH in the ratio
ABC

1:2.
/J

Take
the triangle

the circum-centre of
as the origin so

G
Fig. 67

that the points A, B,

are a,
2

b and
,

c.
2

Also
2

or

O^^OB^OC

a ^=b

OA^GB^OC, =c2 ...... (1)


Q
e~)

Again position vector of centroid

is

---.

From

(1),

a2
.

b2

or
v

b 2 =c 2

or

c2 =a 2

(a+b)
or

(a-b)~ b+c) . (b-c)(c+a)


.

(c-a)=0

[(a+b+c)-c]

(a-b)-O, [(a+b + c)-a]

[b^c]0
[c-al0.
is

and

[(a+b+c)-b]

Now

if

M
.

be the point

then above relations

whose position vector show that


.

i. e.

CM B40, AM CB=0, BM AC=Q CM J. BA, AM J, CB and BM AC.


.

to be

Hence H.

is

ortho-centre of the triangle

which

is

given

0,

__>~ QH-QG**WG
divides

G and

H are collinear.
_*
or

_
2.

C?H20C.
:

OH

in the ratio 1

114 Ex. 17.


in
the

Vector Analysis

7/P

be any point
triangle

side

AB

of a

ABC,
A

such that
.

AP~p

PB,

then prove that

A.
..(1)

..(2)

Multiply
A
.

(1)
.

by A and

(2)

by
.

/z

and add
.

2 <M+f* CB

(A+ft)

CP 2 +A

BP*

Now
or

PB

or

.
/LL

PB-A

p, .

PB+A PA^Q.
.

Hence

etc.

Ex. 18. Prove by


any triangle

vectors that in

ABC,
cos

a^b
and

C+c

cos

a*~b*+c*-2bccos A.

(Agra 42)
Let
modules
a,

C,
6

C^ and
c

4jB

have
*

--and

respectively.

Also

Squaring,

Multiplication of Vectors

115

Squaring,

we

get

cos

(IT

-A)

cos A.

Again
or or or
<2

J5C= -

J5C

BC*=*-CA .BC-AB.BC
2

a cos (T

c)

a cos

(IT

B)

a=6
Ex.
19.
:

cos

C+c cos B
of

Prove by vector method the following formula


cos (<x~ft)^cos

plane trigonometry

cos

ft+sin a sin

ft.

Let there be
vectors i and j

two

unit

along OX and OT, two perpendicular lines


in
If

the

plane of the

paper.

OP and OQ, be any two lines


the

in

same plane
,

making
"O

angles a and ft with OX res~ .. ., , n pectively, then Z.POQ,= a~ ft.

&

Fig No. 70 y

-*.

represent unit vectors along OP and OQ, respectively so that their dot product is cosine of the angle between their directions,

Again

let

OA and OB

i.e.

OA. OB
But OA

l.l.cos(a-0)

cos(a-0)
i

(1)

inclined at an angle a to the direction of


.

can

be expressed as 0-4=cos a

i+sin a
i+sin

and

similarly
/.
.

OB=cos
a

ft .

ft . j.

OA OB=[cos

i+sin a

j]

[cos
ft

ft .

i+sin

j]
(2)

cos a cos /3+sin a sin

116

Vector Analysis

V
Hence from

i~jl
(1)

or

i.jj.i0.
get

and

(2),

we

the

cos (a-/3)s=cos a cos /3+sin a sin j9. Ex. 20. Particles of masses ml} wa, w3 ...... are placed at Prove that points A,B,C.. .respectively and G is their C. M.
,

for any point P,

+(2? mj)
It

PG 2

will

be convenient

if

we choose

the centre of
. . .

mass

G as origin and the position vectors of A, B, C. as a, b, c. .respectively and that of P be p.


. .

be taken

.".

AP=GP-GA=*p-*, BF^p-b
c.

etc.

Also since the

m.

is

at the origin,

we have 2
...(1)

Also
2

2 -2p a+a
.

aswit (p

- 2p . a+a 2)+wa

(p

- 2p

b+b 2)+ ....


2

+(m1a
=(2:

+m b
2

+....)

2 2 m^ PG 2 +m1 ^G +m2 BG + .... V 2?m1a0by(2)

Ex. 21.
and

If there be four non-coplanar

straight lines
a.,

and unit

vectors parallel to their directions be denoted by

b, c and
lines

res-

pectively

cos (a, b) stands for the angle

between the

which

are parallel to

a and b
1

then prove that


(a, b)

cos
a)
a)

cos

(a, c)

cos cos cos

(a, d)

cos

(b,

cos
b}

(b, c)

(i,

d)

cos cos

(c,

cos cos

(c,

1
cos
(rf,

(c,

f)

(d, a)

(d, b)

c)

Multiplication of Vectors

117

We know

that there exists a linear relation


let it

between

any four non-coplanar vectors and

be

xa-kyb+zc+wd^Q
Multiplying
(1)

......... (1)
d
in succession

scalarly

by
PS

a, b,

c and

and

noting
. .

that

a.al
we
(a,

is

a unit

vector

and

b=l
x

1
1

cos
.

(a, b),

+j>

cos

get on multiplying (1) scalarly by a .(2) b)+z cos (a, c)+w cos (fl, d)=sO
. .

We
we

can write down similar

relations

as above

on

multiplication of (1) scalarly ting x, y, z and w between the


get the required result.

by

b,

c and d.

On

elimina-

four relations thus obtained,

Ex. 22.

and

The position vectors of the foci of an ellipse are b, and the length of the major axis is 2a. Prove that the
is

equation of the ellipse

We
major
or

know

that

in

the

focal distances of any point


axis.

on

case of ellipse the sum, of the it is equal to the length of


it,

Hence
|

if

r be any point on
| |

then

r+b + r-b =2a 2 2 (r+b) [2a- r-b


|

[Square of a vector
or
or

is

square of
-fr
2

its

module]

+2r b+b =4a -4a (r-b)


.

+b -2r .b
2
).

[a

a 4 -2a*r

or

r b]=a (r b Square again. 2 b+(r . b) 2 a2 (r2 -2r . b+b a 4 -a 2 (r2 +b2)+(r . b) 2 =0.
2
.
|
|

Ex* 23.
and

Prove that the hyperbola whose foci


is
|

we

points

bx

b2

and whose transverse axis


|

2a

is

given by
|

r _ bl

_
|

r _ ba

-2a.

Ex. 24*

Prove that

b
ab

118
3.

Vector Analysis

Vector Product*

Definition.
51, 57, 585

(Agra 40,
The
a
vector (or cross) product of
respectively
is

Pb. 60; Raj. 57)

two

vectors

a and b
is

of moduli
ab sin Q,

and b
being the

a vector

whose

module
vectors

angle
is

between the

directions
to

of

and

and

whose direction

perpendicular

being regarded positive, if the rotation


clockwise.

a and b, this direction from a /o b appears counterboth

In case the rotation

is

from

to a,

then

it

will be in

clockwise direction

and hence
sin

negative.

Thus
where n

axbfli
is

.n

unit
to

vector

perpendicular

both

and

b.

Q
f

^
Fig.No.il

1.

Vector product is not commutative.

have proved that dot product of two vectors is commutative, i. e. a . b=b a, but this does not hold good in the case of cross product of two vectors a and b, i. e. axb is not equal to bxa because the rotation which
carries

We

to b,

i. e.

counter-clockwise is opposite to that

which carries b to a, i. e. clockwise. Of course the magnitudes of the two are same, but their sence is opposite.

Hence a x b
Therefore
five,
i.

- (b x a).
the
vector

we

conclude that

product

is not

commuta*

e.

the factors in

a vector product can only be interchanged tf


is reversed.

and only if the sign of the product


2.

Vector Product

is associative*

Just as dot product of two vectors is associative, their cross product is also associative, i, e. if either factor a or b, iiithe cross product is multiplied by a scalar m, then
their product
is

al$b multiplied

by that

scalar,

Multiplication of Vectors
i.e.

119
n)

(ma)

X b=a X (mb)=ro

(a

X b)=m

(ab sin

Thus the vector product of two vectors


3.

is associative.

i.

e.

Cross product of two parallel vectors. We know that aXb=0i sin . n. In case a and b are parallel, then angle between them 6 should be either or 180 and in either case sin 0=0.
.'.

axb=0.

From here

it

also follows that

aXa=0.

Thus we
vectors is zero.

conclude that

vector product

of two parallel or equal

0=0

Conversely, if axb=0, /. ab sin 6 n=0, then either or i=0, or sin 0=0, i.e. either of the vectors is a zero or null vector, and in case neither of the vectors is a zero vector, then sin 6 being zero shows that they are parallel.
Thus
if cross product

of two vectors neither of which

is

a zero

vector vanishes, then these vectors are parallel.

4.

Cross product of two perpendicular vectors.


i.e.

In case the vectors are perpendicular,


sin

0=90, then

5=1;
Thus

.'.

aXb=fl.n.

of two perpendicular vectors is a vector whose module is equal to the product of the moduli of the given vectors and whose direction is such that a, b and nform a rightthe cross product

handed system of mutually perpendicular

vectors.

Cross product of unit vectors. 5. In case a and b are unit vectors i.e. their moduli are each unity, then aXb=sin 6 . n.
Thus
module
is

the

cross product
the sine

of two unit
of
the

vectors

is

vector

whose

equal to

angle

between the

directions

of

the given vectors.

6.

Unit vectors
it is

i, j,

k.

(Very important relations)

From above
whereas and

easy to deduce that

iXi=jXjkxk=0

i*sjk2 l ixjk~jxi,

120

Vector Analysis

kxi=j-ixk,
whereas
7.
If

j=j . i~0, j k=k . j=0, k . i=i We know that aXb=ai sin 0.n.
i
.
.

k=0.

b' be

b'4 sin 0.
b so
that

component Also a and b' are


/.

the

of

perpendicular to a then in the same plane as a and

remains the same.

aXb'=fl6

sin 9

n=axb.

Thus we
remains
nent to the other.
8.

conclude that

the
is

vector product

unchanged if one J actor

of two vectors replaced by its normal compo-

given vector a. Let a unit vector


so that

The component of a vector r perpendicular to a (Delhi 59, Lucknow 52)


in the

direction

of

a be denoted by

a=0i and in a direction perpendicular to it be j. Let the unit vector k be perpendicular to the plane i and j. Now if r be any vector in the i j plane inclined at an angle
B to

a then
j
is

its

a,

i.e.

components in the direction perpendicular to ...... (1) rsinfl.j

Now

aXr=0r

sin 8

.k

...... (2)
to i j plane in

where k is a unit vector perpendicular both a and r lie. Now k x i=j.


:.

which

ar sin 6

j**ar sin

c sin
.

kXi=(aXr)xi by (axr)xi sa (aXr)Xfli ~


.

in

(2).

'

"i or

or

component

of

r JL to a

(aXr)Xa

- ax(axr)
.

Note

We
of

have

proved

2'11

P. 103 that
is

the

component

in

a direction perpendicular to a

a
a*
it is

a8

We

will

show

in the

following pages that

same as

found above.

Multiplication of Vectors
9,

121

Distributive Law.

aX(b-fc)=aXb-faXc.
See
5 Cor. 5 P. 138 for proof.
in general

Thus

(a+b+c-t-

. .

.)

X (p+q+ r +

. . .

.)

sraXp+aXq+aXr+....

+bxp+bxq+bxr+....

+cxp+cxq+cxr+ .....
10.

Expression of vector product in terms of


in

unit vectors.
If

a and b be expressed

terms of unit vectors as

and

b=i1i+
.'.

a X b=(a1i+a 2 j+

sk)

x fti+M+W(1)

+(A-A)k
V ixi=jxj=kxk=0
The above may
andixj=-k=--jxietc.
be expressed in determinant form as
j

axb=

bl

b%

bs
(I)

[Remember]

and remembering that square of a vector is the dot product of a vector by itself and also 2 2 l and i . . . i0, that ia *j *k
Squaring both sides of

jj

kk

sin 0*

or

Again if /lf mx of a and b, then

and

/a ,

^,

be the direction cosines

122

Vector Analysis

and
.'.

^sin 2 6

Tv+v5 u< T *1
'

" i= *''

In case the two vectors

are parallel, then


as
-y.

aXb=0

and

hence from determinant


ai
*.P,,
i

(2),

we must have two rows


b3

identical,

-as

a*
,

bl

b%

Again
sin 2 ^

= l-cos2

and

2
1

+V+i

2=ss

l='2
2

+^2 2 +
we

2
2

and putting the value of cos


the well

from 10 of

P. 104,

get

known Lagrange^s

identity

11.

Incase
if

aXb=axc
then

...... (1)

Now

b=c+#a,

axb=ax(c+a)=*axc V axa=0,
From
it

...... (-2)

(1)

and

(2),

we conclude

that

if

axb=axc,
b may
differ

then

does not

mean
2

that
is

b=c only
parallel to
.

but that

from

c by a vector which
12.

a as b=c+A'a.
a unit vector.

(axb)

=a2 b2 -(a.b)2
.

axb=0&

sin

n,

where

is

Squaring both

sides,

we
0,

get

(axb)

=<2 2 6 2 sin 2
2 2

V
2

n2
2 a

(1-cos 0)=a

-02* 2

cos 2 $

=a b
2

-(a.b)

a
,

V
13*

fl

==a and

2
fe

=b

and a
ft

b=ai

cos
/3,

0.

sin (oc-/3)*=sin

a cos

-cos a

sin

sin

(a+j8)sin a cos /?+cos a sin

jS.

Multiplication of Vectors

123

Proceeding as in
P. 115,

Q.

19

04=cos

a i-fsin a

j,

If

OQ' makes an angle


the
direction
of
i

\\ith

in

IT
FigNo.72

opposite direction, then

j,

[sin

/?)=

sin

0#xO^=(cos
or
1

]8

i+sin
J. to

j8 jS

1
is

sin(a

jS)

n==(cos

j)X(cos a i+sin a j) sin a ix j+sin ft cos a jxi),

where n

a unit vector

ij plane.

Now ixi=jXj=0
.*.

and

ixj=n
]8

and

jxi=-n
]8)

sin (a

]8)

n*=(sin a cos

cos a sin

or

sin (a-/3)=(sin

a cos j8-cos a sin

0).

Similarly

)8

and proceeding

OB'xO^=(cos ]8 i-sin as above, we have


-f cos

j)

(cos

ai+sin a j)

sin (a+/3)=sin a cos

a sin

/?.

Exercise
Ex.
i.

Two
:

vectors

a and b
of the

are expressed

in terms of unit

vectors as follows

a=3i+j+2k, b=2i - 2j+4k.


vectors.

W%o/

unit vector perpendicular to each

Also

determine the

sine of the angle between the given vectors ?

(Lucknow
121).

48)

aXb

=8tf-j-k)(fromlOP.

312
2-24

124

Vector Analysis

Nowaxb
or

a and b and hence a

represents a vector perpendicular to both unit vector in this direction is obtained


its

by dividing 8 (i-j-k) by

module,

i.

?.

V(8

+82 +8 a

8V 3.
Hence
the required unit vector
is

(i-j-k) ~
Again (a X b)*=8 (i-j -k)*=8
(ab sin
.

. 3.

n)

=88 .

but

a= V(9+l+4)= V(14)
.

and *=V(4+4+16)=V(24).
.'.

a*b* sin 2 6

1=8*
'
;

8in

"*-]j7i-*
unit

sin *

=
77'
to each

Ex,

2.

Prove that the

vector perpendicular

of

" 3l
the vectors

2i-j-hk am* 3i+4j-k

i*

and

the

sine of the angle between them is

{ufij(Utkal 53) Taking a and b from Ex. 1 or 2, prove that is perpendicular to both a and b. represents a vector which Hint. You should show that dot product of both a and
Ex*
3.

^
of

(155\

axb

with

aXb
4.

is

zero.
the

Ex.
point

Find

equation
tine

the

straight

line

through the

and

equally inclined to

vectors a, b,

c in

the

form

(Imp.)

<=>+

Let the equation


parallel to

of th

line

through the point


.

d be

unit vector

A k

A
equation
is

so that

its

Since the required line


b,
c,

therefore they are equally inclined to k.

.'.

AAA
is

Multiplication of Vectors

125

equally inclined to vectors a,

k=0
/.

1 cos

6,

a
.

A k

b k=i b A
.

1 cos

0,

k^c.

1 cos 6.

c
.

A
k=cos

ib

k=s=
c

or

Above shows tions of a, b and c are


then

A A A A A A a,k:=:b.kc.k=:cos0 A that the resolved parts of k


equal.

(2)

in

the direcA,

In case their module be

A k can

be written as

A A, A

,/a

.
. .

.,.

,(o)

Substituting the value of

A k from

(3)

in

(1),

we

get

the

required equation,
as

=b+/A+-+.
(2),

Replae/Abyj.

Second form.
Again from

we

get (a-b)

AAA k=0, AAA (b-c).k0.


.

Now we know

that dot product of

two vectors

is

zero
that

provided they are perpendicular.

Hence we conclude

A
k k
is

A A
A A
is

A A
and therefore
a vector per-

perpendicular to both (a-b) and (b-c)

A
b.

parallel to

(a-b)x(b-c) because
a and

axb

is

pendicular to both

126

Vector Analysis

.'.

A k=f (axb-axc -bXb+bXc). A A A A A A

AAAAAAAA
and

Put
.\

bxb=0
A

-aXccXa

A A A A A A k=f (axb+bxc+cxa) /axb bxc cXa + be + 'ca V~S~


(bxc)+*(cxa)+c(axb

J
(b)

Prove that in a regular tetrahedron the perpendiculars from


opposite faces meet at their centroids.

the vertices to the

Let the vector


sectors of

the

be chosen as origin and the position other vertices A, B and C be a, b and c.


is

Since the tetrahedron

on the face
i. e.

9C

vectors

ABC will be equally as such a, b and c


;

regular, therefore perpendicular from inclined to OA, OB and


its

equation will be

^
is

r c

But since the tetrahedron


.'.

regular,

OA=OB=OC
:.

or

a**b=*c.

r=^

(a+b+c)
is

......... (1)
......... (2)
on comparing the

Equation to the plane

ABC
of

r(l-*-Oa + fb+fc
For the intersection
:oefficients,
(1)

and

(2),

we have
A
a
, ssl-j-/

A
a
A-

A =BJ a
or
-

=*t

A
*

s=l
a

A -a

A a

=$=*.
where
the

Hence the

position vector of the

point

Multiplication of Vectors

127

perpendicular from

meet

ABC
3

is

a+b+c
which
5.
sides
is

the centroid of face

ABC.
establish
the

By

vector

method
triangle,

relation

between

the

sin

and angles of a

sin

sin

i.

e.

Let BC, CA,


a,

AB be

vectors

b and
.'.

c respectively*

a-fb+c=o.
that cross

Now we know

product of two vectors one of which is zero and cross product of two
zero.

equal vectors

is

fig No 73,

or

or

axb + axc=0, aXb=cXa V -aXc=cXa. or bxa+bxc=0 Similarly, bx(a+b+c) bxc=axb. axb = bXc = cXa
.*.

ax(a-fb-fc)=0

or

.'.

or
or

ah sin

(TT

C)=fo

sin

(TT

A)^ca

sin

(TT

B)

ah sin

C=fo

sin A*=ca sin B.


abc,

Dividing throughout by
sin

we

get

^ sin B "" sin C b c


->
of a constant vector

(b)

//

the

vector product

with

variable vector
that locus

OB
is

in a fixed plane

AOB

be a constant

vector,

show

ofB

a straight

line parallel to

OA.

(Luck. B. Sc f 46, 49, 55)

128

Vector Analysis

OAssti say and OB=*r.

Now aXr=constant

given.

=axb

where
:.

OP=b
is

say.

ax(r-b)=0.
zero, therefore

Since the cross product


llel

r-b

is

para-

to a.
.'.

b=/a
line

or

r=b+/a
P
parallel
to

Above represents a Hence the locus of B is as


Ex.
6.

through

OA.

given.
area of a
inclined

Find

the

vector

triangle

OAB

where

OA~a 0B=b
3

and they are

at

the vector area

of a triangle whose

vertices

and hence find are the points a, b and c.

an angle

(Utkal 50)

We know that axb=06


Now
area of
.'.

sin
.

n.
sin

&OAB=$OA OB
axb=0ft
of
sin

6=\ab

sin

B.

ns=2A n

Hence vector area

A0-4J3=|axb.
is

Also the vector area of the parallelogram two adjacent


sides of

which are a and


if

a X b.

Now
c,

the position vectors of A,

B and C

be

a,

b and

then fiC=sc-b and

&4=a-b.

Therefore vector area of

&ABG

is

Uc-b)X(a-b)

=J (cXa-cXb-bXa+bxb) = i (a X b+ b X c+ c X a) [Remember] V bxb0and -bxaaxb.

Multiplication of Vectors

129
collinear,

Note.
clearly

In case the

three points

are

then

Ex.
tides

aXb-f bXc+cXa=0. In a triangle ABC 7. BC, CA and AB respectively


:

points

D,

E F
9
:

are taken on th*

such that
:

BD DC^CE EA^AF FB=n


:

/,

prove that

A DEF- -*~-? t
(n+2)

ABC.

Taking
let

A
be

as

origin

the

position

vectors

of

B and C
position

b and c
Therefore
of
A

respectively.

the

vectors

F,

and

E are respectively

--

n+l

Vector area of &ABC*=1> (bxc).


>

[by ^.

G.J

Vector area of

&EFD*=\ (EFxED)
(b-.c)x[nc+(l-w) b]

-*

(n^lf

bxb=cxc0

130

Vector Analysis

Note
1,

In case

we

get
8.

Z), E, F are the mid. &EFD=&DEF=*l&ABC.

points, then putting

Ex.

The

infernal bisectors of the angles of a triangle

ABC

meet the opposite sides in

D, E, F; show

that

a, b, c being the lengths

of BC,

CA

and

AB

respectively.

Refer

tig.

Ex.

7.

We

know

that internal bisector

of

an angle divides the


angle
i.e.

opposite side in the ratio of the arms of the divides AB in the ratio CA CB, i.e. b a etc.
: :

/.

position vectors of F,

and
'

E are

respectively

^ib+a.O
b+a
JTt,
'

cc+bb ^a.O+tc a+c c+b


cc
(a4~c)

'

.\

^b Tt. Tt, EF=AF-AE=r~

bb

(b-\-a)

cc

c+b
1

(c+a]

"
Now
multiply
(fl-f i).

+ a)

V+*)
1st

']

bracket above and below by

and 2nd by

X{c (a-*) (a+i)

c-f-A (c+a)

(a+A)

b}].

Now

keeping

in

view

that

bxb=cxc=0

and

-cxb=bxc, we

get

Multiplication of Vectors

131

(b+c)

(c+a) (a+b)]

(b+c) (c+a]

"-*+"+*

Ex.

9.

Prove that (a - bj

X (a + bj
47,

2a X b

0/2 rf

interpret

it.

(Agra

Lucknow

56,

Andhra

36}

We know that aXa=bxb=0


and

-bxa=aXb.
/.

(a-b)x(a + b)

=2aXb.
Interpretation
:

Ftg.No.75

In the parallelogram ^BCZ)


/.
.*.

>

let

<40a

and

OD=b.

0^=-b and
(a

hence

AB=a-b

and

and it whose adjacent


gram. Ex. 10.
by the vectors

b)X(a+b) represents the area of parallelogram being 2axb, i. e. twice the area of the parallelogram
sides are semi-diagonals of the
first

parallelo-

Prove that the

area of the

parallelogram

determined

a=i+2j+3k
Given
the

and

b= -3i-2j-Hk is
r=0i-J-j where
to

6^/5.
i,

Ex.
vectors

n.

vector

are

unit

in the

direction

of the axes, find an expression for the vector


the given vector

of the
origin.

same length perpendicular


of the given

through
2
)

the

(Lucknow
vector
is

46)
it

The module
lies

V( a2 +*

an <*

plane and therefore ixj represents a vector perpendicular to i and j plane and hence perpendicular
in
i,

132
to the given vector
is

Vector Analysis

which

lies in this

plane.

Since

its

length
it

be the same as that of the given vector, hence should be V(02 +* a )iXj.
to

Ex. 12.

Prove that the area of the triangle formed by joining

the middle point of one


the extremities

of

the

non-parallel sides of a trapezium to


is

of the opposite side

half that of the trapezium.

(Agra
Let P the middle point of the
oblique side EC be joined to the extremities of the other side OA,

45, 57)

then

we have

to prove that

Let OA be a and AB be b
so that position vector of

Fig.

No IB

i.

e.

OJ5=a+b.
,

Now OC
The

being parallel to
vectors of

AB
P

/.

OC=kAB=kb.
are

position

B and C

a-fb and kb
point of

therefore the position vector of


J

the middle

BC

is

(a+b+ftb).

(a-J-b+fcb)
...(1)

trapezium OABC=* &OAB+ 2 A0^4Ptrapezium

&OBC**
(1).

axb.

O^BC. from

Multiplication of Vectors

133

Ex.

13.

Q, an d -R are the middle points of the sides

AC

and

AB

of a

triangle

ABC

respectively.

CP

is

drawn

parallel to

AB
and

and meets

BQ

produced in P.

Prove that

&RQP &RCQ

each equal to one-fourth of

&ABC.

Taking

as

origin let the points

B and C

be

b and

respectively so that R and Q, are Ab and

Jc repectively.

A/

/?

*\BCxBA**\ (c-b)X(-b)

Equation to

CP
->

is

r^c+Jb

as

it i?

parallel to
it

AB.

Q,

i.

Equation e. c/2 and

to

BP
i. e.

is

r~b+$
at

(c/2-b) as

passes through

b.

These

lines

intersect

P and hence on comparing,


/.

we

get

1^=5/2 and

t=]

^=2and/s
P is c-b.

1.

Therefore the position vector of

(Jc-ib)x(c-b-Jb)

Similarly

etc.

Ex. 14.
Prove that
its

AC

and

BD
-

are

two diagonals of a quadrilateral.

area is

134

Vector Analysis

If

be the intersection of

the diagonals, then quadrilateral


is

&PAB+ AP#C+ APCD-f &PDA


A
FigNo.78

\ {(

- PB x PA + PD X PA) + (PB X PC - PD X PC )}

__>__ ___.--

~ + PDXP.4)
>

C=^ ACxBD.
15.
^4

//^

F
and

is

drawn

parallel to base

EC

of a triangle

ABC
Cft

meeting
be

AC

A3

drawn

parallel to

E and F respectively. If BR and AC, AB respectively to meet EF in R and


in

that respectively, then prove

&ARB= A^C(9.

(Agra 59)

4.

Product of three vectors.

Scalar triple product and vector triple product. (Agra 37, 40, Raj. 56, 57) We have already seen that the dot product of two
vectors a and
of
i.

is

a mere sclaar vvhereas the cross


is

product

two vectors
e.

a and b

a vector

itself,

\>=ab cos 6
sin 9
.

(scalar)

and

axb=a
is

n, (vector)

where n

a unit vector perpendicular to the plane of

given vectors.

Now

since

axb

is

a vector,

we can

multiply

it

both

Multiplication of Vectors
?calarly

135
c.

and vectorially by another vector say


will be

The

former

called scalar triple product

and the

latter vector triple product. or c . (a x b) .*. (a x b) . c


of
is

is

scalar

triple

product

the vector a,

b and

c whereas

cX (axb)= -(aXb)Xc
.

vector triple product of the vectors a, b and c. \Vhat do (a Naturally the questions arises
:

b)

c and

c X (a

b) stand for
.

?
.

written as a

b is a scalar and as such (a b) c may be b c, it simply represents a vector in the direcThus the tion of c whose module is a b tunes that of c. of two vectors can occur as a coefficient of dot product
Since a
.
. .

a third vector.
Again as above a
therefore
5.
.

being a scalar and not a vector,

cX(a

b)

is

meaningless.

Geometrical interpretation of scalar triple product (axb) c. (Agra 59)


.

In

the

adjoining

paracoterarid

llelopiped

the

three

minous edges OA,

OB

OC
a,

represent

in

magnitude
vectors

and direction

the

b and c

respectively.

Therefore
sents

axb
OADB

reprcis
is

a vector

n whose magnitude
whose direction

the

area

of

the

parallelogram the plane of the face


the directions of
i. e.
.

perpendicular to

OADB. Now
i.

if

B be the angle

between
.

axb

e.

n and

that of c then (axb)

n c=area OADB
product
is

c^s 6 which

we know
value
i,

represents

the volume of the parallelepiped.


triple

The
is

of the scalar
a, b,

+ive when

acute

e.

c form a

right-handed system

of vectors.

136
In a similar

Vector Analysis

manner we can show

(cxa)
piped.

also represents the

that (bxc) a and volume of the above parallele-

Also

we know

that

b=*b
.

a and as such

(axb).

c=c (axb)

(bXc).a=a .(bxc) (cxa).b=b .(cxa)


and each equal to volume.

......... (1) ........ (2) ......... (3)


(1)

Now we

find that

(axb)

c=a

(bxc) from

and

(2).

We

find that
in

we
is

and cross

above scalar

have* interchanged the position of dot triple product, but the cyclic

order of the factors


Similarly,

maintained.

(cxa).b=c*(axb) and (bxc) a=b (cXa). Thus we conclude tbat in the scalar triple product the position of dot and cross can be interchanged at
.
.
'

pleasure provided vectors a, b and c,

we maintain

the cyclic order of the


(Pb. 60,

Agra

59)

Effect of changing the cyclic

order

Now we know
/.

that

(aXb)--(bxa).

F~(aXb) c=-(bXa).c =B-c. (bxa),


.

(bxc)

,a~(cXb).
.

-a.(cXb),

(cXa).b-(aXc) b
Thus we observe
of

fig Mo.

80

the vector a,

that by changing the cyclic order c or b, c, a or c, a, b to b, a, c or b,


sign
of

a, c,

or

c, b,

a the

the

scalar triple product

is

changed.

Multiplication of Vectors
Also

137
so on,
i.e.

we

see that

- fa X c)
order of a,

b*=*

-a

(c

X b) and
if,

the position

of dot and
cyclic

cross can be

changed at pleasure whether

you

maintain the

and c or not;

however , you

change the cyclic order, the sign should be changed.

Notation
is

The

scalar triple

product of three vectors

generally written as [abc].

Thus
of

[abc]=[bca]=[cab]= -[acb]= - [cba]= - [bac]


In each of the above different forms
scalar triple

product the position of dot and cross can be changed.

[ijk] = i.(jXk)=i.i = ], V jXk-i. Cor. i. Condition for three vectors to be coplanar. [abc]=a . (bxc) where a, b, c are three coplanar
.'.

vectors.

Now bxc

represents a vector v\hich

is

perpendi-

the plane containing b and c in which also lies the vector a and hence bxc is perpendicular to a. There. fore a (bxc)=0 being the dot product of two perpendicular
to

cular vectors

a and b X
If

c.

Thus
i.e.

[abc] =0

when

the three

vectors are coplanar.

Converse

[abc]=0,

(bXc)=0 showing

that

bxc is
in the

perpendicular to a.

the plane containing

b and

But bxc is perpendicular to c and h^nce a should also he


c should be coplanar.

plane of
2.

b and

c, i.e. a, b,

Cor.

Scalar triple product


.

when two of

the

vectors are equal.


perpendicular to the = plane containing a and c and therefore a.(axc) 0or otherwise also [aac]=a (axc)^(axa) . c because the posi-

[aac]=a

(aXc).

Now aXc

is

tion of dot

and cross can be changed.


/.

[aac>=(axa).

c0,

Cor*

3.

Scalar triple product

when two of

(aXa)s=0. the

vectors are parallel. Let a and b be parallel so that b=*A;a wheie k


scalar.

is

138
/.
.

Vector Analysis

[abc]a (bXc)=a
.'.

(A;aXc)=A:

[uc]-0.
3,

[by Cor.
the following
will be zero
:

2].

Hence from Cor.


The
scalar
triple

3,

2 and

we have

product

oj three vectors

when

they are coplanar or

two of t htm are equal or

parallel.

Cor.

4.

We

know

that

any vector can be expressed

in

terms of three non-coplanar vectors as

a =0jl +

<7

-f- fl 3

n,

c^l

c2

m + c^n
(btf

bXc

-f b 3

mX n+
*

-b

nX1

/.

[abc]=a

(bxc)

+( Vi

*i f 3)

n x * +-(*i^2 "" Vi) 1 X m] n *l) *i c a) - 2 c,)n (1 X m).

fc

Also other terms in the above product vanish as the


scalar triple
is

zero

when two

of the vectors are equal.

Again we know that


1
.

(mxn)=m

(nxl)=n . (lxm)=[lmn].

[abc] =

[Imn].

(Agra 38)
In case a, b, c be expressed in terms of unit vectors
k, then [ijk]l and hence
i,

j,

Multiplication of Vectors

139

[abc]=

Mi

bi

bz

bz
f3

cl

c2

Above

is

the well

known expression
,

for the

(Agra 40) volume of a

parallelepiped whose one vertex is at the origin and the other three at (%, 2 a 3 ) (b l9 6 2 6 3 ) and (c lt c 2 c3 ) [rectangular
,

coordinates],

Note
cal

In

case

then the two rows

in the

and as such

it

will

any two vectors are equal or parallel above determinant will be identibe zero and hence if two of the vec[abc]=0.
distributive
triple product,

tors a, b, c be either equal or parallel, then

Cor.

5.

To

deduce the

law of cross product of


i.

two vectors by the help of scalar

c.

aX(b+c)=aXb+aXc.
(Agra 51, Raj. 57)
Let r be any
butive,
vector; then since scalar product
is

distri-

we have
r
.

[aX(b-f c)-axb-axc]
.

=r
tion
of

[aX(b + c>]-r
that in

(axb)-r

(aXc)

(1)

Again we know

scalar .triple product the posi-

dot and cross can be changed without altering the value of the product. Hence we can w rite R. H. S. of (1),
r

as

(rXa)

(b+c)-(rxa)

b-(rxa)

which can be written as


(rxa) .[b+c-b-c]=(rxa).0=0 because scalar product is distributive. Hence we have for all values of r,
r
.

[aX(b+c)-aXb-aXc]=U.

)-aXb-aXc=0
or

The

other possibility of either r being zero

or

it

being

140

Vector Analysis
is

fact that r

perpendicular to ax(b-fc) axb aXc is any vector whatsoever.


6.

ruled out by the

Vector Triple Product.


(Alld.

A/-~-^

aX(bXc)=a M. Sc.
c
is

b-a .b

c.

Note
vector

1960, Pb. 60, Agra 32, 35, 59) a scalar and occurs as a coefficient of

b and

similarly,

being scalar occurs

as

coefficient of c.

xst

Method.

Consider the vector


.

triple

product

when

two of the vectors are equal, i> e. aX(aXb) = a . b a-a


\Vc have done that
if

b.

there be a vector b,
*

then

its

component along a given vector a


or
,

is

(Page 103)

b . a a a a
.

.
;

=a
its

a
is

and

in

a direction perpendicular to a

component
-

ax(axb)
-

a&

or

aX(aXb) a
.

(Page 120)

a
Multiplying by
scalar
.

aX(aXb)=b a a V b a=a
.

a .a a and transposing, we get a . a b=a b a a . a b,


.

b.

Rule
the

First

we

take

the dot product

of

the

vector outside

bracket with the extreme vector inside


coefficient

the bracket

and

it

becomes

the

of

the

remaining one.
bracket

Then we
with
the
the

take the dot product

of the
bracket

vector

outside the
it

nearer one inside the

and
is

becomes

the

coefficient

of

remaining one.

The

same rule

true

when

all the vectors are

unequal.

Again
or

(axb)Xa- -aX(aXb)-(a . b a-a .a (aXb)Xa=a. a b-a ba

b)

which obey the rule written above.

Multiplication of Vectors

141

All the vectors being unequal*

Let

sents a vector perpendicular to the plane containing


/.

n but n
of

which repreb andc. P aX(bXc) aXn is perpendicular to both a and being perpendicular to the plane containing b and c

P aX(bxc)=aXn

where

n=bxc

and therefore

is

perpendicular to a and
is

it

lies in the

plane
c.
(1)

b and
Let

c.

Hence P

expressible in terms of

b and

But we have
/.

P=xb+yc P a=xa b-f^a already stated that P


.
;

(2)

is

perpendicular to a.
k

P a=0
.

/.

from

(2),

^ ^~if=
c
in
(1),

say-

Putting the values of x and

y
.

we
.

get
c)
(3)

P=aX(bXc)=(a cb-a b
Now we
have
to find the value of
.

k.

:.
.

P.b=k(a.cb b-a.bb.c)
.

(4)

Now P b=aX(bxc) b

=a
as the
if

.[(bxc)xb]
maintained
,

position of

dot and cross can be changed at pleasure


is

the cyclic order

=a

[b

b c-b
.

=a

b-a

c b] by the rule written before when two vectors are equal bb.c , (5)

Equating the values of

P b
fc==l,

from

(4)

and

(5),

we

get

t-L
Hence from
or
(3)

by putting

we

get
.

P=aX(bXc)=a .cb-a b
(bxc)Xa=-aX(bXc)
The above obeys
rulel.
e. first

the

=a . b c a . c b. same law as written before

in the

we
;

of nearer ones

write the dot product of extremes and then the factor outside the bracket is included

in both the dot products.

142

Vector Analysis

is

Note : Just as a . (bXc) b . (cXa) etc. but aX(bXc) not equal to bX(cXa) as the former is coplanar with
is

b and c and
later

expressible in terms of b and c whereas the coplanar with c and a and is expressible in terms of

c and a,
i. e.

bx(cXa)=b
2nd Method.
Let
Consider unit vectors, i, b=6 2 j sa Y an d let .".

c-b

c a.

P=ax(bxc). j, k and let b be along j. k be perpendicular to b and


let

in

the plane of
/.
i,

b and

c.

c= 2 j+sk
k
/.

and

the third vector

in

terms of

j and

be flji+flj-f

3 k.
2c3 j

b X c=6 2 j X (f 2 j+ 3 k)=
/.

X k=

fc

2 c 3i.

a X ib X c)= (^1+

V jxj=0and jxk=i, j + 3 k) x (Va)


2
<?

^k X

Again

(a

......... (1) V jxi=-iXj=-kandkXi=j. (c j + cjt)] b j c) ^^[(a^+a^ + a^L)


.

(a

b) c^Kflii+flJ

+ flak)
-r

b, j]

U 2 j+c 3k)

(a

c)
(1)

b- (a
and

b)

c=0 3
get
c)

3& 2 j

- aj>tfjk ...... (2)

Hence from

(2),

we
.

a X (b X c)=(a
Similarly,

b - (a

b) c,

bX(cXa)=(b a)c-(b c) a. cx(axb)-(c b) a-(c a) b. Hence proved. Adding, we get aX(bXc) + bx(cXa)+cX(aXb)=0 because (a b) c=(b a) c etc. (Agra 42, 53, Annamalai 38, Andhra 36)
,

Multiplication ot Vectors

143
to

Note
shown
i

The component
r
2

of

r perpendicular
/

a was
\

.1be to

a2
.

.a
a

(See P. 103)

*>

r-r

a a

,ar-r
s=r

.a a
(See

8 P. 120)

ax(rxa)
a2

Or

aX(aXr) ----4

Note
by taking
7.

Verify the above formula


.

aX(bXc)=(a c) b-(a b) c a=i-2j+k, b=2i+j+k, c=i+2j-k.


Scalar product of four vectors

(Agra
c

37, 51)

(axb) .(cxd) =

a
Let us suppose that
.'.

cxd=n.
.

(axb) .(cXd)=(aXb)

n=a

(bxn)

as the position of dot and cross can be changed,

=a

.[bx(cXd)] = a

[b
.

.da. c-b

ca

dc-b c d] b d= a c
. . .

a
8.

'

Vector product of four vectors (a X b) X (c X d)


(Agra 38, 42)
. .

Let c X d = n.
/.

(axb)x(cxd)=(axb)xn=a nb-b na = a (cxd)b-b.(cxd)a ......... (1)


.

Again
/.

let

us put

=[acd]b-[bcd]a. (axb)=sm.
, .

(axb)X(cXd)=mX(cXd)=m d c-na
=(axb) d c-(aXb) . c d
.

Equating

(1)

and

[abd]c-[abc]d (2), we get

......... (2)

[acd]

b-[bcd] a=[abd] c-[abc] d

144
or

Vector Analysis
[bed] a-[acd]

Now

replacing

d by

b-[abd] c-[abc] d=0. r, we get

[abcj r=lbcr] a-[acr] b+[abr] c

...... (3)

a scalar triple product remains unchanged if the cyclic order of the factors is maintained but tor every change of cyclic order, there is a change of minus sign.
/.

Now

[bcr ]=[rbc], [acr]=[rac]=


[abr] = [rabj.

- [rcaj

and

Hence from

(3),

we have
'

" r = [rbc]a+[rca]b+[rab]c

...... (4)
57, 60)

[abcj

(Agra 35, 48,

expressess a vector r in terms of any other three vectors a, b, c provided they are not coplanar
relation
t>.,

Above

[abcj^O.

Rule.

The

last three

vectors
i.e.,

in the

numerator are the

cyclic

arrangement of a,
being r.
9,

and

c,

bca,

cab

and

abc and

the first

-^^
Reciprocal system of vectors.
three vectors

The

___
~[afacj'
If

7
,

b', c' defined

(Agra 59, 60) by the equations

~[abc]'

are called reciprocal system to the vectors a, b, c which are

non-coplanar

i.e.

[abc]^0.
a, b,

Property
system
XT

i.

c and
.

a', b', c'

be

reciprocal

of vectors, then
*

a'^=b
ass

b'=c
,

c'=l.

Now a

=a
/.

b *c
r

ti~i

r~ir~i

SSB

l'

[abc]
Similarly,
.

[abcj

a
,

b b'=c . c'l. . a'+b b'+c


*

c'3
,

or

1
-I-

1
=
.

a ,b

and c

Multiplication of Vectors
It is

145

because of the above property that the two syscalled reciprocal systems.

tems of vectors are

Property
i.e. 9

a.

The product of any

vector

of one system with


to it is

a vector of the other system which does not correspond

ero,

b'=0.

a
as the numerator

,,
.

b'=a

FcXai L
[abc]

U facai]=()
'-

[abc]

is

the scalar triple product of three vectors

two

of

which are equal and hence it is zero. (Cor. 2 P. 137) = b . c' c . a'=0 etc. etc. Similarly a c'
Thus we
reciprocal
a', b', c'.

Cor.
a', b', c' be

conclude

from

the

system

to a,

b, c, then a, b,

two properties that if c is a recipro-

cal system to

Property

3.

The
is

scalar

triple
to

product [abc] of any three

non-coplanar vectors

reciprocal

the corresponding scalar triple


a', b', c'.

product formed out of the reciprocal systtm of vectors

(Agra

47, 51, 59)

''
[a'b'c']=a'.0>'xc')

...... (1)
b' and c' in terms of

Now
a,

substitute
c.
r
., [abc]

the

values of

a',

b anc
.

..

(bxc).f(cxa)x{axb)i --------------label 8

m
.

...

......
c

Now (cXa)x(axb)=(cXa)Xm say=m


.

a-m
.

ac

ca-(axb).ac=[abc]a.
,--

[abc]
"

_.
LabcJS

V (aXb).a=0.
ttom(-)

[abc] [bca]_[abc] [abc] Iabc] fabc] ~[abc]'


.'.

_ ~

__ _

[abc] [a'b'c']=l.

(Agra 47)

146

Vector Analysis Cor.

From

above,

we

conclude that
c,
.

c X a]=[abc] 2 (Agra 36, 41, 51, 53* 57> 6 5 Pb - 6 ; Andhra 38;

a X b, b X \

Benaras
If

56;

Raj pu tana 56)


i. e.

a, b, c be expressed
2

in

tfii-f

j+0 3k

etc

^ien we

terms of unit vectors, have already done that

[abc]=
*3

C*

(Cor. 4 P. 138)

Again

ax b=(a 1i
i*2- fl A)

(P. 121)

and similarly we can write


/.

for

bxc

and

ex a.

[aXb, bxc, cxa]

(Cor. 4, P. 138)

"

-"2

-"

GI

C2

C3

(where capital

letters

denote the co-factors

of the corresponding small letters)

which

is

equal to
*, i. e.

[abc]

2
.

(Refer Author's Algebra]

Multiplication of Vectors

147
e.

Note.

//

a, b,

c are non-coplanar
also

i.

[abcj^O,
their

then

[aXb, bXc, cXa]


product
is

are
is

non-coplanar

as

scalar triple

2 [abc] which

not zero.

Cor.
expressed

We
in

have done before that any vector r can be

terms of three non-coplanar vectors b] c

r JrfElj.
_^

jMi
X c)
,--i

a, b, c.

[rbc] a _ r
[sJbcJ

(b

aa
of vectors to a, b,

[abc]

where
c;
/.

a', b', c'


.

form a reciprocal system


.

r=r a'a+r

b'b-f r

c'c.

(Agra

38)

Also the two systems of vectors [abc], [a'bV], each is and as such any vector r can reciprocal of the other
also be written as

r=r

a a'+r
j
.

b b'+r
and

c'.

Again

i-j

^k k = l

[i j

k]=l.
k,

the system of vectors i, j, k Hence in terms ot unit vectors i,

is its j,

own reciprocal. we have


[See P. 104]

r=r.ii+r
Ex.
i.

.jj + r.kk.

Exercise
Prove
that

being three vectors.

[a+b, b+c, c+a]=2 [abcl, (Dacca

a, b,

27, 29)

Ex.

2.

Prove that

[Imn] [abcjsa

l.a

l.b

l.c

m
n
and give

m
n

n c
.

its cartesian equivalent.

(Agra
Let
a', b', c'

38, 40, 47, 49, 51, 54;

Lucknow

52, 5

Pb. 60; Benaras 52; Annamalai 47)


be a system of vectors leciprocal to
a, b,

and hence [a'bV ] [abcj-L

148

Vector Analysis
Also

we know

that any vector r can be expressed in

terms of any three non-coplanar vectors as r=r . a a'+r . b b'+r . c c'.

We
a', b', c'.

shall

express the vectors

1,

m, n
.

in

terms

of

/.

1=1 . a a'+l . a a'+m n=n . a a' + n l.a 1 . a [lmn]=

b b'+l
.
.

c'
.

m=m

b b'+m c
.

c'.

b b'-fn c c'. 1 . b l.c l.b


,

[a'b'c'l

m
we

m
n
.

c c
[Cor. 4 P. 138]

n*a
get the required result.
'

n b
.

Multiply both sides by [abc] and since [abc] [a'b'c']=l,

Cartesian Equivalent.

Let

l=y+/ 2 j+/3ketc.
etc.

.'.

[lmn][abc]

...(1)

Also

a=
b l.c

i2

=j

=k2 =l.
...(2)

l.a

m.c
,

n a n b n

k
Wl

I*

7^2

Multiplication of Vectors

149
c,
-

Note

In case

1,

m, n
.

are

[abc] [abc]=

a.b

same as a, b, a. c

then
[abc]

we
2

get

b.a
or

b.b

b
.

c . a c .b c c =[axb, bxc, cXa] as proved in Cor. P. 146. Note. 2. In case 1, m, n are reciprocal to a, b, c

then

[Imn] [abc]=l and 1 . . a=*m . c=0, n . a=n

a=m b=n
.

c=l but 1 b=l c=0,


.
.

b=0.

[Prop.

2.

P. 145],

(Benares 55)
/.

we

get 1=

1
1

001
Ex.
3.
Prove independently that
2

[axb, bxc, cxa]=[abc]

a. a

a.b a.c
.

b.a b.b b
c
.

The

first

part

is

proved on page 145.


(1

Now we know

that (aXb) 2 =0 26 2 sin 2 B=a*b*

-cos 2

6)

=a b
2

-(a.b)
itself

2
.

square

oi

a vector

is

square

of its

module and
a2

it

stands for the dot product of a vector by and also a b=a6 cos 6.

i.e.

=a2 =a

Now [(a X b) x c]2 =[m X c]2


2
2

m c - (m
2 2

c)
2

=(aXb) c -[(aXb).c] a a 8 [ab'-(a.b) ]c -[abc] a 2 2 a 2 a =a b c -(a . b) c -[abc]


Again [(aXb)Xc]=(c 2 .% [(axb)xc] [(c .
.

(1)

a)

a)

b-(c a b-(c .b)a]


.

b) a,

= (c

a)

b2 +(c

b)

a3 - 2

(c

a) (c

b) (b

a)

150

Vector Analysis
2

Equating the values of [(aXb)Xc] 2 a 2 aa (a . b) c - [abc] get a*b c

from

(1)

and

(2),

we

/.

b2 +(c . a 2 a a (a [abc] =a b c
(c

a)

b)
.

2 2

a2 - 2 (c
2

a) (c
c)
2

b) (b
.

a).

b) c

- (b

a 2 - (c
.

a)

b2

+ 2(a.b)(b
Now
if

c)(c.a).

we expand

the determinant on L. H.

3.,

we

get

the same result.

Hence proved.

Ex. 4.

Prove that if I,

[lmn](aXb)=

m, n be three l.a l.b


a

non-coplanar vectors

m
.

m
(Agra 49, Dacca 40)

n a
m, n and

n b
b
terms of unit vectors

'Expressing

1,

a,

in

and
etc.

and

[lmn]=

and (axb)=

k
<*a

ml

<*i

^3
bo,

bt

bs.

.'.

[Imn](aXb)

l.a
m,
v

l.b

l.a

l.b

TO.

m
.

m
n

n a
.

n b

n a

n b
.

Multiplication of Vectors

151
vectors, then prove

Ex.5,
the following
1.
:

//a,

\*3

are

three

non-coplanar

[b X

c,

c X a,

a X b]=[abc]2

(Agra
2. 3.
4.

36, 41, 515 53 9 57> 6

Benaras

56,

Andhra

38,

Rajputana

56)

b X c,

c X a, a

Xb

are also non-coplanar.

Express a, b, c in terms
Express
first

ofhxc, c X a, a X b.
terms o/a, b, c.
in Cor.

bxc, cxa,

axb in

The

two parts we have already proved

P. 146 and Note P. 147.


3.

Let

a=/(bxc)+m(cxa)+(axb)
.

...... (1)
.

Multiplying both sides scalarly by a,3

a
or

a=Z a (bXc)+m a . (cXa)-fn a (aXb) a a=6 [abc].


7

scalar triple

product

is

zero

when two

vectors are

equal.

a " /_

'

[abc]'

Similarly multiplying both sides of

(1)

scalarly

by b

and

c,

we

get

m== r~il~~i an u n=: F~T~-i.


[abcj
[abc]'
(c

a.b

(b

X c)=b

X a)=c
Z,

(a

X b)==[abc],
(1),

Substituting the values of

m and

n in

we

get

Similarly
4.

we can write

the value of

b and

c.

Let(bXc)=/a+/Hb+Hc
(2)

...... (2)
by (bxc). . (b X c)+ nc
.

Multiply both sides of

scalarly

\
..

(b

x c)

(b x c)=rJa
i

(b x

c)+mb

(b X c).

as u t before

(bxc). ^---J.
(bxc)

152

Vector Analysis
Similarly multiply both sides of
(2)

scalarly

by

(cxa) and (axb) and

find

m-^^J^-*
_(bxc).(axb) T~ t
;

[abc]
(2),

Substituting the values of

I,

m, n in

we
and

get the

required result.

Similarly
c.

we can
r

express

cxa

axb

in

terms of

a,

b and

Ex. 5.

Express a vector

as a linear combination of a vector

a and another

vector perpendicular to

a and

coplanar with

r and a.

have already stated that the vector aX(aXr) is a vector perpendicular to a and coplanar with a and r and as
such
dot product with a is xero, perpendicular vectors is zero.
its

We

dot product of two

Let

r=/a+wiax(axr)
v\e get
.

...... (1)

Multiplying both sides scalarly by a,

a*=/

a . a,+m a [aX(axr)]=s=Z a
.

Again multiply both sides of


.\

(1)

vectorially

by

a.

rXa=/aXa+[aX(aXr)]Xa
s=0+m
[a
.

a-a. ar]Xa.

rxa=m[a.

r (aXa)-a. a (rXa)]=
"~

-m (a

.a)

(rXa).

a
/

.a'

Substituting the value of

and

in (1),

we

get

P== ?J-? a _

a .a

a .a aX(aXr)

J_

<

Above shows that the component

of a vector

r along

Multiplication of Vectors

353

a given direction a
to

is

a and

in

a direction perpendicular

is

- ao<jaxr) -a
6.

See

g p^

Ex.

Prove that

[aXb, cxd, exf]


[abdj[cef]-fabc][defj
[abe][fcd]-[abf][ecd]

*[cda] [bef J - [cdb] [aef ].

(Agra

36, 61)

\Ve know that any scalar triple product [pqr] is equal to p . (qxr)=q . (rXp)=r . (pxq). Writing the given scalar triple product in the above three ways maintaining
the cyclic order
shall

we

shall get the three results as given.

\Ve

show only one students themselves.

and the

rest

can be done by the

faxb, cxd, exf] =(aXb) [lcXd)X(eXf)] =(aXb) [nX(eXf)] e(aXb) [n. f e-n e f ]
.

(n

f)

Hcxd)
Ex.

[axb) c]-(n . e) [(axb) f [abe]-(cxd) e [abf]


.
.

f]

[abe] [fed]
7. Prove that [a

[abf] [ecd].

+ [a X r, b X p, c X q]= 0.
Expand
and
its
first
.

X p, b X q, c X r]+[a X q, b X r, c X p] ( Agra 34, 48, 59; Luck. 55)

bracket as

third as

C (Ax B)
is

A. (BxC), second as B . (Cx A) and then add keeping in view that

scalar triple

sign

is

unchanged if cyclic order is maintained and changed for every change of cyclic order.
If a, b, c and
a', b', c'
:

Ex.

8.

form

reciprocal system of

vectors, then prove the following

(7j

a.a'+b.b'+c.c'^J*.

154

Vector Analysis
(2) /Si
(

'

axa'+bxb/+cxc'=0. _b'xc' K _ c'Xa' _a'xb'


[a'b'c'j'

[a'b'c']'

~[a'b'c']'

Ex
*

9.

Find

the value

of

P=i X (a+i) + j X (a X j) + k X (a X k)
P-(i

= 3a-a=2a.
Ex. 10*

a)i+(j . j) a-(j . a) j+(k . k) a-(k -(a+a+a)-[(a . i) i+(a . j) J+(a . k) k]


.

i)

a-(i

a)

2'11 P. 10G]

Prove the relation


.

ax[bx(cxd)]=b daxc-b . c aXd =[acd]b-a .b cxd


and hence we prove
that

ax[bx{cx(dxe)}] =[a d c . e-c. d a


.

e]

b-f(a

b) [c

d e-c.

e d]

(Agra

37, 42, 55;

Delhi 51)

L.H.S.=ax[b . d c-b
or

.
.

c d]

etc.

=a
Ex.

(cxd)

b-a b cxd.
.

n. Prove that (bxc) .(aXd)+(cXa) . (bxdj+(axb)


(A+B)
c os
*

(cxd) =

and dednce that


sin
sin

(A-B)=sin

A- sin2 B
2B-cos 2 A) = cos* B-cos 2 A

=J
and
(A

(cos

+ B)
5>

cos

(A - B)

= cos 2 A - sin 2 B.
53; Delhi,
55,

379 4 6 >

539 60;

Benaras

Luck.

Allahabad

Sc.

60

b a b d
.

b
.

d
d
b) (a
.

a.d
b.c
[

a
.

b, d
7 P. 143

=(b

a) (c

d)-(c

a) (b

d)+(c

d)-(a

b) (c

Now

using the fact that

+(a.c)(b.d)-(b.c)(a.d) b a etc., we get t b


.

Multiplication of Vectors

155

L. H. S.=0.

Let

and
Let
:.

LCOD
Fig No 81

:.

LAOD
n be a unit vector perpendicular to d which are assumed to be coplanar.
that

Let
a, b, c,

the

plane of

Now we know

(aXb)~

sin

n where

is

the

angle between the directions of a and


in anti-chockwise direction

treasured from

towards b.

Again we have proved that (bXc) . (aXd) + (cXa) . (bxd)-f(axb)


or or

(cxd)0
.

(bXc). (aXd)-(axc).(bXd) + (aXb)


(be sin

(cxd)=0

EOC) n

(ad sin

AOD) n

-(ac sin AOC)


+(ab sin AOB)
or
abed sin

n n

.
.

(bd sin
(cd sin
.

(A- B)

sin (A

+ B)abcd
2

sin

}-abcd

BOD) n COD) n=0 A sin A sin B . sin -6=0

V
.'.

nl.

sin

(4-B)

sin

OH^sin

-4

-sin 2

Again (aXb)

(cxd)

d
d)
.

c)(b.d)-(b. c)(a

d)

156
or
(ab sin

Vector Analysis
B)

(cd sin

B)

n=(ac cos A) (bd cos A) -{be cos (A-B)} {ad cos

Cancel abed and put


or
or
sin 2

B - cos a
(A+B)

n n=l as n is a unit vector, A - cos (A - B) cos (4 +)


-4

cos

cos (^l-JB)=co& 2
2

-sin 2
sin
2

B
-4.

=cos B
Ex. 12.
Prove that
.

(aXb)
Ex. 13.

axc) + (axb)

(aXc)=(a.a)

(b

c).

(Annamalai 39)
Prove that

2(cXd)X(axb)=

~d
where

(cXd)X(aXb)

(cXd)Xm=(m

c)

d-(m

d) c

[(axb).c]d-[(axb).djc [abc] d-[abd] c d-| 0, c

ft

a9

...(1)

[Cor. 4 P. 138]

Aagain putting

(cXd)=n and

proceeding as above,

(cXd)X(aXb)=

...(2)

rf

Multiplication of Vectors

157

Adding,

we get 2(cxd)x(axb)=

Ci

C%

di

d%

c/

If

we expand
(1)

the above determinant,

we

get the

four determinants of

and

(2).

E%. 14.

Prove that

(aXb)X(cXd)-f(aXc)x(dXb)4-(aXd)x(bXc)=:2[bdc]a. (Andhra 38) Expand ] st in terms of c and d, 2nd terms of a and c and 3rd in terms of a and d etc. Ex. 15. Show that the perpendicular distance of a point C b X c+c X a+a X b from the straight line through A and B is b a where a, b, c are the position vectors of A, B and C. -7The vector area of a triangle ABC is
|

I
Its

(aXb+bXc+cXa).

[Ex. 6 P. 128]

module

is

J base

ABx perpendicular
|
|

from C on

AB

aXb+bXc+cXa ~ b-a

CHAPTER IV THE PLANE AND SPHERE


Vector equation of a plane. Let there be a unit vector
1.

(Agra 31, 39)

A
n normal
to

the plane.

If

be the length of the perpendicular from origin on the plane,

Let
plane f
;

be the position vector of any point then the projection of OP on OJV is p.


r

P on

the

But projection
and
it

of

being equal

OP on OJV is r top, we have

cos

0=r

cos

A 0=r n
.

the required

equation

.................. (1) n=/> standing for the length of the perpendicular from the p
of the plane as r
.

origin.

Cartesian form

Let the coordinates and k through


be
#, jy,

of

referred to

unit vectors

i, j

so that

If

/,

m, n be the direction cosines of normal, then

A
A
;.

or

Ix+ny+ik-p,
i.

2
j

and

=.k=k.i*

The
which
is

Plane and Sphere

159
Coordinate

the standard equation of the plane in

Geometry.
Cor.
In case
there

be

any vector

parallel to unit vector

A
n
and of module
n, then

A
n=rc
.

n.
(1)

Multiplying both sides of

by

we

get

A
w(r.n)=n/>
or
r
.

nnp=q,

say,

where

= qln.

n=<7 represents a plane ; the length of the perpendicular from the origin is obtained by dividing the R. H. S. by the module of n,
r
.

Thus

i.e.

R.H.S. g_ Module n" n


of

Cartesian to vector.

If

the plane be 2*-f 3?-h4e

10,

then

the

corresponding
.

vector

equation

is

evidently
10

(*i-b>j+*k)
Ai

(2i-f-3j4-4k)
r
-

= 10 i.e.t.n^q
-

and the length of

r r perpendicular from

origin b

is

module

T-T

9 ot

r~ n

V(2

KM +3 2 +4 2 02 r^a; 2
)

Cor. 2.

The

equation of a plane that passes through

a given point.

Let the position vector of a point A be a through which the plane passes and whose normal is n. If r be the posiof any point in the plane, then and as such n is perpendicular to AP. plane

tion vector

AP

lies in this

->

Therefore
or

^P.n0
or
r
.

(r-a).n=0

n=sa

q say.

The

length of the perpendicular from origin on the

160

Vector Analysis
is

plane

._

a.n
|n|

|*|
s=*fl

^^

a.n
n
0, i.e.

a.n

IV
L

n *j

1 cos

the projection of 0-4 along the normal.


.

Converse
sents a plane.

To prove

that the equation r

n=q

repre-

*.n=tf
Let a and b be the position vectors A and B which satisfy (1).
/.

...... (1)
of

any two points

and
*

a.n=0 b.n=#
(3)

...... (2)
...... (3) and adding, we get
q

Multiplying

(2)

by

and
.

by n

(/wa+wb)

n=(m+)
n== *

ma+nb

i^rEquation
.

...... (4)
point

vector
> T

is

/wa-f nb -

it

(4)
.

shows that the


.

whose

position

also satisfies

/1V
(1).

jsow

19

an y point on the
:

if

..

line

joining a

and b
that
(1)

and divides

in

the ratio n

m.

Thus we observe

every point on the line joining A and B satisfies equation which therefore should represent a plane.
Cor.
vectors

sides of a plane : The points whose position are a and b lie on the same or opposite sides of a plane
3.

Two

t.n~q

according

asa.n

and

q are of the same or

opposite signs.

Let the line joining A and B intersect the plane at P which divides AB in the ratio n m and therefore its
:

The
position vector
is

Plane and Sphere

161

-----

Since this point

lies

on the plane

n=?
or

(a

n - q)~ - n
n
-

(b

n - q)

or
If

m
n\m

=_a

n q b .n-g
.

is +ive, i.e. a . n-# and b . n-g are of opposite then P divides internally the join of A and B, i e. signs, A and B are on the opposite sides of the plane.

If

n\m

is

ive,

i.e.

a.n

</

and b

# are of

same

sign then P divides the join of A and B externally, i.e. A and B are on the same side of the plane. Hence Proved.

Ex.

1.

Find

the

equation

of

the

plane

through

the point

2i-{-3]~ k

and perpendicular

to the

vector

3i+4j-f7k.

Deter-

mine the perpendicular distance of this plane from origin.

Here n=s.Si+4j + 7k and


.'.

a2i+8j

k.

ft.n=6 + 12-7=41 and/zV(3 2 +4 2 +7 2 )==8...(l)


equation of the plane through a point a
(r is

Now
or

a)

n=0

or

n=a

n.

[Cor. 2]

r.(3i+4j+7k) = ll.
Also

[from(l)]

/>=*^~.
is

[from

(1)]

In cartesian form its equation


2.

3^+4y+7-c=ll.

Equation of plane satisfying the given conditions.


already seen that the equation of a plane is and a plane through a given point is (r-a) . n=0,
.

We
f
i.e.

have

n=
t
.

n=a

n.
shall

Here we

other given conditions


triple product.

deduce the equation of plane satisfying by the help of vector and scalar

162
I.

Vector Analysis

Equation of a plane through three given points.

(Agra 51) Let the position vectors of any three points A, B and C through which the plane passes be a, b and c respectively.

Let

Now

be the position vector of any point Pon the plane. the points P, A, B, C all lie on the same plane, i.e.,
r

are coplathree vectors are coplanar, their scalar triple product vanishes. [Cor. 1 P. 132]

vectors PA,
nar.

AB and BC

i.e.

r-a,

b-a

and

c-b

Now we know
/.

that

if

(r-a).{(b-a)X(c-b)}=0
.

or
or
r

(r-a) a
/.
*

[b

xc-axc+axb]=0,
.

V bxb=0

.[bxc+cxai-axb]
.

(bxc)+a
.

(cxa)+a
.

(axb).
(1)
.

[bxc+cxa+axb]=fabcj
and a
is

V V

-axc=cxa

(cxa)=0*=a

(axb),
vectors are

scalar triple product

zero

when two
r

equal.

[Cor. 2 P. 132]
(1) is

The equation

of the

form

n=<? and

is

therefore

the required equation of the plane.

The

plane

is

clearly perpendicular to

n=sbxc+cxa+axb
which
is

equal to

2&ABC

of perpendicular

ON from origin
A
.

(Q. 6 P. 128). If p be the length on this plane, then

-">

whereas

N=p

n
.

fabcl L
n9

n=/>

^(bxc+cxa-J-axb)
[from (2)] non-parametric vector

Note.

Equation
already

(1)

is

called

equation of a plane through

three points, a,

b and

c and

we have

found

the

corresponding

parametric

The Plane and Sphere


equation of the plane in Chapter II
9,

163

Cor. 2 P. 82 as

r=a+5(b-a)+/(c-a).
Multiply both sides of above scalarly by

bxc-fcxa+axb.
r
.

(bxc+cxa+axb)=a (bxc+cxa + axb) +5 (b- a), (bxc+cxa + axb)


.

Now we know
two
/.

that

+t (c-a) (bxc+cxa+axb). scalar triple product is zero when


,

of the vectors are equal.


coefficient of s is

[Cor. 2 P. 132]

b (cxa)-a
.

(bxc)=[abc]-[abc]
/ is

0.

Similarly coefficient of
r , (b

zero and hence


.

we

get

X c+c X a+a x b)=a


(1)

(b

x c)=[abc]

which

is

same

as equation

found above.

Corresponding Cartesian form.


Let
then
in

terms of unit vectors


a^sflji+^j + ^ak,
(r- *)=(*-*!)

i, j,

r=#i+^j+k; l+(j-aj }+(z-aj k,


5-<2 3) k,
fa
^3)

and

k.

Since r-a,

b-a,

c-b

are coplanar,

we have
=0

as the required equation of plane.


II.

Plane through a given point and parallel to two


parallel to

given lines.

lines

Let the plane pass through the point a and which are parallel to b and c.

164
Since the plane
to

Vector Analysis
is

parallel to b and c is therefore hence the required plane is one bxc, perpendicular through a and perpendicular to bxc and its equation

therefore
or

is

(r-a)
.

(bxc)=(X
.

(bxc)=a (bxc)=[abc] The corresponding parametric equation of the plane is r=a+j b-f t c [Cor. 1 P. 81] and on multiplying both sides scalarly by bxc, we get the form (2). The plane containing a given line and parallel III.
r
('2)

Note

to another line or perpendicular to a given plane r . c=</. Let the plane contain the line r=a+/b and is parallel to c which means perpendicular to plane r . c=^.

Thus the plane contains the point a and is parallel to both b and c and therefore perpendicular to bxc and hence
its

equation
IV.

is

(r

a)
.

(bxc)=0

(bxc)=[abc]. The plane through two given points and para.

(bxc) = a

llel to a given line.

(Pb. 60)

Let plane pass through two points a and b and given line be parallel to c. Thus the plane is one through the point a and parallel to b a and c and therefore perpendicular to (b a)xc. Hence its equation is
the

(r-a).{(b-a)xc)
or
r
.

{(b-a)xc}=a

{bxc-axc} = a

(bxc)=[abc],
line

V.
point.

The plane containing


line

a given

and a given

Let the
plane
is

be

r=a+'b and

through

a and

given point c so that the c and parallel to b i. e. through a


(a c)

and

parallel to a

c and b or perpendicular to
is (t

X b.

Hence
or
r

its

equation

a)

[(a

c)xb]0

,[(a-c)xb]=a .[axb-cxb] = -a. (cxb)=a. (bxc)=[abc].

The
3.

Plane and Sphere

165

Angle between two planes.


.

Let the two planes be r n l ^q l and t . n 2 q2 and the moduli of n t and n 2 be n t and n 2 respectively. Now angle between two planes is equal to the angle between the normals to the planes. If be the angle between n x and n 2 then n x . n 2 cos 0.
,

=i2

Angle between a line and a plane.


Let the

Now
plane.
If

be parallel to b and the plane be r . n=#. angle between a line and a plane is compleline

ment of the angle between the

line

and the normal

to the

be the an^le between the line and plane and

$ be

the angle between normal and line, then n b=ft cos $. Now
;.

0=90
0.

0.

cos r

<= n

nb

=rcos (90-0)=sin v
.

~ Sm_
4.

b
9

nb'

Intercepts on ax:es of coordinates (rectangular).

Let the equation of the plane be r . n=. Let the unit vectors along the axes be denoted by

i, j

and k respectively. If x be the intercept made by the plane on the axis of x, then the point xi lies on the plane.
/.

xi

n=

<?

or

x=r-*
i
.

respectively.
-axis are

Similarly the intercepts on

the^ and

and

T7H
Ex.
intercepts
all

k !n'
of the
fixed plane is same for

2.

Prove that sum of the reciprocals of the squares


rectangular

on

axes

made by a

systems of rectangular axes with a given origin.

166

The Plane and Sphere


Let the plane be r 11=5, If x, y and * are the intercepts on the axes, then
.

-JL
If

-^i
2

the normal

makes angles 6l9


2

2>
i.e.

8%

with

i,

and

k,

then

cos

^+cos
.

+cos
,

= 1,

l*+m*+n*=l.
2

Also

and
'

n=l n=l n cos X k.n=l.KCos03


j

n cos

...... (2)

-72+72+ ?- ?
i.e.

(cos2

ei+cos2

^ +cos2
[from

ea)=s

f'(1)

and

(2)],

constant because the plane


5.

is fixed.

Perpendicular distance of a point from a plane.

(Agra 41)

Let the equation


plane be
r
.

of the

n=j, so that

perTT/

pendicular from the origin on it, i.e. ON=p=qln.

We have

to

find the per-

pendicular distance of a point A, i.e. a from the given plane.

N
Fig. No. 83

M
A and
parallel

Now

consider a plane through

the point
is

to the given plane


(r

whose equation
a)
.

n=0

or

r.n=a.n.
from
origin

ON'p'*= length of perpendicular


.
.

on

this

plane r

is

a.n -n

Also

AI

j-a.n Tr ?-a,n n -^ = AM~*

n.

The

Plane and Sphere


to specify a vector,

167

Whenever we have

we

multiply

its

magnitude with a unit vector in that direction


or

AM=*
The above
value of
lie

~
n2
is

AM

positive for all those points


origin.

on the same side of the plane as the ive. points on the opposite side it will be
which
Alternative Method.

For

Let
then
its

AM be

perpendicular from a
is

to the plane r

n=0

equation

r=a+/n.

. .

.(1)
is

and being perpendicular to plane

through a to normal n, parallel


as

AM passes

M
hence
or

is

the

intersection of

this line

and the plane and


2 [V n =
2
]

we

should have
.

(a-fm)

n=

or

n-ffn =fl,

*=?~
Putting the value of
/

a
ra

Vn

in (1),

we

get the point

M as

n*

n-a

HA|-'-i- .|n|-t^?-a. n
Ex.
(2,1,
3.

Show by
are

-4)
the

vector method tliat the points (1, 1, 1) and on opposite sides of the plane

3*+^+5=9.

Find

perpendicular

distance

of the

former from the plane and

the vector through it perpendicular to the plane.

1G8

Vector Analysis
In terms of unit vectors
i.

j,

k,

A=*i+j+k, B=2i+j-4k,
and the given plane is (*i+j>j-f k) . (8i+4j + f)k)=9 so that 2 2 2 n=s3i+4j + 5k and * = V$ +4 +5 )=5V2.

Now

perpendicular distance of a point from a plane

is

#-a
n
/.

perpendicular distance from A,

5V2

___
i.e.

i-f j

+k

is

~~

"

5V 2
;

~5V2
is

Perpendicular distance from 5,

2i+j-4k 9~(G+4-20) "


i.e.
"

19

Again since
are
,

^-a.n^=-3

and

5V2 ^-b.n=19,
two points
[Cor.
1

x.^.

they

opposite signs, therefore opposite sides of the plane.


of

the

are

on

P. 160]
is

Also the vector through

perpendicular to plane

(b)
equidistant

Show that from


of
it.

the points
r
.

i-j+3k
(5i-f2j

the plane

and 3 (i+j k) are 7k) 9=0 and lie on

opposite sids

Cor.

(Agra 59) Distance of a point from a plane measured in


to find

a given direction.

Here we have

the distance of the point A from the plane but mea-

sured in a given direction

say of the unit vector b.

Fig H*l

A
Let a
line

through

parallel to unit vector

b meet the plane

The Plane and Sphere

109
-*

in

M and
The
on

let

d be the length

AM so
f.

that

->--

AM =*d

A
.

b.

A
b

lies

position vector of Af, the plane r . n=q.

e.

OM=OA+AM~a+d

A
q-a rf~
b
which
is
.
.

n
from
the

the required distance.


4.

Ex.
plane

Find

x+y+z=*5

the distance of the point measured parallel to the line

(1, 2, 3)

The The

given point

(1, 2, 3) is
is

i+2j+3k=a
.

say.

n = i+j+k and q=5.

given plane

(^i+^j+^k)

(i-J-j+k)=5 so that

The
.*.

direction ratios

of the line
is

are

2, 3,

and hence

a vector in this direction

2i

+ 3j-6k=b.
is

a unit vector in this direction

Hence

the distance of

A from

the plane

is

g-a.n 5~(i+2j+3k).(i+j+k)
A
b
n
>(2i+3j-Gk) .(i+j + k)

6.

Planes bisecting the angles between the given

planes.

Let the equations


r
.

of the

njssr?!

two planes be and r . n a =sv

170

Vector Analysis

if r be any point on the plane bisecting the angle the given planes, then the perpendicular* distance between of t from both the planes should be equal.
.M.

Now

Above
7.

are the required equations of the planes.

Plane through the intersection of two given

planes*
,

Let the given planes be


(r

r
.

n 1 ^q l and
(r

t
.

n 2 =^2
n 2 ^q z)^0.
.

Consider the equation wheie A is any constant

nj-^-A

.(1)

Now
as such

all

those

planes also satisfy


it is

points which satisfy both the given the equation (1) for all values of A and

section of the
t
,

the points on the line of intergiven planes. Rewriting (J) in the form we get the required equation of (nj-Analft-A^g,
satisfied

by

all

plane through the line of intersection of the given The value of A is found by an additional given planes. condition of the question. For example, if the required
the

plane passes through a given point

a,

then

a*

n-

In particular, if the required plane passes theough q^qt and hence the origin, then putting 'a=aO, we get A

The Plane and Sphere


equation of the
r

171
(t
.

plane

is

q 2 (f

n t - qj - q l

n2 -

<? 2

)~0

or

.(jtih-ftii^B-oLine of intersection of 8.

two planes. (Agra 51, 55, 61) [Read


two planes be and r . n 2 =<72
being
.

carefully]

Let

te

equations of the

t.n 1 =q l
The
line

of

intersection

common

to

both the

planes is therefore perpendicular to both n a and n 2 the normals of the two given planes. But n x xn 2 represents a vector perpendicular to both b^ and n 2 and hence we

^Xfig- In order to determine its equation completely, we must know a point on it. Now if JV be the foot of the perpendicular from
conclude that the
line
is

parallal to

origin
of

on the

line,

and n a

If

then clearly Off is parallel to the plane we denote the position vector of JV by a,

then

aas/^+^ng, where

Now
satisfy
/t

/ x and / 2 are any constants. on both the planes. .'. li^+lf^ should both the planes and this will give us the values of

JV lies

and
".

12 .

n i2 +/2 tti n 2 tfi (!) . n2 or l^ n 2 +; 2 n 2 2 =# 2 -(2) tfiih+^ng) =# 2 Multiplying (1) by n 2 and (2) by n x n 2 and subtracting, we get h [n^n^-Cii! . n 2 2 ]=^n 8a - ^^ . n 2
tfini-H 2 tt 2 )

ni

=tfi

or

'

:==:

fi

of point

Having found JV on the


its

li

and

/2 ,

we know
is

the position vector


to

line
is

which

parallel

n x X n 2 and

hence

equation

r=:/ 1 n 1 +/ a n 2

+m

x n2

Note,

Above

is

called

paranretric

equation of

the

line of intersection.

172

Vector Analysis

We

can however find the non-parametric equation of


line through a given point

the line as follows.


Equation of a
line.

and parallel

to

a given

be any point on the line which passes through a r a and b are .*. given point a and which is parallel to b. two parallels and we know that cross product of two parar
llel

Let

vectors

is

zero.
/.

(r-a)xb=0

or

rxb=axb
then putting

is

the required non-parametric equation of the line.

In case the line passes through origin,


a=*0,

we

get

its

equation as
is

rxb^O.

In case the line


clearly
it is

perpendicular to c and d, then b by cxd parallel to cxd and hence replacing

we

get

its

equation as (r-a)x(cxd)=0.
the non-parametric vector equation of the line of

Hence

intersection of

two planes

is

(r-a)x(n 1 xn 2)

(2)

where
It

a/

1 n 1 +/2 n a and

Il9 1 2

have values found above.


as in Note 102 that the non-

may

be observed here

line can parametric form of the equation of

be

easily

deduced from the corresponding parametric form

r=a+*

b.

6 P. 47]

Multiply both sides vectorially by b.


.'.

rxb=axb+/bxb=aXb.
is

/.

(r-a}xb=0
5. (a)

the required equation.

Ex.
planes r
.

Find

the equation
.

of the

line

of intersection of the

(3i-i+k)- J and r
line of intersection

(l+4}-Zk)2.

(Agra 45)

The

is clearly parallel to

na X n 2

The Plane and Sphere

173

U
or

(2-4) i+[l-(-6)l J+[12-(-l)] k

In order to find the line

-2i+7j+13k. we must know a

point on

it.

then since

Let a be the foot of the perpendicular from origin on it; it is expressible as a linear combination of n x and

n 2)

let

a=J (3i-j+k)-K2
a
lies

(i-f-4j-2k).

Now

on both the planes.

and

[/,

(3i
.'.

H/,-8/,-1

and

Solving the above two,

we
.

get
.

kthe line
i.e.

.27
r=a+*

and

/2

=.
25

Having found the point and


is

direction, the equation of

given by
of,

b,

05 f-- (3i-J+k)+^j(i+4J-2k)+/ (2i+7j+13k)

Above
Again
the line,

is

the parametric equation of the line.


to find

in order

the non-parametric equation of

(r

- a) X (% X n 2)=0
that
the

[Bottom P. 167]

or

rx(-2i+7j+13k)=5ix6j+4k.
Ex.
5.

(b)
1)

Prove

plane

through

the

point

jf,

7,

and containing
.

the line

of the planes

T
is

(i+3j-k)-0W

r.(i+2j-3k;0.

174
9.

Vector Analysis

Perpendicular distance of a point from a given

line -

(Agra 49)
line u

Let the given

be
u

A
r=a+/b
the point

passing

through

and

parallel to

A
unit vector b.
find

We

are

to
dis-

the perpendicular

tance of any point

C whose

position

vector

is

c from this
2

line i.e.CN. Clearly

C^O^-OC=a-cand G',4 2 =*(a-c)


square of
its

...(L)

because square
Also

of vector is

module.

is

projection of CA along unit vector therefore equal to CA cos 0=*l .CA . cos 0.
is

NA

A
b and
it

A
CJ\T
2

A
z

=CA -

A - c) a - [b

(a

- c)] 2
from
(1)

and

(2),

Also

=(a-c)-[b,(a-c)]b

A
because JV-Assb .(a- c) and A
.'.

A
it

being in the

direction of b,

A
vector then
of b.

NA=[\>. (a-c)]b.
not a unit
is

In case

b be
b

we

shall replace

A*

b by

b/A

where
6.

the

module

Ex.

find

the perpendicular distance


it.

of a corner of a unit

cube from a diagonal not passing through

(Agra

33, 41, 56)

Since the cube

is

a unit cube,

let the

vectors determined

The Plane and Sphere


by conterminous edges OA,
lively so that the diagonal

175

OB and OC
-

be

i,

and k respecpasses

OP

is

b=i-f-

j+k which

through

origin.

Fig. No.

86
is
-

A
If

unit vector in this direction

CM be

perpendicular from

C
-

e.

k on OP, then

OM= project ion of OC


s=sk
.
.

on

OP
OP

unit vector along

i+j+k

Also

OCk;
To

.-.

OC

-1.

10.

find the condition that any


i.

two given

lines

may be

coplanar

e.

they

may

intersect.

Let the given and


i. e.

lines

be

passing through a l9 a 2

and parallel to bj and b2


respectively.

In

case
Fig.No.Q7

they intersect, then their common plane should be

176
parallel to a 1 ~a a bj
,

Vector Analysis

and b 2
.

i. e.

these three
is

vectors should

be coplanar, the condition for which

that

(a!-a 2)
or or
is

(b 1

xb 2)0
.

[Cor. 1 P. 137]
)

aj

X b 2 )a a (b x x b 2 [ib 1 b]-[a I b 1 bJ
(bi

......... (1)

the required condition.

The

plane through the coplanar lines.

In case the given lines intersect, then condition (1) holds good and in order to find the equation of the plane through them, we write the equation of the plane through
the
point
parallel to

and parallel to b x and b a b t and b 2 it is perpendicular to

Since plane

is is

biXb 2 which
is

perpendicular to

r-a x any
,

line in the plane.

V
or or

dot product of

two perpendicular vectors

zero,

[fbibjdOi^bjs]

......... (2)
)

the required equation of the plane and it will pass through the point a 2 if [a a b 1 b 2 ]=L a it i^2] which we know is true by virtue of (2). Hence (ty represents the

Above

is

required plane.

Note

By

also write its

considering the equation as

plane

through a 2

we

could

rrihb.Ma.bjb.l.

Corresponding cartesian form.


[See author's Solid

Geometry P.

125]

Let
i, j,

in

terms of any three non-coplanar unit vectors

along the axes,

and
and
so that the two lines are r=sa1 +/b 1 and

r=a a -f /b a

The
Taking
rsss*i-fjj+k,
the

Plane and Sphere


cartesian

177

corresponding

form

where

we

get their equations as

(Cor, 3 P. 48)

The
where

condition for coplanar lines

is

a 1 =-a 2 =(Ar 1

#a ) i+(^i

^2)

+ (i

2)

k-

The above

condition

means that
[Cor. 4 P. 138]

ijk]^0 but equal to

1,

/.

the required condition

is

Also the equation of the plane containing them

is

(r-a^
or

(b 1

xb )=0
2

or

(r-aa).^;
=0

or

=0.

Mr. N. Sreekantr

O.U.

11.

Shortest distance between two non-intersect-

ing lines, Let the equations of the ?i and t

(Agra
lines

37, 39, 42, 52)

be

178

Vector Analysis

Let
distance
lines.

P& be the
P^
is

shortest

__

At

between the given


perpendiit is

Since

cular to both
parallel

b2 and b 2

to

bjXb^n
is n.
i.e.

say
a 2 be

A2

fig.

No 68

whose module
Let

Al

a x and

A2

i.e.

any two points on the


is

given lines respectively, then shortest distance a 2 on tion of AtAz on PiP& i.e. projection of a x
/aj
a.j) .

the projec-

(aj

a 2^

(b,

Thus

in

order to find the


parallel

S.

D. between two nonto bj


;

intersecting

lines

respectively

and b 2 you
,

should find a point on each of the lines then find th jection of the line joining these points on b 1 xb.2
,

pro-

Equation of the shortest distance.

The

equation of the shortest distance

is

the line of

intersection of the

planes through the given lines and the

shortest distance.

The plane containing parallel to b, X b 2 is


(r-aO
-

t^3L 1 +tb 1 and

S.

D. which

is

f^xfoxb^-O.

2 III P. 161]
is

The

plane containing r=sa 2 +.sb 2 and S. D.

(r-a^.foxfoxbjM-O.
Note.
In case
is

2 III P. 164].

the line be coplanar, then S. D. beta

ween them
or

zero and as such (aj


.

aa

(bj X b 2 )=a 2

(b x

x b 2 )=0

or

as found in

10.

Corresponding cartesian form.


Resolving in terms of unit vectors
i, j

and k as

in

10,

The
S
'

Plane and Sphere

179

b2]
b x xb 2

(b,

bjf-W sin

0-

W
I

(1)

cos

.'.

b x xb 2

Alternative.

bX b
X b2 b!Xb 2

On
planes.

putting for

in
is

The equation

of the S.

D.

<we get S. D. the line of intersection of


(1),

.e.

-*i
- a2

and
i.e.

bi b2

b 1 xb 2 j=0
biXbaJasO,
<;-

jr-j^

=0

m,
112/1

and

Ex.

7.

77z* shortest

distances

between a
a,
b,
c

diagonal of a rect-

angular parallelepiped

whose sides are

and the edges not

180
meeting
it

Vector Analysis
are
be
ca

ab

(Agra 33, 60)


[See Author's Solid Geometry]

Let the unit vectors along

OA OB and OC
9

be

i, j

and

k.

Now

CMssfl,

OBssb and
are
ai, b]

OC=c

and therefore position

vectors of A,

B an i C

and ck respectively.

Fig. No
7

89
the

Now we have to find the shortest distance between diagonal EC and the edge OB which does not meet it.

Shortest distance

is

i~h~~h'~1

where a and

a' are

points on each of the lines and of the lines.

b and
on

b' are the directions

aa0, a point
and
Also

OB

a'=ck, a point

C on EC.

bxb'==05xC*jX(ck-fli-6j)
k,

V jxk=iandixj=-jxi=k.

bxb'
:.

S.D.=

The
abc

Plane and Sphere

181

Similarly

we can

find the S.

D. between other diagonals

and the edges.


Exercises

Ex.
r

1.

Prove that the equation of the plane through the point


to each

(1, 2, 3)
.

and perpendicular

of the planes
.

. (2i+3]+4k)=0 is r (i+6}+5k)=28. n be the normal to the required plane, it is perpendicular to both b and c, the normals of other given planes, and hence parallel to b and c. If a be the given point, then the (r- a) (b X c)=0 [Cor. 2 P. 159] plane is

(i

+ j+k)=3 and r
If

or

Ex.
(1 9 2,

2.

[rbc]=[abc], Find the equation of the plane through

the point

1)

and perpendicular
r
.

to the

line

of intersection ofplanes

(3i- j+k) = J and

(i+4j-2k)=2.
is

The
Ex.

given point 5 P. 173 the line

i4-2j
of

k=a

say
is

and
to

from
to

intersection

parallel

-tH+7j + 13k=n
plane
its

say and it being perpendicular n=0. is given by (r a) equation

the

Ans.
Ex.
r
.

r.(2i-7j + 13k)-l.
Find
the r
.

3.

line

of

intersection

of

the

planes

(i-|-3j_k)=0 and
containing

(j+2k)=0 and
line

hence find the equation


the

of the plane
9

the above

and through

point

(~1 -1,-1). The line of


since
line

intersection

is

parallel to

(i

+3 j_k)x(j+2k)=7i-2j+k
planes pass through origin, and hence the

both the
also

passes through origin.


i. e.

The

plane also

passes

through (-1, -1, -1),

-i-j-k.

Now
Ans.

use case V. P. 164.

r.(i+2j-8k)*0.

182

Vector Analysis

Ex.
*

4.

Prove

that

the

planes

(2i+5j+3k)=0 and

(i- j-Hk)=2

intersect in the line

and hence show that

the given planes

and the plane

have a common line of inter setion. Hint. Prove that the line

of

intersection lies in the

a point on plane perpendicular to the normal.


third
i.

e.

it

satisfies

the

plane and
the

it is

Ex.
Show

5.

Prove that the line of intersection

of

planes

r.(i+2j4-3k)=0
that

and r
is
j.

(3i+2j+k)=0
inclined to
i

is

rf (i-2j+k).
k
and makes an

the

line

equally

and

1 angle \ sec"

3 with

(Agra
line

55, 61;

Utkal 53)

Angle between

and

is

given by
. .

(i-2j+k)
or

= ab cos 0= v'(l+4+l) 1 cos -2=V6cos0; ^ C os0=-2/\/6. cos 20=2 cos 2 0-1=2 f-l=i /. sec 20=3 or 0= sec- 1 3 etc. ;.
.

Ex.6.
the
line

Prove that the plane through the point a paralltl

to
is

r=b+/c

and

perpendicular

to

plane

t.n=q

[rnc]
If

[anc].

to both

n x be the normal to the plane, then it is perpendicular n and c and hence parallel to nxc and therefore
is (r

the required plane

a)

(nxc)=0.

Ex.

7.

parallel to the

Ans.
Ex.
the line

of the line through the point a and perpendicular to line ==b-f tc. n=# r=a-f-/nxc or (r-a)X(nxc)=0.
the
.

Find

equation

plane r

8.

Find

the equation

of

the

plane which passes through


.

of

intersection

of

the planes

ii,**^, r

n 2 =? 2

ana>

*5

parallel to the line

of

intersection

of the planes
,

r.n 8 =08
Any

r.n 4 =04

plane through the line of

intersection of given

The
planes or
is
(r
.

Plane and Sphere

183

%-^+A (r . n 2 -02)=0 *(!!!+ An,) saft+Aj, (1) Above plane is parallel to n3 Xn 4 i. e, the line of intersection of r n 3 =0 3 and r n 4 =0 4 and hence perpendi.
.
.

cular to normal.
/.
tij
.

(iij+An^

(n s

X n 4 )=*0,
.

(n 3 xn 4 )= -A

{(n 2

(n 3

xn 4 )}

J W*]-A.
[WUl
r

Hence
(r

the required plane


.

is
.

nj-ft) [n 2 n 3n 4 ]=(r
Find
the equation

n,- j,) [njn 3 n 4


the plane

].

Ex.

9. (a)

of

which contains two

parallel lines

r=a-f /
II

b,

r==c+/

b.
is

Clearly plane passes through a and

parallel

to

andb.

[Case

158]

Ans. r . {(a-c)xb}+[acb]=0. Ex. 9. (b) Find the equation of the plane which
line

contains

the

r=f a and

is

perpendicular to the plane containing

r=j b

snd

r=

c.

plane containing r=^ b and * =k c will be perpendicular to bxc. The required plane being perpendicular to above plane is therefore parallel to bxc. Also it Hence it is perpendicontains r~t a, i. e. it is parallel to a.

The

cular to

ax (bxc).
is

Since
f
.

equation

passes through origin; hence a X (b X c)=0.


it

it

Ex.
intersect ,

10.

Prove that

the lines

rxabxa and rxbaxb


and find
first their point

(Pb. 60)

of intersection.
i.e.

The

line is

(r-b)XasO
its

a line through
is

and parallel

to a

and

parametric equation
is

fssb-H

a.

Similarly the 2nd line

r=a+j
a,
.

b.

They

will intersect
.

if

[a-b,
a

b]=0,

(a-b)

(axb)0,

(axb)-b . (axb)0,

184

Vector Analysis
is

which

true

as scalar triple product

is

zero

when two

vectors are equal.

For
value of
in

their point of intersection


r for

their

we should have identical which on comparing the coefficients of a and b = equations w e have 1=^ and l f and we get the
r

required point as

a+b.

Ex.

11.

Prove that the lines

r=a-f/(bxc) and
will intersect ij a
.

t=b+s

(ex a)
of four points
the equations

c=b
c,

c.

Ex.
relative to

12.

//a, b,

be the position vectors


interpret geometrically

an origin 0, then

DC

(Pb. 60) (c-d)x(axb)=0, (c-d).(axb)=0. of plane OAB isr,(axb)=0. Also Clearly equation Cross product of two vectors is zero when is c d.

they are parallel.


.'.

CD

is

parallel

to

axb

which

is

normal

to plane

OAB.

Hence

CD is
is
is

normal
is

to plane

OAB.
vectors are perpendicular. it lies in the i. e.

Dot product
Therefore CD
plane OAB. or

zero

when

perpendicular to normal, parallel to it.

Ex.
of
the

13.

Prove that the locus of a point such that the difference

squares

of its distances from two given points

is constant is

plane perpendicular to the lines joining the points.

(Agra 43)

Ex.14.
point a and

Find

the

equation

of the

straight lines through the

intersecting both the lines

r=c+/ d

and r=c'+/'

d'.

Let the equation


and the given

of the given line

(Agra 46, 55, 61, be

Delhi 51)
(1)

r=a+$b
lines are

(2)
'

(3)

The Plane and Sphere


Since
(1)

185

and and

(2) intersect,
(3)

we have
/.
.

[b d

c-a]=0

or

c'-a]=0 b . {dx(c-a)J0 and b (d'x(c'-a)}~0. Above relation shows that b is perpendicular to both dx(c-a)and [d'x(c'-a)]. bis parallel to [d x (c - a)] x [d' x (c' - a)] /. Hence required line is
Again
(I)
<w

also intersect

[b d'

r-a+t[{dx(c-a)}x{d'x(c'-a)}].
(b)

Find

the

straight
asssfl,

line,

through the point

c,

which

is

parallel to the

plane r

and

intersects the line

r-a'=s/b.

(Agra 58)
Let the
line

be

rc+/d
.

passing through
i.

c.

It is parallel to r

a=sO
.'.

e.

JL to

normal

a.

d.a=0 ............ (1)


or
is

It intersects
.'.

r=a'+rt>.
f

[dba -c]-0
(2)

[bx(a'-c)]=0

...(2)

(1)

and

show that d

perpendicular to

both a and

bx(a'-c) and hence


/.

line is

ax{(bx(a'-c)}. r=c4-*[ax{bx(a'-c)}].
the

parallel to

Ex. 15

Find

point

of
,

intersection
.
.

of the planes

t . ==&, r . n a =^ 2 r n 3 =$ 3 n lt n 2 n3 are three given non-coplanar vectors i.e. [n 1 n 2n 3 ]7^0. Since n 1? n 2 n 3 are three given non-coplanar vectors, ntXftg, n 2 xn s n 3 xn! are also non-coplanar Ex, 5. (2) P. 151 and we know that any vector can be expressed as a linear

where

combination of any three non-coplanar vectors. Let the position vector p of the point of intersection be expressed as

where

/,

m, n are to be found.

Now p
is

satisfies the

equations
triple

of the three

given planes and also noting that scalar


vectors are equal
zero.

product

when two

or

186

Vector Analysis
Similarly J

m=
A

and ns=Y
is at

~
a

?.

Hence p

is etc.

Ex.

16.

variable

plane

constant

distance

p from

the origin and

meets the

axes in A,
to

C planes
the locus

are

drawn

parallel

Through A, B and Prove that the coordinate planes.

and C.

of their point of

intersection is given by

Let

[See Author's Solid Geometry, Q, 7 (b) P. 61] the equation to the plane be
r
.

n=q

where /?=

...... (1)

and

let

n be
on the axis
q
i
.

Its intercept

of x
q

is

n~~i
i;

/.

A= ny

similarly "

B= nq
i.e.

and

C=-~
n3

k.

Now

any plane through A,

and

parallel

to j-k

plane whose normal will be along


(r

is
.

given by
.

a)

n=0
.

or
.

n=a n
.

or

1=

q-

or

1=

Similarly

planes

through B,

parallel

to k-i-plane

and

plane through

parallel to i-j-plane are

x,y,z be the coordinates of the point of intersection then xi+y)+zk will satisfy the above three planes.
if

Now

The
Similarly jy=~* '
-

Plane and Sphere

187

(1).

Ex.
the origin

17.

^4

variable plane

is at

constant

distance

p from
.

and meets the axes

in A,

and C.
is

of

the centroid

of the tetrahedron

OABC

Show that the locus 2 *- +Jr2 +C~ 2 ==</6/;- 2


(a) P. 60]

If

.v,

}\

[See Author's Solid Geometry, Q. 7 z be the centroid, then

Equating i, j, Ex. 18. Find


the three planes

k, etc.

we

get

#,

y,

and

etc.
is equidistant

the

locus of a point which

from
51)

11!=^,

P 2 =? 2 r
,

n 3 =2 3
S

(Lucknow

Ans.
Wj

"2

Ex.
lines;

19.

0^,
2

OB

and

OC

are

three

mutually perpendicular

is the

length of

the perpendicular from

to the

plane

ABC;

show
is

that
2

2 2 2 /r =fl- -|-- +r2

and

the

area of the
lengths

triangle

W(b

c*+c a*+a*b*),

a, b, c being

the

of OA,

ABC OB and

OC.

Equation of the plane through a, b, c is r , n=[abc] where n=bxc-f cxa + axb

and

its

vertices are a,

module being twice the area of the triangle whose b and c and perpendicular from is
[abc]

TnT
Here a = <d,
Ex.20.

b6j
b
zero.

and c=dk
the

etc.

Prove

that

four points

4i+5j+k,

-j-k

3i+9j+4k

and

-i-fj+k
d,

are coplanar.

Find a
triple product

d,
is

c-d

and prove that

their scalar

188

Vector Analysis

Ex.21.
from

Prove that

the perpendicular

distance

of a point

the plane through the points a,

b and c

is

[abc] -[abd]+[acd]- [bcdj-Mb

X c+c X a+a x b).


c,

Volume

of tetrahedron

whose

vertices are a, b,

is

J [(abc)-(abd)+(acd)-(bcd)]=:J area of through a, b, c X perpendicular distance of the point d from the plane

through
.*.

a,

b,

c=J
Prove

(bXc+cXa+axb)x/>.
the
shortest
is

/>=as given
22.

etc.

Ex.
edges of a

that

distance
to

between

opposite

regular

tetrahedron

equal

half the

diagonal of the

square described on an edge.

(Agra

50, 59)

We
in a cube.

know

that a regular tetrahedron can be inscribed

N
Fig No 97

Let the unit vectors along OA, OB,


respectively.
/.

OC

be

i, j

and k

OD=i+j+k. BACD
find

is

hedron and we are to

the shortest

the regular tetradistance between

pair of opposite edges say

BC and AD.

and

AD^OD-OA-.

The Plane and Sphere

189

Module

of
is

BC*AD=

-2i

=2.
a point on AD.

Now B

any point on

BC and 4

A AB=OB-OA=}-\.
Required shortest distance
is

the projection of

AB

on

PCX. AD.

:.

S.D..

11 P. 178]

\BCxAD
(j-i).-2i
1,

Now
.*.

ABss
|

i
|

diagonal of the square with one side

AB

.*,

S.D.=l=f =J

the diagonal of square.

Alternative Method,

Let

OABC
and

be the

regular

tetrahedron

of vectors with a,

be the origin b, c the position

vectors of

-4, J5,

and C.

The two

opposite edgs are

OA and BC.
of these are
,

The equations

rfa

and r=(l

The S.D. between them


P
08
|

is

ax(c~b).b
(ax(c-b)|

ax(c-b)

b
d)

aXc-axTT

190

Vector Analysis

A
is

axc^^! OAC
pendicular to the

n x where n x

the

unit

vector

per-

&OAC. A A axb=2^2 OAB n a where n 2 pendicular to the &QAB.


.

is

the

unit vector per-

A
Module
of

A
.

axc-aXb~module

2j 1 n 1 -2J 2 n 2 = V14J 2 (n 1 2 +n 2 2 -2n 1 n t )],


of

as Jjss J 2 being areas of the faces of the regular tetrahedron

where
cos

is

the

0=.
/.

angle

between the two faces and hence


4

module

of

axc-axb=V(B4 2

f)= fo
rr

yo
nxl

...... (2)

TT

Hence

'--T-TVs-?"
*

[from()]

"4"
'

12

half the diagonal of the square described on an edge. Ex. 23. Prove that the S. D. between pairs of opposite edges an isosceles tetrahedron lie along the join of their mid. points and of that the three S.D.'s are perpendicular.
i.e.

Take one vertex as


origin

and the position


a,

vectors of the other be

b and

c respectively.

"

2
s

2
.

Lt5TLa

Ftg.No.MI

The Plane and Sphere


If

191

P(l

perpendicular to both
|

or

if

and

OA and BC then PQ, should be OA and -BC and it will be so if tb+c-a).a=0 andi(b+c-a).(c-b)0 a (1) b+a . c=a2 2 2 c -b =a. c-a. b (2)
is

S.

D. between

Since the tetrahedron

is

isosceles,

we have
.

or or

OA**BC, AB^CO, CA=OB 2 a s=(c-b) or a =b 2 +c 2 -2b


a 2

2b
Similarly

2 c=(b +c -a
2

2
)

$)
(4) (5)

2c 2a

.
.

a=*(c*+a

-b 2

and

Adding
(5)

(4)

and

(5),

2 s b=(a 2 +b -c we get (1) and

subtracting

(4)

and

we get (2). Hence PQ,

is S.

D. between

OA and

BC.
pairs
of opposite

Similarly we can prove for other The three S. D.'s lie along edges.

b+c
and these
will be

a,

c-f

a-b,
if

a+b-c
etc.

perpendicular

if

(b+c-a) . (c+a-b)=0 c 2 -(a-b) 2 =0


2a
Similarly
.

or

if

b=a 2 +b 2

c2 which

is

true

by

(5).

we can

prove other pairs to be perpendicular

by

(H)

and
12.

(4).

General equation of a sphere. (Agra 35, 38, 43, 52, 60)


the centre the
of

If

C be

then by definition,

a sphere, distance of
of

any

point

on

the

surface
is

the

sphere from the


radius
a.

.centre

equal to

Let the position vector of any be r with point P on the surface


respect
to an
origin

and that

of

192

Vector Analysis

the centre be
c, i.e.

OP^t

and 0(7

then

Now

module of
is

CP= radius
its
.'.

and we

know

that

square of a vector

square of

module.

2 2 (r-c) =fl

or

-2r.c+c2 -<z2

......... (1) V c 2 =*c*.


c

We may

for the sake of convenience put

-fl 2 =fc

and

the above equation

becomes
r2

-2r.c+*=0
is

......... (2)
of

Above

is

a relation between the position vector

any

point on the surface of the sphere and of the sphere.

called the equation

Corresponding cartesian form.


If

be the point

(#, y, z)
i,
j

and

(c l9 c. Zy 3 ),

then resolving

in

terms of unit vectors

and

k,

and

Squaring both

sides,

we

get
r3) a

CP

2
fl
|

which

is

the well
is

known

whose

centre

at

(cly c29 cz )

cartesian equation of the sphere and radius is a.

Particular case,
1.

In case the origin of vectors lies on the sphere.

The

Plane and Sphere

193

In
i. e.

this case the

module
from

of

OC

will

be equal to radius

coma and hence

(1),

we

get the equation of the

sphere as

V
Polar form.
C.
(3)

-2r.c=0 ............ (3) *c2 -a 2 a2 =a2 =0.


r a

s=r 2

and

c=rc cos 0=r0 cos


or

6
is

gives r*-2rfl cos 0=*0

r=20

cos 6

the

required polar form.

Cartesian form,
Putting the
i, j,

values of
(3),

and c

in

terms of unit vectors

k,

we

get from

or

* i +J 1 +* i

which represents a sphere whose centre which passes through origin.


2. If

is

at

(c l9

c2

3)

and

the centre
is

is at

the origin.

In this case

at the point

C and hence
r 2 s=a 2 .

clearly

the

equation of the sphere takes the


or

form

(r-a).(r+a)*0 ............ (4)

-*-V
positon vector of

is

a,

and we know

dot product of two vectors be zero, then they are


that
if

perpendicular.

Thus AP is perpendicular
riglit

to BP, showing that

diameter of a sphere subtends a

angle at the surface.

191

Vector Analysis

General

Method

for

above property.

(Agra 52)

Let the equation


r2

of the sphere be

-2r

c+/i=0 where

k*=c z ~a z

...... (5)

If

CM=a,

then

Above

gives us the posi-

tion vectors of
If r

A and

B.

be the position vector of any point

Pon

the circum-

ference, then

_ _* _ ^P-= OP- 0.4 = r - (c+ a)

->->->
and
-

J3jPOP-OBt-(c-a).
-

Now
or

4P, BP=(r-c-a) . (r-c+a) 2 2 2 2 r 2 -9r or (r-c) -a c-fc ~0


.

or

r2

2r

c+fc which

is

zero by

(5).

perpendicular to BP, showing that a diameter subtends a right angle at the circumference.
is

Hence AP

Cor.

Equation of a sphere on the join of two


(Benaras 48, 54)

given points as diameter.


If

g and h be the position vectors of the extremities G and of any diameter and P any point on the surface whose position vector is r, then Z.GPH=7r/2, i. e. GP is

perpendicular to
.%

HP.
is

dot product of

GP and GH

zero

i.e.

(r-g).(r-h)0.

The
Above
13.

Plane and Sphere

195

represents the required equation of the sphere. Points of intersection of a line and a sphere.

Let the equation of


the sphere be

F
where

(r)=r

2r

c+A;=0
....(1)

k*=* c

a 2.

Let
through

the

line

pass

whose

point position vector


a parallel

the

P
is

A
p and be
to unit vector q, so that its equation
is

A r=p+*q
t

(2)

[6,

P. 46]

will

stand for the distance of the point


line.

P from any

point on the

In order to find the points of


eliminate
r

between
is

(1)

and
get,

(2)

unit vector
(2) in (1),

unity,

we

intersection we have to and noting that square of a on putting the value of r from

A
A
or

t*+2t q
*

or

A +2q
*
2

(p-c)

A
or

+2q.(p-c)/+F(p)=0

(3)

cuts the surface of sphere in

Above being a quadratic in / shows that every line two points which will be real if

A
[q
.

(p-c)]*

> F (p),

t. e.

Ba

>

4 AC.

196
If Q,

Vector Analysis

and

of the above equation

are the points of intersection, then the roots (3) will be the values of distances of

from
;.

Q and
PQ
.

/?, i.e.

PQ

and PR.
of

P/?= Product

roots^F

(p).

Above
lines

result

dees not depend upon the direction of

PQ
the

q showing thereby P/J=F (p) for all lines drawn in any direction through point Pto cut the surface of the sphere.
it

as

is

independent

of

Square of tangent from any point. and R coincide at any point T, then PQR () becomes tangent line PT and both P) and PR become PT.
Cor. 2.
In

case

:.

PT.PT=F(p)

or

PT 2

F(p).
to the

Thus square of the tangent from any point


sphere is obtained by
sphere, which
is

surface of a

substituting that point in the


that
is

equation of the

a result

identical

with

the corresponding

result of coordinate geometry.

Cor.

3.

Tangent plane

at

Author's Coordinate Solid Geometry.]

given point. The same procedure

[Refer
is

adopt-

ed for finding tangent planes at any point to a conicoid.

Now let us choose that the point P is on the surface of the sphere so that F (p)=0 and then one root of (3) will be If the line through P zero as PQ, will be zero in this case.
is to

be a tangent line then the other root of


for

(3)

should also

be zero, the condition

which

is

q.(p-c)0from
A

(3)

(4)

Now q
line

being the direction of line which

is

now a

tan-

and p-cis the vector joining the centre to the gent point P and since their dot product is zero we conclude that tangent line is perpendicular to the radius through that
point.

The Plane and Sphere


All such

197

tangent lines will

lie

in

a plane whose equa(2)

A
tion is obtained

by eliminating q between
/,

and

(4) (in

solid

geometry by eliminating

m,

w,

i.e.

direction cosines)

and

we

get the equation of the tangent plane as

(t-p).(p-c)-0.

(5)

Above equation clearly represents a plane through the point p whose normal is p c i.e. the line joining the centre and the point of contact.

Now we know
remains unchanged

that
if

(p)=*0

we add F

and hence the equation (5) Thus the (p) in L. H. S.

tangent plane becomes

(*-P).(P-c)+F(p)=0
or or
r
.

p-p

r
r

c+p c+p 2 -2p c+&=0


.

ip-(r+p).c+fc==0
2 replace t by r

(6)

Above
Rule.
change one of

is

the equation of the tangent plane at p.


the

In

equation

of

the sphere

and

the t's

by the given point

change one of the t's by the given point

p and replace 2r by r-f t and p and this rule is identical

with the corresponding rule of coordinate geometry.

Cor. 4. Condition for any plane to be a tangent plane.

We have seen

pendicular to the any plane is a tangent plane then its perpendicular distance from the centre should be equal to radius.

that tangent plane at any point is perradius through that point and as such if

Let the plane be


radius.
..

nj and c be
_,

the centre and a the

/'q-c.nY

5 P. 166]

198

Vector Analysis

Ex.

1.

Find

the

coordinates

of

the

centre

of

the

sphere

inscribed in the tetrahedron bounded by


r
.

the planes

i=0,

j~0,

k=0

and

(i-f-j

+ k)

0.

Also write down the equation of the sphere.

(Benaras 53)

Let

(*, y, z)

be

the coordinates of the centre so that the


is

position vector of the point

xi+yj+^k.

Since the given planes are all tangent planes, therefore, perpendiculars from the centre to all the planes are equal.

"
|T|

111
)
.

(i+j+k)
get

'Since perpendicular distance

is

not

ive,

we

tr

(3-

V 3)

a,,

Hence on putting the values


vector of the centre
is

of x,y

and ^ the position

-~(3~V3)(i+j-fk).

Hence the equation


[t
(b)
(j-

of the sphere

is

(r-c)

=a 8

(8-V3)(i+j+k)J.[-J
r
.

(8-

V8)J.

Prove that the equation of the sphere circumscribing the

tetrahedron of part (a) is

{t-a (i+j+k)}=:(X

The Plane and Sphere


Cor.
5.

199

Condition for orthogonal intersection of two

spheres.
In case the two spheres cut each other orthogonally, then evidently the tangent plane to one of them at their common point of intersection will pass through the centre
of the other.

Hence

the

square of the
to

distance between

their centres should be


radii,
i.e.
if

equal

sum

of the squares of their

the two spheres be


r2

-2r

. CJL

1 4-^=0, where A^ss^-a


2

and
then
or

r2

-2r

c 2 +#2 =0, where


(c1 -c 2
2
)

=c2a -0 2
-*j

=fl 1 2 +^ 2 2

c^+Ca

-^
'Jcj.

a c,-<? 1 -A;1 +f 1

or

Cj^fcj+fcg.
result.

Corresponding cartesian

Two
and
will cut

spheres

xt+y^tf+
one another orthogonally
2

if

2^ + 2zv + 2^2 = d
Ex.
2.

[Refer Author's Solid Geometry P. 213]

The

sphere which cuts

F^tjO and F (r)=0 ortho2

gonally also cuts

(r)-AF fr)=0
2

orthogonally.

(Alld.

M.
2

Sc. 1960,

Agra

38, 46;

Benaras 55)

Let

F1(r)r ^2r.c1 +A;1 =0 F2 (r)^ra - 2r c 2 +A2 =0


.

...... (1) ...... (2)

.\

F^fJ-AFtCr)-^-

2r

Cj

=ta (l-A)-2t . (^(3)

200

Vector Analysis
ra -2r.
(2)

Let the sphere cut the spheres (1) and

c+*=:0

...... (4)

orthogonally.

...... (5) ...... (6)


Multiplying
(6)

by A and subtracting from


.

(5),

we

get

2c
or

(Cj-

Above
cut sphere
14.

is

evidently the condition that the sphere

(4)

may

(3)

orthogonally.

The
The

polar plane.

Defi.

a sphere

is

polar plane of a given point with respect to the locus of points the tangent planes at which

pass through the given point. Let the equation of the sphere be
a F(r)=r -2t
.

c+*=0.
p
is

The
If

tangent plane at any point


r
.

given by

~(

it

passes through a given point d say, then

d.p-(d+p).c+*-0.
The
or
it

locus of point

P is

therefore

...... (1)
can be written as

r.(d-c)(c.d-fc)

......... (2)

Above equation represents a plane which is clearly perpendicular to the line joining the centre and the given
point.

Thus

the polar plane

of a point

is

perpendicular

to the line

joining the centre

oftfo

sphere to that point.

the polar plane of point d cut the line joining then CT is the perpendicular distance the centre to d in

Again

let

The
of c

Plane and Sphere


of

201
is

from the polar plane


c
d)

d as
(d
;

this line

normal to the

polar plane.

c
-

___
""

--5--c d
-~CZ>

s P. 166]

-'-H

CD

where

Z) is the point

whose position vector


/.

is

d.

points with respect to the sphere.

The two
Also

CD.CT^a D and T arc


(1)

2
.

called

inverse points

it is

easy to

prove from

that

if

the polar plane

of a

point d passes through the point

e,

then the polar

plane of e will pass


15.

through the point d.

Radical plane,
plane of any

The

radical

two given spheres

is the

plane which

contains all such points the squares of the tangents from which to the

given spheres are equal.

Let the two spheres he


r2
r
2

-2r.

Cl

+*i=0

-2r.c 2 +/;2

=0

......... (1) ......... (2)

gents from

Let there be a point p such that squares of the tanit to (1) and (2) are equal.
2
2
.'.
.

or

p -2p c 1 +^ 1 =p -2p c 8 +Aj-0 2p . (Cf^) ss &! &,. Above shows that the point p lies on the plane
.

2r.(c 1 -c 2)
Clearly the above plane the line joining the centres,
is

-A 2

......... (3)
c2
,

perpendicular to c t

i.e.

Rule. The radical plane of two given spheres is obtained by subtracting the equations of the spheres as can be seen from (1), (2) and (3).

202

Vector Analysis

Ex.
distances
sphere.

3.

The

locus

of a point

which moves

so

that

its

from two fixed points are in a constant ratio n : 1 is a Prove also that all such spheres , for different values of n,
.

have a common radical plane

Let the middle point

of the line joining the fixed points

A and B of B is

be origin so that position vector of


a.

is

a and

Let the point Pbe


/. /.

r.

-*

,4P=r-a and

P=r+a.
/>

4P2

(r-

a)

and

=(r+a) 2

Also

we

are given that

4J-:.
or,
2

or
.

AP*=n*.BP*.
.

(r 2

-2r a+0 2 )n2 (r 2 +2r


.

(l-H )-2r

or
clearly represents a sphere. n the values w x and 2 we g e t two spheres. Giving
t
2

Above equation

-2r .a

The

radical

plane of the
is

above sphere

is

obtained by

subtracting

them and

2 }+.;j.o l-"2 5

or

r.a^O
which

which

is

independent of
represents

n^

and

w2

Above

the radical
is

plane

passes

through origin which normal is along a.

the middle

point of

AB

and the

Thus

the

radical

plane

bisects

perpendicularly the

distance between the given

points.

The

Plane and Sphere

203

Exercises

Ex.
squares

1.

Prove that the locus

of a point the sum of


is

the

of whose

distances from a given point


is at the

constant is a

sphere whose centre

centroid of the given points.

Ex,
centre

2.

Prove that
sphere

the

distances

of two points from the


to

of a given

are proportional

the

distances of the

points each from the polar plane of the other.

For the sake


of the sphere as r

of convenience let us choose


2

=a
,

points as a t and a 2
r

the equation centre being at origin and the given whose polar planes are
a
,

2 aj=a and

a 2 =fl 2 etc.

Ex.
lines

3.

From any
to

point

on the surface of a sphere, straight

are

drawn

extremities

of any diameter of a concentric


on these lines
is constant.

sphere.

Prove that

sum of
of

the squares

(Agra 38)
Let the centre
the concentric
that the

spheres be the point c so

equations of spheres are


r2

the

outer

and

inner

-2r.c+*!^0
.

(1)
(2)
.

r*_o r

C +A; 2 =0
.

where fc^c2 -^ 2 and fr 2 =c2 -0 2 2 .(2) Let rj be any point on the outer
sphere so that
If

Fig.No.99

.c+fci=0 g and h be extremities of diameter


its

r1 2 -2r1

(3)

of

the inner

sphere, then

equation
r2

is (r

g)

(r

h)=0
(4)

or.

-r.(g+h)+g.h=0
(4),

Comparing

(2)

and

we

get
.

g+h2c and g h=*


Now

(6)

204

Vector Analysis

or

4c-2*i+2(t1 -2r 1

= 4c 2

.c)

2fc2

+ 2*!
of

[from ()]
the

=twice the sum

squares of the

radii and hence constant.

Ex.

4.

straight line

is

drawn from a point


is

to meet
:

fixed sphere in P.
fixed ratio.

In

OP

a point Q, of

taken so that

OP

0(1

is

Prove that

the locus

is

a sphere.

(Agra 35)

Let
be
rj,

be the position vector of Q, on OP, so that P will where n is constant and this point Plies on the
T!

sphere and hence the locus of


/Ex.
cuts the
5.

Q,

i.e. r a is

etc. etc.

plane passes

axcsinA,B,C.

through a fixed point (a, b, c) and Prove by vectors that the locus of the
-

centre oj the sphere

OABC

is

+ + r2(a) P. 165]
let

[See Author's Solid Geometry Q. 4

The

fixed point
2

A=ai+b]i-ck and

the normal to the

plane be ns=w 1 i+

j+

3 k.

The
or
If

equation of the plane through


r
.

is

given by

n=a n

(fl 1

i+flftj+flsk)

s flfti+6"2+M3
on the axis

:ss
<7

say.

x 1 be the intercept

of *,

then

x^i lies

on

the plane.
(7

ni
k.

.%

point
point

is

i.

Similarly

is

-j

and

is

The

is

origin.

The

Plane and Sphere

205

Let the coordinates of the centre be P(x,y, z\


)+zk.
.\

i.e.,

OP=APs=BP=CP=radms

of the sphere

OABC

Equating

their modules,

we

get

xn is
a2

+/+
Q x*=-^.

etc.

or

.23=2*.

q -~

or

Similarly,
"

,-,
<,
<i

*.
3
.

-~
*

+ T+T y
a, b, c

V (^i+K+^ )= ^
q
to the sphere

Ex.

6.

// any tangent plane

makes intercepts

on the axes, prove by vectors that

[See Q. 5 P. 182 of Author's Solid Geometry],

The
hence
its

centre of the sphere is origin 2 2 vector equation is r =d


*

and radius

is

d and

plane r . n=q will be a tangent plane cular from centre is equal to radius

Any

if

perpendi-

U,
The

^|
intercepts
q
.

rf

or

g=nd.

made by
q
""i*, =0,
.'

the plane on the axes are


-A *^v
,

q
9

"

111.
)

.n

.n
\-\

k n

2i

^2

q%

206

Vector Analysis

= -yi

2
[fl

cos 2 a+fl2 cos 2 /3+n 1 ccs 1 y]

=~ V
a,
/?,

cos 2

a+cos 2 0+cos2

y-

being the angles which the normal makes with the


7.

axes.

Ex.
Ex.

The

plane

through the

intersection

of two spheres

is

perpendicular
8.

to the line joining their centres.

The mid.

points

of
r;

the

six

edges of a tetrahedron
:

ABCD
(i)

lies on

a sphere of radius

then prove that

Centre of the sphere

is the centroid

of the tetrahedron.
the

(ii)

The sum of the squares on


2
.

the line joining

centre

to

the vertices

(Hi)
is

of the tetrahedron is 12r The sum of the squares on the pairs of opposite edges
that
these

16r*

and

edges are

perpendicular .

Let the centre of the sphere


be taken as origin and the position vector of the vertices be a,
b, c

and d respectively.

The mid.
(writen
in

points of the edges


of

groups

opposite

edges) are

a+b
2
i. e.
'

c+d
2
'

b+c
2
i. e.
'

+d
2
'

+c
'

b+d
2

"2

P and
r,

Q,

Since these
origin

points

lie

and S i. e. L and on the sphere whose centre

is

and radius

we have

c+d\2

/b+c\

/a+d\ 2 /a+c\2

(1)

We have to prove

that centre of the sphere

is

the

The Plane and Sphere


centroid or

207

we have

to

----prove that

--,=0

as

we

have taken the centre

at the origin.
a

Now
or or
Similarly,

+ b)2 =(c+d) 2 4r 2 (a+b + c+d) (a + b-c-d)=:0


(a
.

(a+b+c+d). iQP-0.
->
(a

+ b+c+d)
->
-

%SR=*Q and

(a+b+c+d)

JAfZ,=0.
perpendicular

Above
to Q, P,

relations

show

that

a+b+c+d

is

are the joins of the middle points of pairs of opposite edges and are non-coplanar vectors. Now if a vector is perpendicular to three non-coplanar vectors,
it

SR and

ML which

then

should be a zero vector.

(2)

Now

have

to

prove that
12r 2
2
.

OA 2 +OB 2 +OC*+OD*
or

+b +c +d
(1),

=a2r
get
2

Adding the relations in

we

T_
2 / (a-l-b\
)

6r

or

3Za +227a

b=*24r 2

. .

.(2)

Nowa+b+c+d=0.
Squaring,
or
/.
(3)
2 2

we

get

^a 2 +227a b=0 ^a 2 =227a b.


.
.

3ra -27a =24f

or

=12r 2 from

(2).

The sum

4B

+CD

on the opposite edges is 2 2 =(b-a) +(d-c) =2:a -2 (a . b+c . d)


of the squares
2

= 12r
Now
or

-2(a.b+c.d)

(3)

from

(1),

aa +2

(a .

b+ c d)8r.
.

208
/.

Vector Analysis
2
(a
.

b+c

d)8r 2 -Za 2 =8r 2 - 12r2 =- -4r 2


2r 2 s=b
.

Again a

b+c

d=
.

c+a d
.

b
i. *.

(a-c)-d

(a~c)=0
-

or

(a-c)

(b-d)=0

CA.DB.
Ex.
9.

Hence proved.

Prove

that any
is

to intersect

a sphere

drawn from a point cut harmonically by the surface and the polar
straight line

plane of 0.

(Agra 53,60)
2 2r . c+/c=0 and sphere be r be taken as origin. Therefore any line through

Let the equation


the point

to the

A
is

r=/ b where
0.
Its

stands for the distance of any point on


intersection with

it

from

points of

the sphere are

given by
*

A -2b
/

A
.

c t+k*=Q',
in

V
Q,,

b2

l.

13 P. 195]
are

If

it

meets the sphere


of

P and

then

OP and OQ
A

the

two values

given by above
1

JL

l
/

JL

l
/

_'lft_ 2b ""
""

-? ......
.

m u;

Again polar plane

of

w.r.t. the

sphere

is r
[

c=sk.

14 P. 200]

A
Again
t
if

A
cuts this plane in
-R,

r =*fb

then b

c=A; where

now

stands for OR.

A 2b.c

OR=T
*

op+da

[from

nxl (1)]

j_ OP

i
1

OR

are

OQ,

The Plane and Sphere


or

209

OP, OR,

OQ

are in H.P.

Hence
16.

the

line is cut

harmonically by the surface and

the polar plane of 0.

Volume

of a tetrahedron.
vec-

Let the
tors

position

of

the three

cotermi-

nous edges OA,


of the
a,

OB and OC tetrahedron OABC be


c
respectively

b,

with

respect to origin.

Now we
volume
?f

know

that
is

of

a tetrahedron

area of base

OBCx height
~

of

A from
Area

the plane of base.


of

&OBC %bxc (Ex.


of

6,

P. 128) which represents

a vector perpendicular to the plane of


/.

&OBC.
.

volume

tetrahedron=

(ibxc)

5, P. 130 that the volume Again we know from parallelepiped whose three coterminous edges are a,
is [abc].
.'.

of a
b, c

volume
1.

of

tetrahedron=
in

volume

of paiallelo-

piped.
Cor.
of

Volume of tetrahedron
which

terms oj position vectors

the four vertices, neither oj

is at origin.

We

have seen that when one

of the vertices
is

is

at the

origin, the

volume

of the tetrahedron

Let the position vectors

of the four points A, B,

C and

210

Vector Analysis
a, b,

O be

c and d with respect to any origin 0'.

(Agra 39, 51)

Similarly OJ3*=:b-d and


/.

OC=c-d.

volume

of tetrahedron
f

-i.[a-d b-d, c-d]


-J.(a-d).[(b-d)X(c-d)].

I/^ffta-dMbXc-dxc-bXd}],
.
.

V dxd=0
.

Scalar triple
equal.

=i[a (bxc)-a.(dxc)-a (bxd)-d (bxc)]. product is zero when two vectors are
(Cor. 2, P. 137)

=
V
'

{[abc]-[abd]+[acd]-[bcd]}.

ber

quite convenient to rememd and strike one letter from the a, b, c, end and then the next and soon. Then form the scalar
:
;

Rule
i.e.

. (d x c)=a The above form is

-a

(c

X d)[acd].

write

left and connect and ive signs, alternately +ive Cor. 2. Condition for any four points to be coplanar. In case the four points are coplanar, then the volume of the tetrahedron should be zero or otherwise a- d, b-d, c d are coplanar, i.e. [a d, b d, c d]=0 which when

triple

product of the three vectors thus

them with

expanded reduces to
[abc]

- [abd]+[acd] - [bcd]-0.
of

Cor. 3.

Volume
r),

tetrahedron

in

terms

of the

coordinates of the vertices*

(Agra 48)
2, 3,

Let
the
four

(x r ,yr ,

where r=l,
the

4 be the coordinates of

vertices of

tetrahedron so that the position


i, j

vectors of the four points in terms of unit vectors a *ii+Jii+tk etc,, are

and

The Plane and Sphere


so that

211

Now
or

the

volume

of the tetrahedron

is

(a-d, b-d, c-d]

F=~

[Cot. 4 P. 1381

The
make
it

value of this determinant remains unchanged

if

we
and

of fourth order
1, 0, 0,
1

one column of

by adding one row of as shown below


*4

1, #4,^4,

^4
J'l-J'i

*4

*1~*4
Ar

*l-*4

-X 4

^-^4

^-^4
^3-^4
to 2nd,

^3~^4

^3^4
1st

Adding the elements


rows,

of

row

3rd and 4th

we

get

JL

Xi
*2
*3

Exercise

Ex.

1.

Prove that the volume


.

of a tetrahedron bounded

by the

four planes r
r
.

(mj+k}=0,

(flk+/i)

0, r

(li+m])=0 and

[Q, 7 P. 80 of Author's Solid Geometry]

(Agra

39, 45, 59;

Benaras 54; Lucknow 52)

212

Vector Analysis

Let us
planes are

find the

points of intersection of the four planes


time.

taken three

at
(i)

The
(ii)

cartesian equations

of the

m^+w^O,
first

ws+/#=0,

(iii)

lx+my=*Q

and
find

(iv)

lx+ny+nz**p.
three
intersect at origin.

Clearly the
the

Now
(iv).

let

us
it

be

(x, y, z)

point of intersection of (i), (ii) so that the position vector

and
of

Let
point

this

is

Adding
or

(i)

and

(ii),

we

get

p+nzQ
.

[from

(iv)];

.'.

z= -p\n.

my*=*p

or

y^pjm
#=/>//

[from

(i)l
(ii)].

and

lx=sp

or

[from

Hence
Similarly (iv) arc

--

^n

k=c (say).
(i),

points of

intersection of

(iii),

(iv)

and

(ii)

(iii),

(say,

and

Now

the
is

volume
| [abc]
pirn

of a tetrahedron

whose one vertex


1

is

at the origin

-Pll
pll

pin
pin

p* 1-1

-pirn

I
1

-1
1

pll

pirn

-p]n

-1
6
s

Ex.
mutual

2.

Pwtf
in

^ following
of lengths

formula for
of

the volume

Vofa
their

tetrahedron

terms

three concurrent edges

and

inclinations.

[Refer Author's Solid Geometry P. 75]

The

Plane and Sphere


1
COS
COS

213

F2 =

36
COS
<$>

COS

COS

Jl

cos

(Agta

57,

Luck. 55)
and

Let the three concurent edges concur at origin


the position vectors of the other vertices A, B,

be

Again a

b^a^+Ji^+^a^ cos
c=J!7# 2 *8=i

b
c

cos

0,

a sasZxiXjssca cos 0.

/I

a2

ab cos
6*

ac cos
be cos

ab cos
ac cos

<f>

fa cos 6

c*

214
abc

Vectot Analysis
a
a cos ^ cos
a*b*c*
3(3

b cos
b

<f>

cos

"36
c

cos

b cos B

COS

<

COS

cos
cos

<

cos e
1
centroids of (he

cos

Ex.

3.

Gj,

GJJ,

G 8 0r0

/fo

triangular faces

OBC, OCA, OAB of a


of the tetrahedron
constructed uith

tetrahedron

OABC

OG lf OG 2

Prove that the volume is to the volume of the parallelepiped and OG 3 as coterminous edges as 9 : 4.

OABC.

Ex.4.
Let

Prove that

each

of the four faces


b, c

of a terahedron

subtends the same volume at the centroid.

the centroid of
/.

a,

and d be taken as

origin.

a+b+c+d=0

......... (1)

Volume of tetrahedron GABC, G being origin=i-[abc]. Volume of tetrahedron GBCD= J [bed]


.

(cxd)=b {cx(-a-b-c)} from -ib (cxa)


.
.

(1)

-*[abc].
Ex.
5.

In tetrahedron
J
.

given by the formula


distance between

OABC prove that the AB OC p sin 6 where p is


.

volume
the

is

shortest

AB

and OC.

From
that
is

the figure
parallel to

we observe

AB
/.

is

b-a

and

OC

parallel to c.

shortest distance

is

paral-

lel

to

(b-a)xc.
is

Also a
is

a point on

OA and
is

c
A/o.98

a point on OC.
:.

shortest distance

projec-

The Plane and Sphere


tion of

215
[

a-c on (b-a)Xc.
be the shortest distance, then

11 P. 177]

If p

(a-c).

(bxc-axc) AB OC sin
.

a .(bxc)

~AB OC
.

sin

'

V
equal.

scalar triple product vanishes

when two

vectors are

"
/.

AB.OC
.

sinO

F= J
that

[abc]fclB
the

OC

/>

sin

0.

Ex.
vertex
is

volume of a pyramid of which the a given point (x,y,z) and the base a triangle formed by
6.

Show

joining

the

points
is

(a, 0, 0), (0, b, 0)

and

(0, 0, c) in rectangular

coordinates

(Agra 47) that a triangular pyramid is a tetrahedron. In terms of the unit vectors the given points are ai, b}, ck and xi+y]+zk, say vectors A, B, C and D.

We

know

V=l\A-D
C-- D

BD

C-D\

[Cor.

IP.

188]

*i-j1+(*-)k;

-x

b-y

-z
-

--\abc

I-*/*

-j/ft

l-y\b

216

Vector Analysis

Adding column nos.

1, 2,

and

3,

we

get

abc (1 - */a -y\b -

zjc)

=f
Ex.

afo;

(!-*/ -.?/*-

7.
is

G w

the

centroid

of

the

tetraltedron

OABC

O'A'B'C
are
all

another tetrahedron such that 00', AA',

BB' and CC'


of the

bisected at

G;

show that

is

also

the

centroid

tetrahedron

O'A'B'C'.

AGRA UNIVERSITY SOLVED PAPERS


195* Find the vector equation of a straight line passing through two given points. Prove by vectorial methods that
1.

the following are coricurent the bisectors of the angles of a triangle (a)
:

and

(b)

the

medians of a
(a)
2.

triangle.

Cor. 2 P. 48. (b) Ex. I P. 52, Ex* 2 P. 53. Find the vector equation of a sphere. (a)
(i)

that any diameter of a sphere right angle at a point on the surface.


(b)
(ii)

Show

subtends a

Prove
of

vertices

that if a point is a right-angled triangle,


is

its

equidistant from the join to the middle

point of the hypetenuse


triangle,

perpendicular to the plane of the

(a)

12 P. 191.
(i)

(b)

See general

method

P. 194. (ii)Q,.
straight

n (b) P.

HI.

3. (a)

cular to
(b)

Obtain the equation of a two non-intersecting lines.


Prove that the locus
lines
of

line

perpend'-

the middle
fixed

points of all

straight

terminated
is

by

two

non-intersecting

straight lines

a plane

bisecting

their

common

perpendi-

cular at right-angles. ii P. 177. (a)

1953
1. (a)

Define 'Centroid'.

Show

that

the centroid

is

independent of the origin of vectors.


(b)

Prove that the

lines joining

the vertices of a tetra-

hedron to the centroids of area of the opposite faces are


concurrent.

218
(a)

Vector Analysis
2 P. 37, 4 P. 40. 6. P. 59.
In a tetrahedron,
if

(b)
2. (a)

Ex.

two

are perpendicular, prove that the pendicular to each other, and the

pairs of opposite edges third pair are also per-

sum

of

the squares on

two opposite edges is the same for each pair. Prove that any straight line drawn from a point (b) intersect a sphere is cut harmonically by the surface to and the polar plane of 0.
2. (a)
3.
(i)
(ii)

Ex. i P. 104, (b) Ex. 19 P. 208. Establish the following relations


:
.

ax(bXc)+bX(cXa)+cX(aXb)=0. a faXb bxc cxa]=[abc]


(bxc) . (aXd)+(cxa) See bottom P. 142. Cor. P. 146.
Q,. ii P. 154.
.

(iii)

(bxd)+(aXb)

(cxd)0.

(i)

(ii)

(iii)

1954
1. (a)

Prove the following by vector methods

The

internal

bisectors of the angles of a triangle

are concurrent.
(b)

ABCD

is

parallelogram and

the
for

point

of

intersection of
(not necessarily

the diagonals.
in the

Show

that

any

origin

plane of the figure) the sum of the vectors of the vertices is equal to four times that position
of 0.
(a)
2.
(a)

Ex. 2 P. 53,

(b)

Just as

Q,. 15 (b) P. 30.


:

Give vectorial solutions of the following

The

area of the triangle formed by joining the


of

middle point of one of the non-parallel sides of a trapezium


to the

extremities

the

opposite side

is

half that of the

trapezium.

Agra University Solved Papers


(b)

219

Find the coordinates

of

the centre of the sphere

inscribed in

r
(a)

the tetrahedron bounded by the planes i0, r . j =0, r . k=0 and r . (i+j+k)=a.

Ex. 12 P. 132,

(b)

Ex.

P. 198.

Q,. 10 P. 154, (b) Ex. 2 P. 147.

1.

(a)

1955 Prove the following by vector methods The medians of a triangle are concurrent.
:

four diagonals of a parallelepiped, and the joins of the mid. points of opposite edges, are concurrent at a common point of bisection.
(b)
(c)

The

The
Ex.
i

three points

whose position vectors are

a, b,

o'a

- 2b
(a)
(c)

are collinear.

P, 52, (b) Ex. 7 P. 61.


77line

Q.26,(b)P.

2. (a)

Find by vector method, the equation of the

of intersection of
(b)

Show
r

two planes. that the line of intersection of


.

(i+2j+3k)=0 and r
i

(3i+2j+k)=0
sec"" 1

is

equally inclined to
j.

and k, and makes an angle J

with

(a)
3. (a)

8 P. 171, (b) Ex. 5 P. 182. Prove the relation

ax{bx(cXd)}=b
and hence expand

daXc-b caxd,
.

ax[bx{cx(dxe)}].

szu

veciur analysis
(b)

Find the equation

of the straight line

through the

ooint c, intersecting both the lines

r-a=A and

r-a'=fb'.

(a) ft. 10 P. 154, (b)

Ex. 14 P. 184.
:

1. (a)
(i)

1956 Prove the following by vector methods


:

4BC
ingle

The internal bisector of the angle A of a triangle divides the side BC in the ratio AB AC. The
join
of the

(ii)

mid. points of

two
of

sides of a

tri-

is

parallel to the third side,

and

of half its length.

(b)

What

is

the

vector

equation

the straight line

hrough the points i-2j+k and 8k- 2j ? Find where the line cuts the plane through the origin
ind the points 4j
(a) (i) Ex. 3 P. 57, (b) Q..2 P. go.
2. (a)

and 2i+k. (ii) Ex. 19 P. 72.

Find the equations of the planes bisecting the ingles between the two given planes r . n=*q and r n'==<?'.
.

(b)

ascribed in the tetrahedron

Find the coordinates of the centre of the sphere bounded by the planes
i=0, r j=0, r . 6 P. 169, (b) Ex.
.

r
(a)

k0 and r
i

(i+j+k)=a,

P. 198.

3. (a)

Prove that

[lmn]][abc]=

l.a

l.b

l.c

m
(b)

m .b
n .b

m .c
n .c

n a
.

Prove the formula

[axb, cxd, cxf]=[abd] [cef]-[abc] [def]


[abe] [fcd]-[abf ] [ecd]

(a)

Ex. 2 P. 148,

~[cda][bef]-[cdb][aefj. (b) Ex. 6 P. 153.

Agra University Solved Papers


*957 Prove the following by vector methods

221

1.

(a)

The

internal

bisectors of the angles of a triangle

are concurrent.
(b)

The

area of

the

triangle

formed by joining the

mid,
to

point of one of the non-parallel sides of a trapezium the extremities of the opposite side is half that of the

trapezium.
(a)

Ex. 2 P. 53

2. (a)

Show

in

terms

of

; (b) Q,. 12 P. 132. that any given vector r can be expressed three given non-coplanar vectors a, ft y in the

form

(b)

Prove that [axb, bxc, cXa]=[abc]2 and express


8 P. 143-1449 (b) Cor. P. 146. Explain the terms scalar and vector products

the result by means of determinants.


(a)
ft.

(a)

of

two
(b)

vectors, giving illustrations.

Prove the following formula for the volume V of a tetrahedron, in terms of the lengths of three concurrent
edges and their mutual inclinations 1 cos
30
:

<fr

cos

cos

<

cos 6
1

cosJ*
(a)

cos

2 P. 99

and

3 P. 118, (b) Ex. 2 P. 212.

(a)

Define 'centroid'.

Show

that the

centroid

is

independent of the origin of vectors.


(b)

Two

forces

act at the corner A of a

quadrilateral

222

Vector Analysis

ABCD,

represented by -*

AB and AD
Show

and two
their

at

repreis

sented by

CB and CD.
,

that

resultant

represented by

4PQ where
37,

P, Q, a * e

t' ie

mid. points of AC,

BD respectively.
(a)
(*>)

aP.
ft'

4 P-

4-

5 (a) P. 30.

If any within a tetrahedron ABCD is pDint joined to the vertices, and AO, BO, CO, DO are produced to cut the opposite faces in P, R, S respectively, then show that
2. (a)
,

Z z
(b)

OP
J/--

are a,
,

b and 3a-2b are


Ex.
i

Prove that the three points whose position vectors


colliuear.

(a)

8. (a)

P. 89; (b) Q,. 26 (b) P. 77. Define (1) the scalar, ('2) the vector product of

two

vectors,

and give instances

of

their

application to

mechanics.

is

Find the straight line, through the point Cj which parallel to the plane r . a=0, and intersects the line
(b)

r-a'^b.
2 P. 98

and

3 P. 119

and see Q. 14

(b) P. 185*

1.

(a)

Prove bo vector method the following If a line be drawn parallel to the base of a

triangle,

the line which joins the opposite vertex to the intersection of the diagonals of the trapezoid thus formed bisects the base.

i- j-f 3k and B (i+j- k) are equidistant from the plane r . (si+2j-7k)=0, and are on oppposite
(b)

The

points

sides of

it.

(a)

Ex. 22 P. 72. (b)

5 P. iC6

and Ex.

3. (b)

P. 168.

Agra University Solved Papers


2.

223

Prove that

CB, and that the volume of the tetrahedron bounded by the


. .

(BxC)x A=A

BC-A

four planes

r
r
is
.

(mj-fnk)=sO, r
.

(/i+mj)=0 and r

(li+wj+nk)=p

Zpsfilmn.

6 P. 140, Ex. i P. 211. Prove that the shortest distance between the two opposite edges of a regular tetrahedron is equal to half the diagonal of the square described on an edge.
3. (a)

(b)

Establish the vector formula

[axpbxq cXr]+[aXq bxr cXp] +[aXr bXp cxq]=0.


(a)

Ex. 22 (d) P. 188.

(b)

Ex. 7 P. 153.

1960 Find the vector equation to a sphere. Prove also to intersect that any straight line drawn from a point a sphere is cut harmonically by the surface and the polar
1. (a)

plane of 0.
(b)

Find the equation of the straight

line

through the

point

d and

equally inclined to the vectors a, b, c in the

form

+T+T)
and
,

,,

^ J. Ex. 19 P. 208, (b) Ex. 4 P. 124. 12 P. 191, (a) What do you understand by a system of reciproB. (a) cal vectors ? Show that any given vector r can be expressed in terms of three given non-coplanar vectors a, /?, y in the form [rffy] a+[rya] /?4-[rafl y

---

*(cxa)+*(axbn - ------

r^d+5^

(b)

Prove that

[aXb bXc cXa]=[abc]2 and


of determinants.

express

the result
(a)

by means

9 P. 144* (b) Cor. P. 146.

224
3. (a)

Vector Analysis

Prove the formula


to

(bxc).(axd)+(cxa),(bxd)+(axb).(cxd)=0
and use
it

show

that
sin

sin

(A+B)

M-B)=sina

^4-sin 2 5.

Prove that the shortest distances between a diago(b) nal of a rectangular parallelepiped whose sides are a, b, c and the edges not meeting it, are
be
ca

ab

(a)

Ex. iz P. 154, (b) Ex. 7 P. 180.


1961
:

Prove by vector methods the following If any within a tetrahedron ABCD is (a) point joined to the vertices, and AO, BO, CO, DO are produced to cut the opposite faces in P, Q,, R, S respectively, then prove
1.

(b) If

through any point within a triangle, lines be drawn

sides, show that the these lines to their corresponding sides

parallel

to the

sum
is 2.

of the

ratios of

Ex.

2. (a)

P. 89, Ex. 22 (b), P, 73. Show that the line of intersection of

r
is

(i+2j+Pk)*0 and r . (8i+2j+k)-0


to i

equally inclined
j.

and

k and makes

an angle i sec^1 3
line of

with

(b)

Find by vector method, the equation to the

intersection of
(a)

two given

3. (a)

Ex. 5 P. 182, Find the equation of the straight


c,

planes. 8 P. 171. (b)

line

through

the point

intersecting both the lines

r-a=5b
(b)

and r

a'=*/b'.

Prove the formulae

[axb, cxd, exf][abe] [fcd]-[abf] [ecd]


[cda] [bef J-[cdb] [aef ].
(a)

Ex. 14 P. 184,

(b) Ex. 6 P. 153.

RAJPUTANA UNIVERSITY SOLVED PAPERS


1959
1.

Define scalar triple product and prove the follow-

ing

(a)

cyclic

permutation of
of

three

vectors does not

change the value


cyclic

the

scalar triple product but


in

an

anti-

permutation changes the value

sign

but not in

magnitude.
(b)

The

position of

a dot and cross can be


its

inter-

changed without changing

value.

4 and 5 page 134-136.


2.

If p, q,

r are three vectors defined by the relation


^

~(ibcj'

where

a,

b,

c are vectors and the scalar terms of p,


3

(abc)9^0, prove that (abc)


a,

b and
9

in

(pqr)l and q and r.


1960

triple product obtain the values of

and Property

page 144-145.

1 00%
1. (a)

of the questions were set from this book.

that every vector can be represented as a linear combination of two non-collinear vectors coplanar with
the original vector.

Show

vectors that the stiayht line joining the middle points of two sides of a triangle is parallel to the third side and is of half its length.
(b)

Show by

the

method

of

(a)

6 page 14, (b) Ex. 19 page 72.

PUNJAB UNIVERSITY SOLVED PAPERS


1960

100% of the questions were set from this book.


1.

(a)

The

necessary

and

sufficient condition for four

points with position vectors there exist four scalars x,y,

a, b, c, d,
,

to be coplanar
all

is

that

/,

not

zero, such that

the internal bisector of any angle of a the opposite side internally in the ratio of triangle the sides containing the angle.
(b)

Show

that

divides

Show that the external bisectors of the three plane (c) angles of each trihedron of a given tetrahedron are coplanar.
(a)
2. (a)

10 page 86, (b) Ex. 3 page 57.


Define the
vector

product of two vectors and

prove that

/-

a*\
(b)

bX.c

-A /-> -\- /-> -VJ=\a c ) b-\a .b )


.

c.

Find the condition that

and interpret it geometrically. Prove that (c)

a,b
(&)

+ b.c

a=*2c

b,

3 P.

118,

6 P. 140, (b)

Expand both

sides

by

(a), (c)

Cor. P. 146.

Punjab University Solved Paperk


3. (a)

227

Show

that

-/- -\ -- -*a\ b+c J=a.b+a.c.


(b)

Find the distance of the point,

a,

from the plane

nq
(c)

measured

parallel to the line

rb+t c.

Show

that the middle point of the hypotenuse of a


is

right-angled triangle
(a)
4. (a)

equidistant from its vertices.

9 P. xoo, (b) Cor. P. 168, (c) Ex. 10 P. 109.

Show

and cross may


(b)

product, the dot be interchanged without changing the result.


triple
c,

that in the scalar

If a, b>

d be four vectors, express d

as

linear

combination of the three non-coplanar vectors

a, b, c.

Hence prove

that

-->-->
q.q'
f
.

P-P' P-9' P**'

q.p'

r'

jf

r.r'

where

are any vectors. 6 (iii) P. 16, (c) Ex. 2 P. 148. 5 P. 135, 136, (b) Show that the equation of the plane through two 5. (a)
/>,

r' q' r'; p' q'

given points A, -

with position vectors

a,

and

parallel to

given vector

c is

t(.

228
(b)

Vector Analysis

Show

that the lines

intersect
(c)

and

find the point of intersection.

The
any

position vectors

of

four points

-4,

5, C,

relative to

origin

are denoted

by

0, b,

c,

d.

Interpret

geometrically the equations

((i)

~*\

(-> ~*\

->

\a-bJx\cxdJ~Q,
\a-b
(b)

(ii)

(a)
'(c)

Result IV page 164,

Ex* 10 page 183,

Ex. 12 page 184.

Potrebbero piacerti anche